You are on page 1of 205

Question 1 of 438

A 72 year old man has a fall. He is found to have a fractured neck of femur and goes on to have a left hip
hemiarthroplasty. Two months post operatively he is found to have an odd gait. When standing on his left leg
his pelvis dips on the right side. There is no foot drop. What is the cause?

A. Sciatic nerve damage

B. L5 radiculopathy

C. Inferior gluteal nerve damage

D. Previous poliomyelitis

E. Superior gluteal nerve damage

Theme from 2010 Exam

This patient has a trendelenburg gait caused by damage to the superior gluteal nerve causing weakness of the
abductor muscles. Classically a patient is asked to stand on one leg and the pelvis dips on the opposite side. The
absence of a foot drop excludes the possibility of polio or L5 radiculopathy.

Gluteal region

Gluteal muscles

• Gluteus maximus: inserts to gluteal tuberosity of the femur and iliotibial tract
• Gluteus medius: attach to lateral greater trochanter
• Gluteus minimis: attach to anterior greater trochanter
• All extend and abduct the hip

Deep lateral hip rotators

• Piriformis
• Gemelli
• Obturator internus
• Quadratus femoris

Nerves
Superior gluteal nerve (L5, S1) • Gluteus medius
• Gluteus minimis
• Tensor fascia lata

Inferior gluteal nerve Gluteus maximus


Damage to the superior gluteal nerve will result in the patient developing a Trendelenberg gait. Affected
patients are unable to abduct the thigh at the hip joint. During the stance phase, the weakened abductor muscles
allow the pelvis to tilt down on the opposite side. To compensate, the trunk lurches to the weakened side to
attempt to maintain a level pelvis throughout the gait cycle. The pelvis sags on the opposite side of the lesioned
superior gluteal nerve.

Question 2 of 438
Which of the following structures lies posterior to the femoral nerve in the femoral triangle?

A. Adductor longus

B. Pectineus

C. Psoas major

D. Iliacus

E. None of the above

The iliacus lies posterior to the femoral nerve in the femoral triangle. The femoral sheath lies anterior to the
iliacus and pectineus muscles.

Femoral nerve

Root values L2, 3, 4


Innervates • Pectineus
• Sartorius
• Quadriceps femoris
• Vastus lateralis/medialis/intermedius

Branches • Medial cutaneous nerve of thigh


• Saphenous nerve
• Intermediate cutaneous nerve of thigh

Path
Penetrates psoas major and exits the pelvis by passing under the inguinal ligament to enter the femoral triangle,
lateral to the femoral artery and vein.
Image sourced from Wikipedia

Mnemonic for femoral nerve supply

(don't) M I S V Q Scan for PE


M edial cutaneous nerve of the thigh
I ntermediate cutaneous nerve of the thigh
S aphenous nerve

V astus
Q uadriceps femoris
S artorius

PE ectineus

Question 3 of 438
You are assisting in an open right adrenalectomy for a large adrenal adenoma. The consultant is distracted and
you helpfully pull the adrenal into the wound to improve the view. Unfortunately this is followed by brisk
bleeding. The vessel responsible for this is most likely to be:

A. Portal vein

B. Phrenic vein
C. Right renal vein

D. Superior mesenteric vein

E. Inferior vena cava

It drains directly via a very short vessel. If the sutures are not carefully tied then it may be avulsed off the IVC.
An injury best managed using a Satinsky clamp and a 6/0 prolene suture.

Adrenal gland anatomy

Anatomy

Location Superomedially to the upper pole of each kidney


Relationships of the right Diaphragm-Posteriorly, Kidney-Inferiorly, Vena Cava-Medially, Hepato-renal
adrenal pouch and bare area of the liver-Anteriorly
Relationships of the left Crus of the diaphragm-Postero- medially, Pancreas and splenic vessels-Inferiorly,
adrenal Lesser sac and stomach-Anteriorly
Superior adrenal arteries- from inferior phrenic artery, Middle adrenal arteries -
Arterial supply from aorta, Inferior adrenal arteries -from renal arteries
Venous drainage of the Via one central vein directly into the IVC
right adrenal
Venous drainage of the Via one central vein into the left renal vein
left adrenal

Question 4 of 438
A 28 year old lady requires an episiotomy for a ventouse vaginal delivery. Which of the nerves listed below will
usually be anaesthetised to allow the episiotomy?

A. Femoral

B. Ilioinguinal

C. Pudendal

D. Genitofemoral

E. Sacral plexus

Theme from April 2013 Exam


The pudendal nerve innervates the posterior vulval area and is routinely blocked in procedures such as
episiotomy.

Pudendal nerve

The pudendal nerve arises from nerve roots S2, S3 and S4 and exits the pelvis through the greater sciatic
foramen. It re-enters the pelvis through the lesser sciatic foramen. It travels inferior to give innervation to the
anal sphincters and external urethral sphincter. It also provides cutaneous innervation to the region of perineum
surrounding the anus and posterior vulva.

Traction and compression of the pudendal nerve by the foetus in late pregnancy may result in late onset
pudendal neuropathy which may be part of the process involved in the development of faecal incontinence.

Question 5 of 438
A motorcyclist is involved in a road traffic accident. He suffers a complex humeral shaft fracture which is
plated. Post operatively he complains of an inability to extend his fingers. Which of the following structures is
most likely to have been injured?

A. Ulnar nerve

B. Radial nerve

C. Median nerve

D. Axillary nerve

E. None of the above


Mnemonic for radial nerve muscles:
BEST

B rachioradialis
E xtensors
S upinator
T riceps

The radial nerve is responsible for innervation of the extensor compartment of the forearm.

Radial nerve

Continuation of posterior cord of the brachial plexus (root values C5 to T1)

Path

• In the axilla: lies posterior to the axillary artery on subscapularis, latissimus dorsi and teres major.
• Enters the arm between the brachial artery and the long head of triceps (medial to humerus).
• Spirals around the posterior surface of the humerus in the groove for the radial nerve.
• At the distal third of the lateral border of the humerus it then pierces the intermuscular septum and
descends in front of the lateral epicondyle.
• At the lateral epicondyle it lies deeply between brachialis and brachioradialis where it then divides into a
superficial and deep terminal branch.
• Deep branch crosses the supinator to become the posterior interosseous nerve.
In the image below the relationships of the radial nerve can be appreciated

Image sourced from Wikipedia

Regions innervated
Motor (main nerve) • Triceps
• Anconeus
• Brachioradialis
• Extensor carpi radialis

Motor (posterior • Extensor carpi ulnaris


interosseous branch • Extensor digitorum
• Extensor indicis
• Extensor digiti minimi
• Extensor pollicis longus and brevis
• Abductor pollicis longus

Sensory The area of skin supplying the proximal phalanges on the dorsal aspect of the hand is
supplied by the radial nerve (this does not apply to the little finger and part of the ring
finger)

Muscular innervation and effect of denervation


Anatomical Muscle affected Effect of paralysis
location
Shoulder Long head of triceps Minor effects on shoulder stability in abduction
Arm Triceps Loss of elbow extension
Forearm Supinator Weakening of supination of prone hand and elbow flexion
Brachioradialis in mid prone position
Extensor carpi radialis longus
and brevis
The cutaneous sensation of the upper limb- illustrating the contribution of the radial nerve

Question 6 of 438
An enthusiastic surgical registrar undertakes his first solo splenectomy. The operation is far more difficult than
anticipated and the registrar leaves a tube drain to the splenic bed at the end of the procedure. Over the
following 24 hours approximately 500ml of clear fluid has entered the drain. Biochemical testing of the fluid is
most likely to reveal:

A. Elevated creatinine

B. Elevated triglycerides

C. Elevated glucagon

D. Elevated amylase

E. None of the above

During splenectomy the tail of the pancreas may be damaged. The pancreatic duct will then drain into the
splenic bed, amylase is the most likely biochemical finding. Glucagon is not secreted into the pancreatic duct.

Splenic anatomy

The spleen is the largest lymphoid organ in the body. It is an intraperitoneal organ, the peritoneal attachments
condense at the hilum where the vessels enter the spleen. Its blood supply is from the splenic artery (derived
from the coeliac axis) and the splenic vein (which is joined by the IMV and unites with the SMV).

• Embryology: derived from mesenchymal tissue


• Shape: clenched fist
• Position: below 9th-12th ribs
• Weight: 75-150g

Relations

• Superiorly- diaphragm
• Anteriorly- gastric impression
• Posteriorly- kidney
• Inferiorly- colon
• Hilum: tail of pancreas and splenic vessels
• Forms apex of lesser sac (containing short gastric vessels)

Question 7 of 438
A 48 year old lady is undergoing an axillary node clearance for breast cancer. Which of the structures listed
below are most likely to be encountered during the axillary dissection?

A. Cords of the brachial plexus

B. Thoracodorsal trunk

C. Internal mammary artery

D. Thoracoacromial artery

E. None of the above


Beware of damaging the thoracodorsal trunk if a latissimus dorsi flap reconstruction is planned.

Theme from 2009 Exam


The thoracodorsal trunk runs through the nodes in the axilla. If injured it may compromise the function and
blood supply to latissimus dorsi, which is significant if it is to be used as a flap for a reconstructive procedure.

Axilla

Boundaries of the axilla


Medially Chest wall and Serratus anterior
Laterally Humeral head
Floor Subscapularis
Anterior aspect Lateral border of Pectoralis major
Fascia Clavipectoral fascia

Content:
Long thoracic nerve (of Derived from C5-C7 and passes behind the brachial plexus to enter the axilla. It
Bell) lies on the medial chest wall and supplies serratus anterior. Its location puts it at
risk during axillary surgery and damage will lead to winging of the scapula.
Thoracodorsal nerve and Innervate and vascularise latissimus dorsi.
thoracodorsal trunk
Axillary vein Lies at the apex of the axilla, it is the continuation of the basilic vein. Becomes
the subclavian vein at the outer border of the first rib.
Intercostobrachial nerves Traverse the axillary lymph nodes and are often divided during axillary surgery.
They provide cutaneous sensation to the axillary skin.
Lymph nodes The axilla is the main site of lymphatic drainage for the breast.

Question 8 of 438
A 53 year old lady is recovering following a difficult mastectomy and axillary nodal clearance for carcinoma of
the breast. She complains of shoulder pain and on examination has obvious winging of the scapula. Loss of
innervation to which of the following is the most likely underlying cause?

A. Latissimus dorsi

B. Serratus anterior

C. Pectoralis minor

D. Pectoralis major

E. Rhomboids

Theme from April 2012 Exam


Winging of the scapula is most commonly the result of long thoracic nerve injury or dysfunction. Iatrogenic
damage during the course of the difficult axillary dissection is the most likely cause in this scenario. Damage to
the rhomboids may produce winging of the scapula but would be rare in the scenario given.

Long thoracic nerve

• Derived from ventral rami of C5, C6, and C7 (close to their emergence from intervertebral foramina)
• It runs downward and passes either anterior or posterior to the middle scalene muscle
• It reaches upper tip of serratus anterior muscle and descends on outer surface of this muscle, giving
branches into it
• Winging of Scapula occurs in long thoracic nerve injury (most common) or from spinal accessory nerve
injury (which denervates the trapezius) or a dorsal scapular nerve injury

Question 9 of 438
A 56 year old lady is referred to the colorectal clinic with symptoms of pruritus ani. On examination a
polypoidal mass is identified inferior to the dentate line. A biopsy confirms squamous cell carcinoma. To which
of the following lymph node groups will the lesion potentially metastasise?

A. Internal iliac

B. External iliac

C. Mesorectal
D. Inguinal

E. None of the above

Theme from September 2011 Exam


Theme from April 2012 Exam
Lesions distal to the dentate line drain to the inguinal nodes. Occasionally this will result in the need for a block
dissection of the groin.

Rectum

The rectum is approximately 12 cm long. It is a capacitance organ. It has both intra and extraperitoneal
components. The transition between the sigmoid colon is marked by the disappearance of the tenia coli.The
extra peritoneal rectum is surrounded by mesorectal fat that also contains lymph nodes. This mesorectal fatty
layer is removed surgically during rectal cancer surgery (Total Mesorectal Excision). The fascial layers that
surround the rectum are important clinical landmarks, anteriorly lies the fascia of Denonvilliers. Posteriorly lies
Waldeyers fascia.

Extra peritoneal rectum

• Posterior upper third


• Posterior and lateral middle third
• Whole lower third

Relations
Anteriorly (Males) Rectovesical pouch
Bladder
Prostate
Seminal vesicles
Anteriorly (Females) Recto-uterine pouch (Douglas)
Cervix
Vaginal wall
Posteriorly Sacrum
Coccyx
Middle sacral artery
Laterally Levator ani
Coccygeus

Arterial supply
Superior rectal artery

Venous drainage
Superior rectal vein

Lymphatic drainage

• Mesorectal lymph nodes (superior to dentate line)


• Internal iliac and then para-aortic nodes
• Inguinal nodes (inferior to dentate line)

Question 10 of 438
A 20 year old man is hit with a hammer on the right side of the head. He dies on arrival in the emergency
department. Which of these features is most likely to be found at post mortem?

A. Hydrocephalus

B. Supra tentorial herniation

C. Laceration of the middle meningeal artery

D. Sub dural haematoma

E. Posterior fossa haematoma

Theme based on 2011 exam


This will account for the scenario given where there is a brief delay prior to death. The other options are less
acute and a supratentorial herniation would not occur in this setting.

Head injury

• Patients who suffer head injuries should be managed according to ATLS principles and extra cranial
injuries should be managed alongside cranial trauma. Inadequate cardiac output will compromise CNS
perfusion irrespective of the nature of the cranial injury.

Types of traumatic brain injury


Extradural Bleeding into the space between the dura mater and the skull. Often results from
haematoma acceleration-deceleration trauma or a blow to the side of the head. The majority of
extradural haematomas occur in the temporal region where skull fractures cause a rupture
of the middle meningeal artery.

Features

• Raised intracranial pressure


• Some patients may exhibit a lucid interval

Subdural Bleeding into the outermost meningeal layer. Most commonly occur around the frontal and
haematoma parietal lobes. May be either acute or chronic.

Risk factors include old age and alcoholism.

Slower onset of symptoms than a extradural haematoma.


Subarachnoid Usually occurs spontaneously in the context of a ruptured cerebral aneurysm but may be
haemorrhage seen in association with other injuries when a patient has sustained a traumatic brain injury
Pathophysiology

• Primary brain injury may be focal (contusion/ haematoma) or diffuse (diffuse axonal injury)
• Diffuse axonal injury occurs as a result of mechanical shearing following deceleration, causing
disruption and tearing of axons
• Intra-cranial haematomas can be extradural, subdural or intracerebral, while contusions may occur
adjacent to (coup) or contralateral (contre-coup) to the side of impact
• Secondary brain injury occurs when cerebral oedema, ischaemia, infection, tonsillar or tentorial
herniation exacerbates the original injury. The normal cerebral auto regulatory processes are disrupted
following trauma rendering the brain more susceptible to blood flow changes and hypoxia
• The Cushings reflex (hypertension and bradycardia) often occurs late and is usually a pre terminal event

Management

• Where there is life threatening rising ICP such as in extra dural haematoma and whilst theatre is
prepared or transfer arranged use of IV mannitol/ frusemide may be required.
• Diffuse cerebral oedema may require decompressive craniotomy
• Exploratory Burr Holes have little management in modern practice except where scanning may be
unavailable and to thus facilitate creation of formal craniotomy flap
• Depressed skull fractures that are open require formal surgical reduction and debridement, closed
injuries may be managed non operatively if there is minimal displacement.
• ICP monitoring is appropriate in those who have GCS 3-8 and normal CT scan.
• ICP monitoring is mandatory in those who have GCS 3-8 and abnormal CT scan.
• Hyponatraemia is most likely to be due to syndrome of inappropriate ADH secretion.
• Minimum of cerebral perfusion pressure of 70mmHg in adults.
• Minimum cerebral perfusion pressure of between 40 and 70 mmHg in children.

Interpretation of pupillary findings in head injuries


Pupil size Light response Interpretation
Unilaterally dilated Sluggish or fixed 3rd nerve compression secondary to tentorial
herniation
Bilaterally dilated Sluggish or fixed • Poor CNS perfusion
• Bilateral 3rd nerve palsy

Unilaterally dilated or Cross reactive (Marcus - Optic nerve injury


equal Gunn)
Bilaterally constricted May be difficult to assess • Opiates
• Pontine lesions
• Metabolic encephalopathy

Unilaterally constricted Preserved Sympathetic pathway disruption

Question 11 of 438
Which of the following ligaments contains the artery supplying the head of femur in children?
A. Transverse ligament

B. Ligamentum teres

C. Iliofemoral ligament

D. Ischiofemoral ligament

E. Pubofemoral ligament

Theme from 2010 Exam

Hip joint

• Head of femur articulates with acetabulum of the pelvis


• Both covered by articular hyaline cartilage
• The acetabulum forms at the union of the ilium, pubis, and ischium
• The triradiate cartilage (Y-shaped growth plate) separates the pelvic bones
• The acetabulum holds the femoral head by the acetabular labrum
• Normal angle between femoral head and femoral shaft is 130o

Ligaments

• Transverse ligament: joints anterior and posterior ends of the articular cartilage
• Head of femur ligament (ligamentum teres): acetabular notch to the fovea. Contains arterial supply to
head of femur in children.

Image sourced from Wikipedia


Image sourced from Wikipedia

Extracapsular ligaments

• Iliofemoral ligament: inverted Y shape. Anterior iliac spine to the trochanteric line
• Pubofemoral ligament: acetabulum to lesser trochanter
• Ischiofemoral ligament: posterior support. Ischium to greater trochanter.

Blood supply
Medial circumflex femoral and lateral circumflex femoral arteries (Branches of profunda femoris)

2 anastomoses: Cruciate and the trochanteric anastomoses (provides most of the blood to the head of the femur)
Hence the need for hemiarthroplasty when there is a displaced femoral head fracture. These anastomoses exist
between the femoral artery or profunda femoris and the gluteal vessels.

Question 12 of 438
A 72 year old man develops a hydrocele which is being surgically managed. As part of the procedure the
surgeons divide the tunica vaginalis. From which of the following is this structure derived?

A. Peritoneum

B. External oblique aponeurosis

C. Internal oblique aponeurosis

D. Transversalis fascia
E. Rectus sheath

The tunica vaginalis is derived from peritoneum, it secretes the fluid that fills the hydrocele cavity.

Scrotal and testicular anatomy

Spermatic cord
Formed by the vas deferens and is covered by the following structures:
Layer Origin
Internal spermatic fascia Transversalis fascia
Cremasteric fascia From the fascial coverings of internal oblique
External spermatic fascia External oblique aponeurosis

Contents of the cord


Vas deferens Transmits sperm and accessory gland secretions
Testicular artery Branch of abdominal aorta supplies testis and epididymis
Artery of vas deferens Arises from inferior vesical artery
Cremasteric artery Arises from inferior epigastric artery
Pampiniform plexus Venous plexus, drains into right or left testicular vein
Sympathetic nerve fibres Lie on arteries, the parasympathetic fibres lie on the vas
Genital branch of the genitofemoral nerve Supplies cremaster
Lymphatic vessels Drain to lumbar and para-aortic nodes

Scrotum

• Composed of skin and closely attached dartos fascia.


• Arterial supply from the anterior and posterior scrotal arteries
• Lymphatic drainage to the inguinal lymph nodes
• Parietal layer of the tunica vaginalis is the innermost layer

Testes

• The testes are surrounded by the tunica vaginalis (closed peritoneal sac). The parietal layer of the tunica
vaginalis adjacent to the internal spermatic fascia.
• The testicular arteries arise from the aorta immediately inferiorly to the renal arteries.
• The pampiniform plexus drains into the testicular veins, the left drains into the left renal vein and the
right into the inferior vena cava.
• Lymphatic drainage is to the para-aortic nodes.

Question 13 of 438
A 23 year old man falls and slips at a nightclub. A shard of glass penetrates the skin at the level of the medial
epicondyle, which of the following sequelae is least likely to occur?

A. Atrophy of the first dorsal interosseous muscle


B. Difficulty in abduction of the the 2nd, 3rd, 4th and 5th fingers

C. Claw like appearance of the hand

D. Loss of sensation on the anterior aspect of the 5th finger

E. Partial denervation of flexor digitorum profundus


Injury to the ulnar nerve in the mid to distal forearm will typically produce a claw hand. This consists of
flexion of the 4th and 5th interphalangeal joints and extension of the metacarpophalangeal joints. The effects
are potentiated when flexor digitorum profundus is not affected, and the clawing is more pronounced.More
proximally sited ulnar nerve lesions produce a milder clinical picture owing to the simultaneous paralysis of
flexor digitorum profundus (ulnar half).

This is the 'ulnar paradox', due to the more proximal level of transection the hand will typically not have a claw
like appearance that may be seen following a more distal injury. The first dorsal interosseous muscle will be
affected as it is supplied by the ulnar nerve.

Ulnar nerve

Origin

• C8, T1

Supplies (no muscles in the upper arm)

• Flexor carpi ulnaris


• Flexor digitorum profundus
• Flexor digiti minimi
• Abductor digiti minimi
• Opponens digiti minimi
• Adductor pollicis
• Interossei muscle
• Third and fourth lumbricals
• Palmaris brevis

Path

• Posteromedial aspect of ulna to flexor compartment of forearm, then along the ulnar. Passes beneath the
flexor carpi ulnaris muscle, then superficially through the flexor retinaculum into the palm of the hand.
Image sourced from Wikipedia

Branches
Branch Supplies
Articular branch Flexor carpi ulnaris
Medial half of the flexor digitorum profundus
Palmar cutaneous branch (Arises near the middle Skin on the medial part of the palm
of the forearm)
Dorsal cutaneous branch Dorsal surface of the medial part of the hand
Superficial branch Cutaneous fibres to the anterior surfaces of the medial one
and one-half digits
Deep branch Hypothenar muscles
All the interosseous muscles
Third and fourth lumbricals
Adductor pollicis
Medial head of the flexor pollicis brevis

Effects of injury
Damage at the wrist • Wasting and paralysis of intrinsic hand muscles (claw hand)
• Wasting and paralysis of hypothenar muscles
• Loss of sensation medial 1 and half fingers

Damage at the elbow • Radial deviation of the wrist


• Clawing less in 3rd and 4th digits

Question 14 of 438
A 43 year old lady is donating her left kidney to her sister and the surgeons are harvesting the left kidney.
Which of the following structures will lie most anteriorly at the hilum of the left kidney?

A. Left renal artery

B. Left renal vein

C. Left ureter

D. Left ovarian vein

E. Left ovarian artery

The renal veins lie most anteriorly, then artery and ureter lies posteriorly.

Renal arteries

• The right renal artery is longer than the left renal artery
• The renal vein/artery/pelvis enter the kidney at the hilum

Relations

• Right:

Anterior- IVC, right renal vein, the head of the pancreas, and the descending part of the duodenum.

• Left:

Anterior- left renal vein, the tail of the pancreas.

Branches

• The renal arteries are direct branches off the aorta (upper border of L2)
• In 30% there may be accessory arteries (mainly left side). Instead of entering the kidney at the hilum,
they usually pierce the upper or lower part of the organ.
• Before reaching the hilum of the kidney, each artery divides into four or five segmental branches (renal
vein anterior and ureter posterior); which then divide within the sinus into lobar arteries supplying each
pyramid and cortex.
• Each vessel gives off some small inferior suprarenal branches to the suprarenal gland, the ureter, and the
surrounding cellular tissue and muscles.

Question 15 of 438
What is the sensory nerve supply to the angle of the jaw?

A. Maxillary branch of the trigeminal nerve

B. Mandibular branch of the trigeminal nerve

C. C3-C4

D. Greater auricular nerve (C2-C3)

E. Buccal branch of the facial nerve

The trigeminal nerve is the major sensory nerve to the face except over the angle of the jaw. The angle of the
jaw is innervated by the greater auricular nerve.

Trigeminal nerve

The trigeminal nerve is the main sensory nerve of the head. In addition to its major sensory role, it also
innervates the muscles of mastication.

Distribution of the trigeminal nerve


Sensory • Scalp
• Face
• Oral cavity (and teeth)
• Nose and sinuses
• Dura mater

Motor • Muscles of mastication


• Mylohyoid
• Anterior belly of digastric
• Tensor tympani
• Tensor palati

Autonomic connections (ganglia) • Ciliary


• Sphenopalatine
• Otic
• Submandibular

Path

• Originates at the pons


• Sensory root forms the large, crescentic trigeminal ganglion within Meckel's cave, and contains the cell
bodies of incoming sensory nerve fibres. Here the 3 branches exit.
• The motor root cell bodies are in the pons and the motor fibres are distributed via the mandibular nerve.
The motor root is not part of the trigeminal ganglion.
Branches of the trigeminal nerve
Ophthalmic nerve Sensory only
Maxillary nerve Sensory only
Mandibular nerve Sensory and motor

Sensory
Ophthalmic Exits skull via the superior orbital fissure
Sensation of: scalp and forehead, the upper eyelid, the conjunctiva and cornea of the eye, the
nose (including the tip of the nose, except alae nasi), the nasal mucosa, the frontal sinuses, and
parts of the meninges (the dura and blood vessels).
Maxillary Exit skull via the foramen rotundum
nerve Sensation: lower eyelid and cheek, the nares and upper lip, the upper teeth and gums, the nasal
mucosa, the palate and roof of the pharynx, the maxillary, ethmoid and sphenoid sinuses, and
parts of the meninges.
Mandibular Exit skull via the foramen ovale
nerve Sensation: lower lip, the lower teeth and gums, the chin and jaw (except the angle of the jaw),
parts of the external ear, and parts of the meninges.

Motor
Distributed via the mandibular nerve.
The following muscles of mastication are innervated:

• Masseter
• Temporalis
• Medial pterygoid
• Lateral pterygoid

Other muscles innervated include:

• Tensor veli palatini


• Mylohyoid
• Anterior belly of digastric
• Tensor tympani

Question 16 of 438
A 63 year old man is undergoing a coronary artery bypass procedure. During the median sternotomy which
structure would routinely require division?

A. Parietal pleura

B. Interclavicular ligament

C. Internal mammary artery

D. Brachiocephalic vein

E. Left vagus nerve


Theme from January 2011 Exam
The interclavicular ligament lies at the upper end of a median sternotomy and is routinely divided to provide
access. The pleural reflections are often encountered and should not be intentionally divided, if they are, then a
chest drain will need to be inserted on the affected side as collections may then accumulate in the pleural cavity.
Other structures encountered include the pectoralis major muscles, again if the incision is truly midline then
these should not require formal division. The close relationship of the brachiocephalic vein should be borne in
mind and it should be avoided, iatrogenic injury to this structure will result in considerable haemorrhage.

Sternotomy

A median sternotomy is the commonest incision utilised to access the heart and great vessels.
An midline incision is made from the interclavicular fossa to the xiphoid process and the fat and subcutaneous
tissues are divided to the level of the sternum. The periosteum may be gently mobilised off the midline,
although vigorous periosteal stripping is best avoided. A bone saw is used to divide the bone itself. Posteriorly
the reflections of the parietal pleura should be identified and avoided (unless surgery to the lung is planned).
The fibrous pericardium is incised and the heart brought into view. Bleeding from the bony edges of the cut
sternum is stopped using roller ball diathermy or bone wax. The left brachiocephalic vein is an important
posterior relation at the superior aspect of the sternotomy incision and should be avoided. More inferior the
thymic remnants may be identified. At the inferior aspect of the incision the abdominal cavity may be entered
(though this is seldom troublesome).

Question 17 of 438
Which of the following structures separates the subclavian artery from the subclavian vein?

A. Scalenus anterior

B. Scalenus medius

C. Sternocleidomastoid

D. Pectoralis major

E. Pectoralis minor

The artery and vein are separated by scalenus anterior. This muscle runs from the the transverse processes of
C3,4,5 and 6 to insert onto the scalene tubercle of the first rib.

Subclavian artery

Path

• The left subclavian comes directly off the arch of aorta


• The right subclavian arises from the brachiocephalic artery (trunk) when it bifurcates into the subclavian
and the right common carotid artery.
• From its origin, the subclavian artery travels laterally, passing between anterior and middle scalene
muscles, deep to scalenus anterior and anterior to scalenus medius. As the subclavian artery crosses the
lateral border of the first rib, it becomes the axillary artery. At this point it is superficial and within the
subclavian triangle.

Image sourced from Wikipedia

Branches

• Vertebral artery
• Internal thoracic artery
• Thyrocervical trunk
• Costocervical trunk
• Dorsal scapular artery

Question 18 of 438
A 56 year old lady is due to undergo a left hemicolectomy for carcinoma of the splenic flexure. The surgeons
decide to perform a high ligation of the inferior mesenteric vein. Into which of the following does this structure
usually drain?

A. Portal vein

B. Inferior vena cava

C. Left renal vein

D. Left iliac vein

E. Splenic vein
Beware of ureteric injury in colonic
surgery.

The IMV drains into the splenic vein.

Left colon
Position

• As the left colon passes inferiorly its posterior aspect becomes extraperitoneal, and the ureter and
gonadal vessels are close posterior relations that may become involved in disease processes
• At a level of L3-4 (variable) the left colon becomes the sigmoid colon and wholly intraperitoneal once
again
• The sigmoid colon is a highly mobile structure and may even lie of the right side of the abdomen
• It passes towards the midline, the taenia blend and this marks the transition between sigmoid colon and
upper rectum.

Blood supply

• Inferior mesenteric artery


• However, the marginal artery (from the right colon) contributes and this contribution becomes clinically
significant when the IMA is divided surgically (e.g. During AAA repair)

Question 19 of 438
A man undergoes a high anterior resection for carcinoma of the upper rectum. Which of the following vessels
will require ligation?

A. Superior mesenteric artery

B. Inferior mesenteric artery

C. Coeliac axis

D. Perineal artery

E. Middle colic artery

Theme from April 2013


The IMA is usually divided during anterior resection. Not only is this borne out of oncological necessity but it
also permits sufficient colonic mobilisation for anastomosis.

Colon anatomy

The colon commences with the caecum. This represents the most dilated segment of the human colon and its
base (which is intraperitoneal) is marked by the convergence of teniae coli. At this point is located the
vermiform appendix. The colon continues as the ascending colon, the posterior aspect of which is
retroperitoneal. The line of demarcation between the intra and retro peritoneal right colon is visible as a white
line, in the living, and forms the line of incision for colonic resections.
The ascending colon becomes the transverse colon after passing the hepatic flexure. At this located the colon
becomes wholly intra peritoneal once again. The superior aspect of the transverse colon is the point of
attachment of the transverse colon to the greater omentum. This is an important anatomical site since division of
these attachments permits entry into the lesser sac. Separation of the greater omentum from the transverse colon
is a routine operative step in both gastric and colonic resections.
At the left side of the abdomen the transverse colon passes to the left upper quadrant and makes an oblique
inferior turn at the splenic flexure. Following this, the posterior aspect becomes retroperitoneal once again.
At the level of approximately L4 the descending colon becomes wholly intraperitoneal and becomes the
sigmoid colon. Whilst the sigmoid is wholly intraperitoneal there are usually attachments laterally between the
sigmoid and the lateral pelvic sidewall. These small congenital adhesions are not formal anatomical attachments
but frequently require division during surgical resections.
At is distal end the sigmoid passes to the midline and at the region around the sacral promontary it becomes the
upper rectum. This transition is visible macroscopically as point where the teniae fuse. More distally the rectum
passes through the peritoneum at the region of the peritoneal reflection and becomes extraperitoneal.

Arterial supply
Superior mesenteric artery and inferior mesenteric artery: linked by the marginal artery.
Ascending colon: ileocolic and right colic arteries
Transverse colon: middle colic artery
Descending and sigmoid colon: inferior mesenteric artery

Venous drainage
From regional veins (that accompany arteries) to superior and inferior mesenteric vein

Lymphatic drainage
Initially along nodal chains that accompany supplying arteries, then para-aortic nodes.

Embryology
Midgut- Second part of duodenum to 2/3 transverse colon
Hindgut- Distal 1/3 transverse colon to anus

Peritoneal location
The right and left colon are part intraperitoneal and part extraperitoneal. The sigmoid and transverse colon are
generally wholly intraperitoneal. This has implications for the sequelae of perforations, which will tend to result
in generalised peritonitis in the wholly intra peritoneal segments.

Colonic relations
Region of colon Relation
Caecum/ right colon Right ureter, gonadal vessels
Hepatic flexure Gallbladder (medially)
Splenic flexure Spleen and tail of pancreas
Distal sigmoid/ upper rectum Left ureter
Rectum Ureters, autonomic nerves, seminal vesicles, prostate, urethra (distally)
Question 20 of 438
A 43 year old lady is due to undergo an axillary node clearance as part of treatment for carcinoma of the breast.
Which of the following fascial layers will be divided during the surgical approach to the axilla?

A. Sibsons fascia

B. Pre tracheal fascia

C. Waldayers fascia

D. Clavipectoral fascia
E. None of the above

The clavipectoral fascia is situated under the clavicular portion of pectoralis major. It protects both the axillary
vessels and nodes. During an axillary node clearance for breast cancer the clavipectoral fascia is incised and this
allows access to the nodal stations. The nodal stations are; level 1 nodes inferior to pectoralis minor, level 2 lie
behind it and level 3 above it. During a Patey Mastectomy surgeons divide pectoralis minor to gain access to
level 3 nodes. The use of sentinel node biopsy (and stronger assistants!) have made this procedure far less
common.

Axilla

Boundaries of the axilla


Medially Chest wall and Serratus anterior
Laterally Humeral head
Floor Subscapularis
Anterior aspect Lateral border of Pectoralis major
Fascia Clavipectoral fascia

Content:
Long thoracic nerve (of Derived from C5-C7 and passes behind the brachial plexus to enter the axilla. It
Bell) lies on the medial chest wall and supplies serratus anterior. Its location puts it at
risk during axillary surgery and damage will lead to winging of the scapula.
Thoracodorsal nerve and Innervate and vascularise latissimus dorsi.
thoracodorsal trunk
Axillary vein Lies at the apex of the axilla, it is the continuation of the basilic vein. Becomes
the subclavian vein at the outer border of the first rib.
Intercostobrachial nerves Traverse the axillary lymph nodes and are often divided during axillary surgery.
They provide cutaneous sensation to the axillary skin.
Lymph nodes The axilla is the main site of lymphatic drainage for the breast.

Question 21 of 438
What are the boundaries of the 'safe triangle' for chest drain insertion?

A. Bounded by trapezius, latissimus dorsi, and laterally by the vertebral


border of the scapula

B. Bounded by latissimus dorsi, pectoralis major, line superior to the


nipple and apex at the axilla

C. Bounded by latissimus dorsi, serratus anterior, line superior to the


nipple and apex at the axilla

D. Bounded by trapezius, deltoid, rhomboid major and teres minor

E. Bounded by trapezius, deltoid and latissimus dorsi


Theme from April 2012 Exam

Chest drains

There are a number of different indications for chest drain insertion. In general terms large bore chest drains are
preferred for trauma and haemothorax drainage. Smaller diameter chest drains can be used for pneumothorax or
pleural effusion drainage.

Insertion can be performed either using anatomical guidance or through ultrasound guidance. In the exam, the
anatomical method is usually tested.

It is advised that chest drains are placed in the 'safe triangle'. The triangle is located in the mid axillary line of
the 5th intercostal space. It is bordered by:
Anterior edge latissimus dorsi, the lateral border of pectoralis major, a line superior to the horizontal level of the
nipple, and the apex below the axilla.

Another triangle is situated behind the scapula. It is bounded above by the trapezius, below by the latissimus
dorsi, and laterally by the vertebral border of the scapula; the floor is partly formed by the rhomboid major. If
the scapula is drawn forward by folding the arms across the chest, and the trunk bent forward, parts of the sixth
and seventh ribs and the interspace between them become subcutaneous and available for auscultation. The
space is therefore known as the triangle of auscultation.

Question 22 of 438
The vertebral artery traverses all of the following except?

A. Transverse process of C6

B. Transverse process of the axis

C. Vertebral canal

D. Foramen magnum

E. Intervertebral foramen

The vertebral artery passes through the foramina which lie within the foramina of the transverse processes of
the cervical vertebral, not the intervertebral foramen.

Vertebral artery

The vertebral artery is the first branch of the subclavian artery. Anatomically it is divisible into 4 regions:

• The first part runs to the foramen in the transverse process of C6. Anterior to this part lies the vertebral
and internal jugular veins. On the left side the thoracic duct is also an anterior relation.
• The second part runs superiorly through the foramina of the the transverse processes of the upper 6
cervical vertebrae. Once it has passed through the transverse process of the axis it then turns
superolaterally to the atlas. It is accompanied by a venous plexus and the inferior cervical sympathetic
ganglion.
• The third part runs posteromedially on the lateral mass of the atlas. It enters the sub occipital triangle, in
the groove of the upper surface of the posterior arch of the atlas. It then passes anterior to the edge of the
posterior atlanto-occipital membrane to enter the vertebral canal.
• The fourth part passes through the spinal dura and arachnoid, running superiorly and anteriorly at the
lateral aspect of the medulla oblongata. At the lower border of the pons it unites to form the basilar
artery.

Question 23 of 438
A 60 year old female attends the preoperative hernia clinic. She reports some visual difficulty. On examination
she is noted to have a homonymous hemianopia. Where is the lesion most likely to be?

A. Frontal lobe

B. Pituitary gland

C. Parietal lobe

D. Optic chiasm

E. Optic tract
Lesions before optic chiasm:
Monocular vision loss = Optic nerve
lesion
Bitemporal hemianopia = Optic chiasm
lesion

Lesions after the optic chiasm:


Homonymous hemianopia = Optic tract
lesion
Upper quadranopia = Temporal lobe
lesion
Lower quadranopia = Parietal lobe
lesion

Theme from April 2012 exam


Unfortunately we thought as surgeons we could forget about visual field defects! However the college seem to
like them. Remember a homonymous hemianopia is indicative of an optic tract lesion. Parietal lobe lesions tend
to cause inferior quadranopias and there is a bitemporal hemianopia with optic chiasm lesion or pituitary
tumours.

Visual field defects

Theme from January 2012 exam


Theme from April 2012 exam

• left homonymous hemianopia means visual field defect to the left, i.e. Lesion of right optic tract
• homonymous quadrantanopias: PITS (Parietal-Inferior, Temporal-Superior)
• incongruous defects = optic tract lesion; congruous defects = optic radiation lesion or occipital cortex

Homonymous hemianopia

• Incongruous defects: lesion of optic tract


• Congruous defects: lesion of optic radiation or occipital cortex
• Macula sparing: lesion of occipital cortex

Homonymous quadrantanopias

• Superior: lesion of temporal lobe


• Inferior: lesion of parietal lobe
• Mnemonic = PITS (Parietal-Inferior, Temporal-Superior)

Bitemporal hemianopia

• Lesion of optic chiasm


• Upper quadrant defect > lower quadrant defect = inferior chiasmal compression, commonly a pituitary
tumour
• Lower quadrant defect > upper quadrant defect = superior chiasmal compression, commonly a
craniopharyngioma

Question 24 of 438
During an Ivor Lewis Oesophagectomy for carcinoma of the lower third of the oesophagus which structure is
divided to allow mobilisation of the oesophagus?

A. Vagus nerve

B. Azygos vein

C. Right inferior lobar bronchus

D. Phrenic nerve

E. Pericardiophrenic artery

The azygos vein is routinely divided during an oesophagectomy to allow mobilisation. It arches anteriorly to
insert into the SVC on the right hand side.

Treatment of oesophageal cancer

• In general resections are not offered to those patients with distant metastasis, and usually not to those
with N2 disease.
• Local nodal involvement is not in itself a contra indication to resection.
• Surgical resection is the mainstay of treatment.
• Neoadjuvent chemotherapy is given in most cases prior to surgery.
• In situ disease may be managed by endoscopic mucosal resection, although this is still debated.
• In patients with lower third lesions an Ivor - Lewis type procedure is most commonly performed. Very
distal tumours may be suitable to a transhiatal procedure. Which is an attractive option as the penetration
of two visceral cavities required for an Ivor- Lewis type procedure increases the morbidity considerably.
• More proximal lesions will require a total oesphagectomy (Mckeown type) with anastomosis to the
cervical oesophagus.
• Patients with unresectable disease may derive benefit from local ablative procedures, palliative
chemotherapy or stent insertion.

Operative details of Ivor- Lewis procedure

• Combined laparotomy and right thoracotomy

Indication

• Lower and middle third oesophageal tumours

Preparation

• Staging with a combination of CT chest abdomen and pelvis- if no metastatic disease detected then
patients will undergo a staging laparoscopy to detect peritoneal disease.
• If both these modalities are negative then patients will finally undergo a PET CT scan to detect occult
metastatic disease. Only in those whom no evidence of advanced disease is detected will proceed to
resection.
• Patients receive a GA, double lumen endotracheal tube to allow for lung deflation, CVP and arterial
monitoring.

Procedure

• A rooftop incision is made to access the stomach and duodenum.

Laparotomy To mobilize the stomach

• The greater omentum is incised away from its attachment to the right gastroepiploic vessels along the
greater curvature of the stomach.
• Then the short gastric vessels are ligated and detached from the greater curvature from the spleen.
• The lesser omentum is incised, preserving the right gastric artery.
• The retroperitoneal attachments of the duodenum in its second and third portions are incised, allowing
the pylorus to reach the oesophageal hiatus. Some surgeons perform a pyloroplasty at this point to
facilitate gastric emptying.
• The left gastric vessels are then ligated, avoiding any injury to the common hepatic or splenic arteries.
Care must be taken to avoid inadvertently devascularising the liver owing to variations in anatomy.

Right Thoracotomy Oesophageal resection and oesophagogastric anastomosis


• Through 5th intercostal space
• Dissection performed 10cm above the tumour
• This may involve transection of the azygos vein.
• The oesophagus is then removed with the stomach creating a gastric tube.
• An anastomosis is created.

The chest is closed with underwater seal drainage and tube drains to the abdominal cavity.

Post operatively

• Patients will typically recover in ITU initially.


• A nasogastric tube will have been inserted intraoperatively and must remain in place during the early
phases of recovery.
• Post operatively these patients are at relatively high risk of developing complications:

* Atelectasis- due to the effects of thoracotomy and lung collapse


* Anastomotic leakage. The risk is relatively high owing to the presence of a relatively devascularised stomach.
Often the only blood supply is from the gastroepiploic artery as all others will have been divided. If a leak does
occur then many will attempt to manage conservatively with prolonged nasogastric tube drainage and TPN. The
reality is that up to 50% of patients developing an anastomotic leak will not survive to discharge.
* Delayed gastric emptying (may be avoided by performing a pyloroplasty).

Question 24 of 438
During an Ivor Lewis Oesophagectomy for carcinoma of the lower third of the oesophagus which structure is
divided to allow mobilisation of the oesophagus?

A. Vagus nerve

B. Azygos vein

C. Right inferior lobar bronchus

D. Phrenic nerve

E. Pericardiophrenic artery

The azygos vein is routinely divided during an oesophagectomy to allow mobilisation. It arches anteriorly to
insert into the SVC on the right hand side.

Treatment of oesophageal cancer

• In general resections are not offered to those patients with distant metastasis, and usually not to those
with N2 disease.
• Local nodal involvement is not in itself a contra indication to resection.
• Surgical resection is the mainstay of treatment.
• Neoadjuvent chemotherapy is given in most cases prior to surgery.
• In situ disease may be managed by endoscopic mucosal resection, although this is still debated.
• In patients with lower third lesions an Ivor - Lewis type procedure is most commonly performed. Very
distal tumours may be suitable to a transhiatal procedure. Which is an attractive option as the penetration
of two visceral cavities required for an Ivor- Lewis type procedure increases the morbidity considerably.
• More proximal lesions will require a total oesphagectomy (Mckeown type) with anastomosis to the
cervical oesophagus.
• Patients with unresectable disease may derive benefit from local ablative procedures, palliative
chemotherapy or stent insertion.

Operative details of Ivor- Lewis procedure

• Combined laparotomy and right thoracotomy

Indication

• Lower and middle third oesophageal tumours

Preparation

• Staging with a combination of CT chest abdomen and pelvis- if no metastatic disease detected then
patients will undergo a staging laparoscopy to detect peritoneal disease.
• If both these modalities are negative then patients will finally undergo a PET CT scan to detect occult
metastatic disease. Only in those whom no evidence of advanced disease is detected will proceed to
resection.
• Patients receive a GA, double lumen endotracheal tube to allow for lung deflation, CVP and arterial
monitoring.

Procedure

• A rooftop incision is made to access the stomach and duodenum.

Laparotomy To mobilize the stomach

• The greater omentum is incised away from its attachment to the right gastroepiploic vessels along the
greater curvature of the stomach.
• Then the short gastric vessels are ligated and detached from the greater curvature from the spleen.
• The lesser omentum is incised, preserving the right gastric artery.
• The retroperitoneal attachments of the duodenum in its second and third portions are incised, allowing
the pylorus to reach the oesophageal hiatus. Some surgeons perform a pyloroplasty at this point to
facilitate gastric emptying.
• The left gastric vessels are then ligated, avoiding any injury to the common hepatic or splenic arteries.
Care must be taken to avoid inadvertently devascularising the liver owing to variations in anatomy.

Right Thoracotomy Oesophageal resection and oesophagogastric anastomosis


• Through 5th intercostal space
• Dissection performed 10cm above the tumour
• This may involve transection of the azygos vein.
• The oesophagus is then removed with the stomach creating a gastric tube.
• An anastomosis is created.

The chest is closed with underwater seal drainage and tube drains to the abdominal cavity.

Post operatively

• Patients will typically recover in ITU initially.


• A nasogastric tube will have been inserted intraoperatively and must remain in place during the early
phases of recovery.
• Post operatively these patients are at relatively high risk of developing complications:

* Atelectasis- due to the effects of thoracotomy and lung collapse


* Anastomotic leakage. The risk is relatively high owing to the presence of a relatively devascularised stomach.
Often the only blood supply is from the gastroepiploic artery as all others will have been divided. If a leak does
occur then many will attempt to manage conservatively with prolonged nasogastric tube drainage and TPN. The
reality is that up to 50% of patients developing an anastomotic leak will not survive to discharge.
* Delayed gastric emptying (may be avoided by performing a pyloroplasty).

Question 26 of 438
A 23 year old climber falls and fractures his humerus. The surgeons decide upon a posterior approach to the
middle third of the bone. Which of the following nerves is at greatest risk in this approach?

A. Ulnar

B. Antebrachial

C. Musculocutaneous

D. Radial

E. Intercostobrachial

Theme from April 2012 Exam


The radial nerve wraps around the humerus and may be injured during a posterior approach. An IM nail may be
preferred as it avoids the complex dissection needed for direct bone exposure.

Radial nerve

Continuation of posterior cord of the brachial plexus (root values C5 to T1)

Path
• In the axilla: lies posterior to the axillary artery on subscapularis, latissimus dorsi and teres major.
• Enters the arm between the brachial artery and the long head of triceps (medial to humerus).
• Spirals around the posterior surface of the humerus in the groove for the radial nerve.
• At the distal third of the lateral border of the humerus it then pierces the intermuscular septum and
descends in front of the lateral epicondyle.
• At the lateral epicondyle it lies deeply between brachialis and brachioradialis where it then divides into a
superficial and deep terminal branch.
• Deep branch crosses the supinator to become the posterior interosseous nerve.

In the image below the relationships of the radial nerve can be appreciated

Image sourced from Wikipedia

Regions innervated
Motor (main nerve) • Triceps
• Anconeus
• Brachioradialis
• Extensor carpi radialis

Motor (posterior • Extensor carpi ulnaris


interosseous branch • Extensor digitorum
• Extensor indicis
• Extensor digiti minimi
• Extensor pollicis longus and brevis
• Abductor pollicis longus

Sensory The area of skin supplying the proximal phalanges on the dorsal aspect of the hand is
supplied by the radial nerve (this does not apply to the little finger and part of the ring
finger)
Muscular innervation and effect of denervation
Anatomical Muscle affected Effect of paralysis
location
Shoulder Long head of triceps Minor effects on shoulder stability in abduction
Arm Triceps Loss of elbow extension
Forearm Supinator Weakening of supination of prone hand and elbow flexion
Brachioradialis in mid prone position
Extensor carpi radialis longus
and brevis

The cutaneous sensation of the upper limb- illustrating the contribution of the radial nerve

1/3 Question 27-29 of 438


Theme: Nerve injury

A. Median nerve
B. Ulnar nerve
C. Radial nerve
D. Posterior interosseous nerve
E. Anterior interosseous nerve
F. Musculocutaneous nerve
G. Axillary nerve
H. Brachial Trunks C5-6
I. Brachial trunks C6-7
J. Brachial Trunks C8-T1

Please select the most likely lesion site for each scenario. Each option may be used once, more than once or not
at all.

27. A 42 year old teacher is admitted with a fall. An x-ray confirms a fracture of the surgical neck of the
humerus. Which nerve is at risk?

You answered Radial nerve

The correct answer is Axillary nerve

The Axillary nerve winds around the bone at the neck of the humerus. The axillary nerve is also at
risk during shoulder dislocation.

28. A 32 year old window cleaner is admitted after falling off the roof. He reports that he had slipped off
the top of the roof and was able to cling onto the gutter for a few seconds. The patient has Horner's
syndrome.

Brachial Trunks C8-T1

The patient has a Klumpke's paralysis involving brachial trunks C8-T1. Classically there is weakness
of the hand intrinsic muscles. Involvement of T1 may cause a Horner's syndrome. It occurs as a result
of traction injuries or during delivery.

29. A 32 year old rugby player is hit hard on the shoulder during a rough tackle. Clinically his arm is
hanging loose on the side. It is pronated and medially rotated.

You answered Median nerve

The correct answer is Brachial Trunks C5-6

The patient has an Erb's palsy involving brachial trunks C5-6.

Brachial plexus

Origin Anterior rami of C5 to T1


Sections of the plexus • Roots, trunks, divisions, cords, branches
• Mnemonic:Real Teenagers Drink Cold Beer

Roots • Located in the posterior triangle


• Pass between scalenus anterior and medius

Trunks • Located posterior to middle third of clavicle


• Upper and middle trunks related superiorly to the subclavian artery
• Lower trunk passes over 1st rib posterior to the subclavian artery
Divisions Apex of axilla
Cords Related to axillary artery

Diagram illustrating the branches of the brachial plexus

Image sourced from Wikipedia

Cutaneous sensation of the upper limb

Question 30 of 438
A 68 year old man with critical limb ischaemia is undergoing a femoro-distal bypass graft. During mobilisation
of the proximal part of the posterior tibial artery which of the following is at greatest risk of injury?

A. Tibial nerve

B. Sciatic nerve

C. Saphenous nerve

D. Common peroneal nerve

E. Medial superior genicular artery

The tibial nerve is closely related to the posterior tibial artery. The tibial nerve crosses the vessel posteriorly
approximately 2.5cm distal to its origin. At its origin the nerve lies medial and then lateral after it crosses the
vessel as described.

Posterior tibial artery

• Larger terminal branch of the popliteal artery


• Terminates by dividing into the medial and lateral plantar arteries
• Accompanied by two veins throughout its length
• Position of the artery corresponds to a line drawn from the lower angle of the popliteal fossa, at the level
of the neck of the fibula, to a point midway between the medial malleolus and the most prominent part
of the heel

Relations of the posterior tibial artery


Proximal to distal
Anteriorly Tibialis posterior
Flexor digitorum longus
Posterior surface of tibia and ankle joint
Posterior Tibial nerve 2.5 cm distal to its origin
Fascia overlying the deep muscular layer
Proximal part covered by gastrocnemius and soleus
Distal part covered by skin and fascia

Question 31 of 438
A 67 year old man is undergoing an angiogram for gastro intestinal bleeding. The radiologist advances the
catheter into the coeliac axis. At what spinal level does this vessel typically arise from the aorta?

A. T10

B. L3

C. L4
D. T12

E. None of the above

The coeliac axis lies at T12, it takes an almost horizontal angle off the aorta. It has three major branches.

Abdominal aortic branches

Branches Level Paired Type


Inferior phrenic T12 (Upper border) Yes Parietal
Coeliac T12 No Visceral
Superior mesenteric L1 No Visceral
Middle suprarenal L1 Yes Visceral
Renal L1-L2 Yes Visceral
Gonadal L2 Yes Visceral
Lumbar L1-L4 Yes Parietal
Inferior mesenteric L3 No Visceral
Median sacral L4 No Parietal
Common iliac L4 Yes Terminal

Question 32 of 438
Which of the following muscles does not insert to the medial surface of the greater trochanter?

A. Gemelli

B. Obturator internus

C. Piriformis

D. Quadratus femoris

E. Obturator externus

The quadratus femoris fibres pass laterally to be inserted into the quadrate tubercle on the intertrochanteric crest
of the femur. The other muscles all insert on the trochanteric fossa lying medial to the greater trochanter.

Gluteal region

Gluteal muscles

• Gluteus maximus: inserts to gluteal tuberosity of the femur and iliotibial tract
• Gluteus medius: attach to lateral greater trochanter
• Gluteus minimis: attach to anterior greater trochanter
• All extend and abduct the hip

Deep lateral hip rotators

• Piriformis
• Gemelli
• Obturator internus
• Quadratus femoris

Nerves
Superior gluteal nerve (L5, S1) • Gluteus medius
• Gluteus minimis
• Tensor fascia lata

Inferior gluteal nerve Gluteus maximus


Damage to the superior gluteal nerve will result in the patient developing a Trendelenberg gait. Affected
patients are unable to abduct the thigh at the hip joint. During the stance phase, the weakened abductor muscles
allow the pelvis to tilt down on the opposite side. To compensate, the trunk lurches to the weakened side to
attempt to maintain a level pelvis throughout the gait cycle. The pelvis sags on the opposite side of the lesioned
superior gluteal nerve.

Question 33 of 438
During a radical gastrectomy for carcinoma of the stomach the surgeons remove the omentum. What is the main
source of its blood supply?

A. Ileocolic artery

B. Superior mesenteric artery

C. Gastroepiploic artery

D. Middle colic artery

E. Inferior mesenteric artery

The vessels supplying the omentum are the omental branches of the right and left gastro-epiploic arteries. The
colonic vessels are not responsible for the arterial supply to the omentum. The left gastro-epiploic artery is a
branch of the splenic artery and the right gastro-epiploic artery is a terminal branch of the gastroduodenal artery.

Omentum

• The omentum is divided into two parts which invest the stomach. Giving rise to the greater and lesser
omentum. The greater omentum is attached to the inferolateral border of the stomach and houses the
gastro-epiploic arteries.
• It is of variable size but is less well developed in children. This is important as the omentum confers
protection against visceral perforation (e.g. Appendicitis).
• Inferiorly between the omentum and transverse colon is one potential entry point into the lesser sac.
• Several malignant processes may involve the omentum of which ovarian cancer is the most notable.

Question 34 of 438
A 38 year old lady is due to undergo a parathyroidectomy for hyperparathyroidism. At operation the inferior
parathyroid gland is identified as being enlarged. A vessel is located adjacent to the gland laterally. This vessel
is most likely to be the:

A. External carotid artery

B. Common carotid artery

C. Internal carotid artery

D. External jugular vein

E. None of the above

The common carotid artery is a lateral relation of the inferior parathyroid.

Parathyroid glands- anatomy

• Four parathyroid glands


• Located posterior to the thyroid gland
• They lie within the pretracheal fascia

Embryology
The parathyroids develop from the extremities of the third and fourth pharyngeal pouches. The parathyroids
derived from the fourth pharyngeal pouch are located more superiorly and are associated with the thyroid gland.
Those derived from the third pharyngeal pouch lie more inferiorly and may become associated with the thymus.

Blood supply
The blood supply to the parathyroid glands is derived from the inferior and superior thyroid arteries[1]. There is
a rich anastomosis between the two vessels. Venous drainage is into the thyroid veins.

Relations
Laterally Common carotid
Medially Recurrent laryngeal nerve, trachea
Anterior Thyroid
Posterior Pretracheal fascia

Question 35 of 438
A 45 year old man has a long femoral line inserted to provide CVP measurements. The catheter passes from the
common iliac vein into the inferior vena cava. At which of the following vertebral levels will this occur?

A. L5

B. L4

C. S1

D. L3

E. L2

The common iliac veins fuse with the IVC at L5.

Inferior vena cava

Origin

• L5

Path

• Left and right common iliac veins merge to form the IVC.
• Passes right of midline
• Paired segmental lumbar veins drain into the IVC throughout its length
• The right gonadal vein empties directly into the cava and the left gonadal vein generally empties into the
left renal vein.
• The next major veins are the renal veins and the hepatic veins
• Pierces the central tendon of diaphragm at T8
• Right atrium
Image sourced from Wikipedia

Relations
Anteriorly Small bowel, first and third part of duodenum, head of pancreas, liver and bile duct, right common
iliac artery, right gonadal artery
Posteriorly Right renal artery, right psoas, right sympathetic chain, coeliac ganglion

Levels
Level Vein
T8 Hepatic vein, inferior phrenic vein, pierces diaphragm
L1 Suprarenal veins, renal vein
L2 Gonadal vein
L1-5 Lumbar veins
L5 Common iliac vein, formation of IVC

Question 36 of 438
Following a carotid endarterectomy a man notices that he has a weakness of his tongue. Damage to which of the
following nerves is the most likely explanation for this process?

A. Hypoglossal

B. Accessory

C. Ansa cervicalis

D. Vagus

E. Cervical plexus

Theme from April 2013


The hypoglossal nerve innervates the tongue and is one of the structures more commonly at risk in carotid
surgery.

Internal carotid artery

The internal carotid artery is formed from the common carotid opposite the upper border of the thyroid
cartilage. It extends superiorly to enter the skull via the carotid canal. From the carotid canal it then passes
through the cavernous sinus, above which it divides into the anterior and middle cerebral arteries.

Relations in the neck


Posterior • Longus capitis
• Pre-vertebral fascia
• Sympathetic chain
• Superior laryngeal nerve

Medially • External carotid (near origin)


• Wall of pharynx
• Ascending pharyngeal artery

Laterally • Internal jugular vein (moves posteriorly at entrance to skull)


• Vagus nerve (most posterolaterally)

Anteriorly • Sternocleidomastoid
• Lingual and facial veins
• Hypoglossal nerve

Relations in the carotid canal

• Internal carotid plexus


• Cochlea and middle ear cavity
• Trigeminal ganglion (superiorly)
• Leaves canal lies above the foramen lacerum

Path and relations in the cranial cavity


The artery bends sharply forwards in the cavernous sinus, the aducens nerve lies close to its inferolateral aspect.
The oculomotor, trochlear, opthalmic and, usually, the maxillary nerves lie in the lateral wall of the sinus. Near
the superior orbital fissure it turns posteriorly and passes postero-medially to pierce the roof of the cavernous
sinus inferior to the optic nerve. It then passes between the optic and oculomotor nerves to terminate below the
anterior perforated substance by dividing into the anterior and middle cerebral arteries.

Branches

• Anterior and middle cerebral artery


• Ophthalmic artery
• Posterior communicating artery
• Anterior choroid artery
• Meningeal arteries
• Hypophyseal arteries

Image demonstrating the internal carotid artery and its relationship to the external carotid artery
Question 37 of 438
At which of the following levels does the inferior vena cava exit the abdominal cavity?

A. T6

B. T7

C. T10

D. T8

E. T12

Theme from April 2012 Exam

Levels

Transpyloric plane
Level of the body of L1

• Pylorus stomach
• Left kidney hilum (L1- left one!)
• Right hilum of the kidney (1.5cm lower than the left)
• Fundus of the gallbladder
• Neck of pancreas
• Duodenojejunal flexure
• Superior mesenteric artery
• Portal vein
• Left and right colic flexure
• Root of the transverse mesocolon
• 2nd part of the duodenum
• Upper part of conus medullaris
• Spleen

Can be identified by asking the supine patient to sit up without using their arms. The plane is located where the
lateral border of the rectus muscle crosses the costal margin.

Anatomical planes
Subcostal plane Lowest margin of 10th costal cartilage
Intercristal plane Level of body L4 (highest point of iliac crest)
Intertubercular plane Level of body L5

Common level landmarks


Inferior mesenteric artery L3
Bifurcation of aorta into common iliac arteries L4
Formation of IVC L5 (union of common iliac veins)
Diaphragm apertures • Vena cava T8
• Oesophagus T10
• Aortic hiatus T12

Question 38 of 438
Which of the following structures lies deepest in the popliteal fossa?

A. Popliteal artery

B. Popliteal vein

C. Tibial nerve

D. Common peroneal nerve

E. Popliteal lymph nodes

Theme from January 2012 exam

From superficial to deep:


The common peroneal nerve exits the popliteal fossa along the medial border of the biceps tendon. Then the
tibial nerve lies lateral to the popliteal vessels to pass posteriorly and then medially to them. The popliteal vein
lies superficial to the popliteal artery, which is the deepest structure in the fossa.

Popliteal fossa

Boundaries of the popliteal fossa


Laterally Biceps femoris above, lateral head of gastrocnemius and plantaris below
Medially Semimembranosus and semitendinosus above, medial head of gastrocnemius below
Floor Popliteal surface of the femur, posterior ligament of knee joint and popliteus muscle
Roof Superficial and deep fascia

Image showing the popliteal fossa

Image sourced from Wikipedia

Contents

• Popliteal artery and vein


• Small saphenous vein
• Common peroneal nerve
• Tibial nerve
• Posterior cutaneous nerve of the thigh
• Genicular branch of the obturator nerve
• Lymph nodes

Question 39 of 438
Which of the following nerves is responsible for innervation of the triceps muscle?

A. Radial

B. Ulnar

C. Axillary

D. Median

E. None of the above


To remember nerve roots and their
relexes:

1-2 Ankle (S1-S2)


3-4 Knee (L3-L4)
5-6 Biceps (C5-C6)
7-8 Triceps (C7-C8)

The radial nerve innervates all three heads of triceps, with a separate branch to each head.

Triceps

Origin • Long head- infraglenoid tubercle of the scapula.


• Lateral head- dorsal surface of the humerus, lateral and proximal to the groove of the
radial nerve
• Medial head- posterior surface of the humerus on the inferomedial side of the radial
groove and both of the intermuscular septae

Insertion • Olecranon process of the ulna. Here the olecranon bursa is between the triceps tendon and
olecranon.
• Some fibres insert to the deep fascia of the forearm, posterior capsule of the elbow
(preventing the capsule from being trapped between olecranon and olecranon fossa during
extension)

Innervation Radial nerve


Blood Profunda brachii artery
supply
Action Elbow extension. The long head can adduct the humerus and and extend it from a flexed position
Relations The radial nerve and profunda brachii vessels lie between the lateral and medial heads

Question 40 of 438
An intravenous drug user develops a false aneurysm and requires emergency surgery. The procedure is difficult
and the femoral nerve is inadvertently transected. Which of the following muscles is least likely to be affected
as a result?

A. Sartorius

B. Vastus medialis

C. Pectineus

D. Quadriceps femoris

E. Adductor magnus
Mnemonic for femoral nerve supply

(don't) M I S V Q Scan for PE


M edial cutaneous nerve of the thigh
I ntermediate cutaneous nerve of the
thigh
S aphenous nerve
V astus
Q uadriceps femoris
S artorius

PE ectineus

Adductor magnus is innervated by the obturator and sciatic nerve. The pectineus muscle is sometimes supplied
by the obturator nerve but this is variable. Since the question states least likely, the correct answer is adductor
magnus

Femoral nerve

Root values L2, 3, 4


Innervates • Pectineus
• Sartorius
• Quadriceps femoris
• Vastus lateralis/medialis/intermedius

Branches • Medial cutaneous nerve of thigh


• Saphenous nerve
• Intermediate cutaneous nerve of thigh

Path
Penetrates psoas major and exits the pelvis by passing under the inguinal ligament to enter the femoral triangle,
lateral to the femoral artery and vein.
Image sourced from Wikipedia

Mnemonic for femoral nerve supply

(don't) M I S V Q Scan for PE


M edial cutaneous nerve of the thigh
I ntermediate cutaneous nerve of the thigh
S aphenous nerve

V astus
Q uadriceps femoris
S artorius

PE ectineus

Question 41 of 438
What is the nerve root value of the external urethral sphincter?

A. S4

B. S1, S2, S3

C. S2, S3, S4
D. L3, L4, L5

E. L5, S1, S2

Theme from April 2012 Exam


The external urethral sphincter is innervated by branches of the pudendal nerve, therefore the root values are S2,
S3, S4.

Urethral anatomy

Female urethra
The female urethra is shorter and more acutely angulated than the male urethra. It is an extra-peritoneal
structure and embedded in the endopelvic fascia. The neck of the bladder is subjected to transmitted intra-
abdominal pressure and therefore deficiency in this area may result in stress urinary incontinence. Between the
layers of the urogenital diaphragm the female urethra is surrounded by the external urethral sphincter, this is
innervated by the pudendal nerve. It ultimately lies anterior to the vaginal orifice.

Male urethra
In males the urethra is much longer and is divided into four parts.

Pre-prostatic urethra Extremely short and lies between the bladder and prostate gland.It has a stellate
lumen and is between 1 and 1.5cm long.Innervated by sympathetic
noradrenergic fibres, as this region is composed of striated muscles bundles
they may contract and prevent retrograde ejaculation.

Prostatic urethra This segment is wider than the membranous urethra and contains several
openings for the transmission of semen (at the midpoint of the urethral crest).

Membranous urethra Narrowest part of the urethra and surrounded by external sphincter. It traverses
the perineal membrane 2.5cm postero-inferior to the symphysis pubis.

Penile urethra Travels through the corpus songiosum on the underside of the penis. It is the
longest urethral segment.It is dilated at its origin as the infrabulbar fossa and
again in the gland penis as the navicular fossa. The bulbo-urethral glands open
into the spongiose section of the urethra 2.5cm below the perineal membrane.

The urothelium is transitional in nature near to the bladder and becomes squamous more distally.

Question 42 of 438
A 45 year old man is stabbed in the abdomen and the inferior vena cava is injured. How many functional valves
does this vessel usually have?
A. 0

B. 1

C. 3

D. 2

E. 4
Mnemonic for the Inferior vena cava tributaries: I
Like To Rise So High:

Iliacs
Lumbar
Testicular
Renal
Suprarenal
Hepatic vein

The lack of valves in the IVC is important clinically when it is cannulated during cardiopulmonary bypass,
using separate SVC and IVC catheters, such as when the right atrium is to be opened. Note that there is a non
functional valve between the right atrium and inferior vena cava.

Inferior vena cava

Origin

• L5

Path

• Left and right common iliac veins merge to form the IVC.
• Passes right of midline
• Paired segmental lumbar veins drain into the IVC throughout its length
• The right gonadal vein empties directly into the cava and the left gonadal vein generally empties into the
left renal vein.
• The next major veins are the renal veins and the hepatic veins
• Pierces the central tendon of diaphragm at T8
• Right atrium
Image sourced from Wikipedia

Relations
Anteriorly Small bowel, first and third part of duodenum, head of pancreas, liver and bile duct, right common
iliac artery, right gonadal artery
Posteriorly Right renal artery, right psoas, right sympathetic chain, coeliac ganglion

Levels
Level Vein
T8 Hepatic vein, inferior phrenic vein, pierces diaphragm
L1 Suprarenal veins, renal vein
L2 Gonadal vein
L1-5 Lumbar veins
L5 Common iliac vein, formation of IVC

Question 43 of 438
Which of the following structures does not pass posteriorly to the medial malleolus?

A. Posterior tibial artery

B. Tibial nerve

C. Tibialis anterior tendon

D. Tendon of flexor digitorum longus

E. Tendon of flexor hallucis longus


Mnemonic for structures posterior to the
medial malleolus:
Tom Dick And Nervous Harry

T ibialis posterior tendon


flexor Digitorum longus
A rtery
N erve
H allucis longus

Medial malleolus

The following structures pass posterior to medial malleolus (in order):

• Tibialis posterior tendon


• Flexor digitorum longus tendon
• Posterior tibial artery
• Tibial nerve
• Tendon of flexor hallucis longus

Question 44 of 438
Which of the following statements relating to the root of the neck is false?

A. The lung projects into the neck beyond the first rib and is constrained
by Sibson's fascia

B. The subclavian artery arches over the first rib anterior to scalenus
anterior

C. The roots and trunks of the Brachial plexus lie posterior to the
subclavian artery on the first rib

D. The roots and trunks of the Brachial plexus lie between scalenus
anterior and scalenus medius muscles

E. The thyrocervical trunk is a branch of the subclavian artery

The subclavian artery lies posterior to scalenus anterior, the vein lies in front. Sibson's fascia is another name
for the suprapleural membrane.

Root of the neck

Thoracic Outlet

• Where the subclavian artery and vein and the brachial plexus exit the thorax and enter the arm.
• They pass over the 1st rib and under the clavicle.
• The subclavian vein is the most anterior structure and is immediately anterior to scalenus anterior and its
attachment to the first rib.
• Scalenus anterior has 2 parts, the subclavian artery leaves the thorax by passing over the first rib and
between these 2 portions of the muscle.
• At the level of the first rib, the lower cervical nerve roots combine to form the 3 trunks of the brachial
plexus. The lowest trunk is formed by the union of C8 and T1, and this trunk lies directly posterior to the
artery and is in contact with the superior surface of the first rib.

Thoracic outlet obstruction causes neurovascular compromise.

Question 45 of 438
A patient presents to the clinic following a surgical procedure. She complains that she is unable to shrug her
shoulder. What is the most likely underlying nerve injury?

A. Accessory nerve

B. Cervical plexus

C. Ansa cervicalis

D. Long thoracic nerve

E. Axillary nerve

Theme from April 2013 Exam


The accessory nerve may be injured in operations in the posterior triangle. Injury will affect trapezius.

Cranial nerves

Cranial nerve lesions


Olfactory nerve May be injured in basal skull fractures or involved in frontal lobe tumour extension. Loss
of olfactory nerve function in relation to major CNS pathology is seldom an isolated
event and thus it is poor localiser of CNS pathology.
Optic nerve Problems with visual acuity may result from intra ocular disorders. Problems with the
blood supply such as amaurosis fugax may produce temporary visual distortion. More
important surgically is the pupillary response to light. The pupillary size may be altered
in a number of disorders. Nerves involved in the resizing of the pupil connect to the
pretectal nucleus of the high midbrain, bypassing the lateral geniculate nucleus and the
primary visual cortex. From the pretectal nucleus neurones pass to the Edinger -
Westphal nucleus, motor axons from here pass along with the oculomotor nerve. They
synapse with ciliary ganglion neurones; the parasympathetic axons from this then
innervate the iris and produce miosis. The miotic pupil is seen in disorders such a
Horner's syndrome or opiate overdose.
Mydriasis is the dilatation of the pupil in response to disease, trauma, drugs (or the
dark!). It is pathological when light fails to induce miosis. The radial muscle is innervated
by the sympathetic nervous system. Because the parasympathetic fibres travel with the
oculomotor nerve they will be damaged by lesions affecting this nerve (e.g. cranial
trauma).
The response to light shone in one eye is usually a constriction of both pupils. This
indicates intact direct and consensual light reflexes. When the optic nerve has an afferent
defect the light shining on the affected eye will produce a diminished pupillary response
in both eyes. Whereas light shone on the unaffected eye will produce a normal pupillary
response in both eyes. This is referred to as the Marcus Gunn pupil and is seen in
conditions such as optic neuritis. In a total CN II lesion shining the light in the affected
eye will produce no response.
Oculomotor nerve The pupillary effects are described above. In addition it supplies all ocular muscles apart
from lateral rectus and superior oblique. Thus the affected eye will be deviated
inferolaterally. Levator palpebrae superioris may also be impaired resulting in impaired
ability to close the eye.
Trochlear nerve The eye will not be able to look down.
Trigeminal nerve Largest cranial nerve. Exits the brainstem at the pons. Branches are ophthalmic, maxillary
and mandibular. Only the mandibular branch has both sensory and motor fibres. Branches
converge to form the trigeminal ganglion (located in Meckels cave). It supplies the
muscles of mastication and also tensor veli palatine, mylohyoid, anterior belly of
digastric and tensor tympani. The detailed descriptions of the various sensory functions
are described in other areas of the website. The corneal reflex is important and is elicited
by applying a small tip of cotton wool to the cornea, a reflex blink should occur if it is
intact. It is mediated by: the naso ciliary branch of the ophthalmic branch of the
trigeminal (sensory component) and the facial nerve producing the motor response.
Lesions of the afferent arc will produce bilateral absent blink and lesions of the efferent
arc will result in a unilateral absent blink.
Abducens nerve The affected eye will have a deficit of abduction. This cranial nerve exits the brainstem
between the pons and medulla. It thus has a relatively long intra cranial course which
renders it susceptible to damage in raised intra cranial pressure.
Facial nerve Emerges from brainstem between pons and medulla. It controls muscles of facial
expression and taste from the anterior 2/3 of the tongue. The nerve passes into the petrous
temporal bone and into the internal auditory meatus. It then passes through the facial
canal and exits at the stylomastoid foramen. It passes through the parotid gland and
divides at this point. It does not innervate the parotid gland. Its divisions are considered in
other parts of the website. Its motor fibres innervate orbicularis oculi to produce the
efferent arm of the corneal reflex. In surgical practice it may be injured during parotid
gland surgery or invaded by malignancies of the gland and a lower motor neurone on the
ipsilateral side will result.
Vestibulo-cochlear Exits from the pons and then passes through the internal auditory meatus. It is implicated
nerve in sensorineural hearing loss. Individuals with sensorineural hearing loss will localise the
sound in webers test to the normal ear. Rinnes test will be reduced on the affected side
but should still work. These two tests will distinguish sensorineural hearing loss from
conductive deafness. In the latter condition webers test will localise to the affected ear
and Rinnes test will be impaired on the affected side. Surgical lesions affecting this nerve
include CNS tumours and basal skull fractures. It may also be damaged by the
administration of ototoxic drugs (of which gentamicin is the most commonly used in
surgical practice).
Glossopharyngeal Exits the pons just above the vagus. Receives sensory fibres from posterior 1/3 tongue,
nerve tonsils, pharynx and middle ear (otalgia may occur following tonsillectomy). It receives
visceral afferents from the carotid bodies. It supplies parasympathetic fibres to the parotid
gland via the otic ganglion and motor function to stylopharyngeaus muscle. The sensory
function of the nerve is tested using the gag reflex.
Vagus nerve Leaves the medulla between the olivary nucleus and the inferior cerebellar peduncle.
Passes through the jugular foramen and into the carotid sheath. Details of the functions of
the vagus nerve are covered in the website under relevant organ sub headings.
Accessory nerve Exists from the caudal aspect of the brainstem (multiple branches) supplies trapezius and
sternocleidomastoid muscles. The distal portion of this nerve is most prone to injury
during surgical procedures.
Hypoglossal nerve Emerges from the medulla at the preolivary sulcus, passes through the hypoglossal canal.
It lies on the carotid sheath and passes deep to the posterior belly of digastric to supply
muscles of the tongue (except palatoglossus). Its location of the carotid sheath makes it
vulnerable during carotid endarterectomy surgery and damage will produce ipsilateral
defect in muscle function.

Question 46 of 438
Which of the following are not generally supplied by the right coronary artery?

A. The sino atrial node

B. The circumflex artery

C. The atrioventricular node

D. Most of the right ventricle

E. The right atrium

The circumflex artery is generally a branch of the left coronary artery.

Heart anatomy

The walls of each cardiac chamber comprise:

• Epicardium
• Myocardium
• Endocardium

Cardiac muscle is attached to the cardiac fibrous skeleton.

Relations
The heart and roots of the great vessels within the pericardial sac are related anteriorly to the sternum, medial
ends of the 3rd to 5th ribs on the left and their associated costal cartilages. The heart and pericardial sac are
situated obliquely two thirds to the left and one third to the right of the median plane.

The pulmonary valve lies at the level of the left third costal cartilage.
The mitral valve lies at the level of the fourth costal cartilage.

Coronary sinus
This lies in the posterior part of the coronary groove and receives blood from the cardiac veins. The great
cardiac vein lies at its left and the middle and small cardiac veins lie on its right. The smallest cardiac veins
drain into the atria directly.

Aortic sinus
Right coronary artery arises from the right aortic sinus, the left is derived from the left aortic sinus and no vessel
emerges from the posterior sinus.

Right and left ventricles

Structure Left Ventricle


A-V Valve Mitral (double leaflet)
Walls Twice as thick as right
Trabeculae carnae Much thicker and more numerous

Right coronary artery


The RCA supplies:

• Right atrium
• Diaphragmatic part of the left ventricle
• Usually the posterior third of the interventricular septum
• The sino atrial node (60% cases)
• The atrio ventricular node (80% cases)

Left coronary artery


The LCA supplies:

• Left atrium
• Most of left ventricle
• Part of the right ventricle
• Anterior two thirds of the inter ventricular septum
• The sino atrial node (remaining 40% cases)

Innervation of the heart


Autonomic nerve fibres from the superficial and deep cardiac plexus. These lie anterior to the bifurcation of the
trachea, posterior to the ascending aorta and superior to the bifurcation of the pulmonary trunk. The
parasympathetic supply to the heart is from presynaptic fibres of the vagus nerves.

Valves of the heart


Mitral valve Aortic valve Pulmonary valve Tricuspid valve
2 cusps 3 cusps 3 cusps 3 cusps
First heart sound Second heart sound Second heart sound First heart sound
1 anterior cusp 1 anterior cusp 2 anterior cusps 2 anterior cusps
Attached to chordae tendinae No chordae No chordae Attached to chordae tendinae

Question 47 of 438
A 44 year old man has a malignant melanoma and is undergoing a block dissection of the groin. The femoral
triangle is being explored for intra operative bleeding. Which of the following forms the medial border of the
femoral triangle?

A. Femoral artery

B. Biceps femoris

C. Adductor longus

D. Sartorius

E. Adductor magnus
Vastus medialis forms the lateral border of the adductor canal. The sartorius muscles forms the roof of the
adductor canal.

Adductor longus forms the medial boundary of the femoral triangle (see below).

Femoral triangle anatomy

Boundaries
Superiorly Inguinal ligament
Laterally Sartorius
Medially Adductor longus
Floor Iliopsoas, adductor longus and pectineus
Roof • Fascia lata and Superficial fascia
• Superficial inguinal lymph nodes (palpable below the inguinal ligament)
• Great saphenous vein
Image sourced from Wikipedia

Contents

• Femoral vein (medial to lateral)


• Femoral artery-pulse palpated at the mid inguinal point
• Femoral nerve
• Deep and superficial inguinal lymph nodes
• Lateral cutaneous nerve
• Great saphenous vein
• Femoral branch of the genitofemoral nerve

Question 48 of 438
The foramen marking the termination of the adductor canal is located in which of the following?

A. Adductor longus

B. Adductor magnus

C. Adductor brevis

D. Sartorius

E. Semimembranosus
The foramen marking the distal limit of the adductor canal is contained within adductor magnus. The vessel
passes through this region to enter the popliteal fossa.

Adductor canal

• Also called Hunter's or subsartorial canal

• Immediately distal to the apex of the femoral triangle, lying in the middle third of the thigh. Canal
terminates at the adductor hiatus.

Borders Contents
Laterally Vastus medialis muscle Saphenous nerve
Posteriorly Adductor longus, adductor magnus Superficial femoral artery
Roof Sartorius Superficial femoral vein

In the image below the sartorius muscle is removed to expose the canal contents
Question 49 of 438
Which of the following is the first vessel to branch from the external carotid artery?

A. Superior thyroid artery

B. Inferior thyroid artery

C. Lingual artery

D. Facial artery

E. Occipital artery

The first branch of the external carotid artery is the superior thyroid artery. The inferior thyroid artery is derived
from the thyrocervical trunk. The other branches are illustrated below.

External carotid artery

The external carotid commences immediately lateral to the pharyngeal side wall. It ascends and lies anterior to
the internal carotid and posterior to the posterior belly of digastric and stylohyoid. More inferiorly it is covered
by sternocleidomastoid, passed by hypoglossal nerves, lingual and facial veins.
It then pierces the fascia of the parotid gland finally dividing into its terminal branches within the gland itself.

Surface marking of the carotid


This is an imaginary line drawn from the bifurcation of the common carotid passing behind the angle of the jaw
to a point immediately anterior to the tragus of the ear.

Branches of the external carotid artery


It has six branches, three in front, two behind and one deep.
Three in front Superior thyroid
Lingual
Facial
Two behind Occipital
Posterior auricular
Deep Ascending pharyngeal

It terminates by dividing into the superficial temporal and maxillary arteries in the parotid gland.
Question 50 of 438
A motorcyclist is injured in a road traffic accident and is not wearing a helmet. He suffers a severe closed head
injury and develops raised intracranial pressure. The first cranial nerve to be affected by this process is likely to
be:

A. Oculomotor

B. Hypoglossal

C. Motor branch of the trigeminal

D. Abducens

E. Trochlear

The abducens nerve (CN VI) has the longest intra cranial course and is thus the most susceptible to raised intra
cranial pressure. It also passes over the petrous temporal bone and 6th nerve palsies are also seen in mastoiditis.

Cranial nerves

Cranial nerve lesions


Olfactory nerve May be injured in basal skull fractures or involved in frontal lobe tumour extension. Loss
of olfactory nerve function in relation to major CNS pathology is seldom an isolated
event and thus it is poor localiser of CNS pathology.
Optic nerve Problems with visual acuity may result from intra ocular disorders. Problems with the
blood supply such as amaurosis fugax may produce temporary visual distortion. More
important surgically is the pupillary response to light. The pupillary size may be altered
in a number of disorders. Nerves involved in the resizing of the pupil connect to the
pretectal nucleus of the high midbrain, bypassing the lateral geniculate nucleus and the
primary visual cortex. From the pretectal nucleus neurones pass to the Edinger -
Westphal nucleus, motor axons from here pass along with the oculomotor nerve. They
synapse with ciliary ganglion neurones; the parasympathetic axons from this then
innervate the iris and produce miosis. The miotic pupil is seen in disorders such a
Horner's syndrome or opiate overdose.
Mydriasis is the dilatation of the pupil in response to disease, trauma, drugs (or the
dark!). It is pathological when light fails to induce miosis. The radial muscle is innervated
by the sympathetic nervous system. Because the parasympathetic fibres travel with the
oculomotor nerve they will be damaged by lesions affecting this nerve (e.g. cranial
trauma).
The response to light shone in one eye is usually a constriction of both pupils. This
indicates intact direct and consensual light reflexes. When the optic nerve has an afferent
defect the light shining on the affected eye will produce a diminished pupillary response
in both eyes. Whereas light shone on the unaffected eye will produce a normal pupillary
response in both eyes. This is referred to as the Marcus Gunn pupil and is seen in
conditions such as optic neuritis. In a total CN II lesion shining the light in the affected
eye will produce no response.
Oculomotor nerve The pupillary effects are described above. In addition it supplies all ocular muscles apart
from lateral rectus and superior oblique. Thus the affected eye will be deviated
inferolaterally. Levator palpebrae superioris may also be impaired resulting in impaired
ability to close the eye.
Trochlear nerve The eye will not be able to look down.
Trigeminal nerve Largest cranial nerve. Exits the brainstem at the pons. Branches are ophthalmic, maxillary
and mandibular. Only the mandibular branch has both sensory and motor fibres. Branches
converge to form the trigeminal ganglion (located in Meckels cave). It supplies the
muscles of mastication and also tensor veli palatine, mylohyoid, anterior belly of
digastric and tensor tympani. The detailed descriptions of the various sensory functions
are described in other areas of the website. The corneal reflex is important and is elicited
by applying a small tip of cotton wool to the cornea, a reflex blink should occur if it is
intact. It is mediated by: the naso ciliary branch of the ophthalmic branch of the
trigeminal (sensory component) and the facial nerve producing the motor response.
Lesions of the afferent arc will produce bilateral absent blink and lesions of the efferent
arc will result in a unilateral absent blink.
Abducens nerve The affected eye will have a deficit of abduction. This cranial nerve exits the brainstem
between the pons and medulla. It thus has a relatively long intra cranial course which
renders it susceptible to damage in raised intra cranial pressure.
Facial nerve Emerges from brainstem between pons and medulla. It controls muscles of facial
expression and taste from the anterior 2/3 of the tongue. The nerve passes into the petrous
temporal bone and into the internal auditory meatus. It then passes through the facial
canal and exits at the stylomastoid foramen. It passes through the parotid gland and
divides at this point. It does not innervate the parotid gland. Its divisions are considered in
other parts of the website. Its motor fibres innervate orbicularis oculi to produce the
efferent arm of the corneal reflex. In surgical practice it may be injured during parotid
gland surgery or invaded by malignancies of the gland and a lower motor neurone on the
ipsilateral side will result.
Vestibulo-cochlear Exits from the pons and then passes through the internal auditory meatus. It is implicated
nerve in sensorineural hearing loss. Individuals with sensorineural hearing loss will localise the
sound in webers test to the normal ear. Rinnes test will be reduced on the affected side
but should still work. These two tests will distinguish sensorineural hearing loss from
conductive deafness. In the latter condition webers test will localise to the affected ear
and Rinnes test will be impaired on the affected side. Surgical lesions affecting this nerve
include CNS tumours and basal skull fractures. It may also be damaged by the
administration of ototoxic drugs (of which gentamicin is the most commonly used in
surgical practice).
Glossopharyngeal Exits the pons just above the vagus. Receives sensory fibres from posterior 1/3 tongue,
nerve tonsils, pharynx and middle ear (otalgia may occur following tonsillectomy). It receives
visceral afferents from the carotid bodies. It supplies parasympathetic fibres to the parotid
gland via the otic ganglion and motor function to stylopharyngeaus muscle. The sensory
function of the nerve is tested using the gag reflex.
Vagus nerve Leaves the medulla between the olivary nucleus and the inferior cerebellar peduncle.
Passes through the jugular foramen and into the carotid sheath. Details of the functions of
the vagus nerve are covered in the website under relevant organ sub headings.
Accessory nerve Exists from the caudal aspect of the brainstem (multiple branches) supplies trapezius and
sternocleidomastoid muscles. The distal portion of this nerve is most prone to injury
during surgical procedures.
Hypoglossal nerve Emerges from the medulla at the preolivary sulcus, passes through the hypoglossal canal.
It lies on the carotid sheath and passes deep to the posterior belly of digastric to supply
muscles of the tongue (except palatoglossus). Its location of the carotid sheath makes it
vulnerable during carotid endarterectomy surgery and damage will produce ipsilateral
defect in muscle function.

Question 51 of 438
A 32 year old man is undergoing a splenectomy. Division of which of the following will be necessary during
the procedure?

A. Left crus of diaphragm

B. Short gastric vessels

C. Gerotas fascia

D. Splenic flexure of colon

E. Marginal artery

Theme from 2011 Exam


During a splenectomy the short gastric vessels which lie within the gastrosplenic ligament will need to be
divided. The splenic flexure of the colon may need to be mobilised. However, it will almost never need to be
divided, as this is watershed area that would necessitate a formal colonic resection in the event of division.

Splenic anatomy

The spleen is the largest lymphoid organ in the body. It is an intraperitoneal organ, the peritoneal attachments
condense at the hilum where the vessels enter the spleen. Its blood supply is from the splenic artery (derived
from the coeliac axis) and the splenic vein (which is joined by the IMV and unites with the SMV).

• Embryology: derived from mesenchymal tissue


• Shape: clenched fist
• Position: below 9th-12th ribs
• Weight: 75-150g

Relations

• Superiorly- diaphragm
• Anteriorly- gastric impression
• Posteriorly- kidney
• Inferiorly- colon
• Hilum: tail of pancreas and splenic vessels
• Forms apex of lesser sac (containing short gastric vessels)

Question 52 of 438
A 24 year old motor cyclist is involved in a road traffic accident. He suffers a tibial fracture which is treated
with an intra medullary nail. Post operatively he develops a compartment syndrome. Surgical decompression of
the anterior compartment will relieve pressure on all of the following muscles except?

A. Peroneus brevis

B. Peroneus tertius

C. Extensor digitorum longus

D. Tibialis anterior

E. None of the above

The anterior compartment contains:


Tibialis anterior
Extensor digitorum longus
Peroneus tertius
Extensor hallucis longus
Anterior tibial artery
All the muscles are innervated by the deep peroneal nerve.

Lower limb- Muscular compartments

Anterior compartment
Muscle Nerve Action
Tibialis anterior Deep peroneal nerve Dorsiflexes ankle joint, inverts foot
Extensor digitorum longus Deep peroneal nerve Extends lateral four toes, dorsiflexes ankle joint
Peroneus tertius Deep peroneal nerve Dorsiflexes ankle, everts foot
Extensor hallucis longus Deep peroneal nerve Dorsiflexes ankle joint, extends big toe

Peroneal compartment
Muscle Nerve Action
Peroneus longus Superficial peroneal nerve Everts foot, assists in plantar flexion
Peroneus brevis Superficial peroneal nerve Plantar flexes the ankle joint

Superficial posterior compartment


muscle Nerve Action
Gastrocnemius Tibial nerve Plantar flexes the foot, may also flex the knee
Soleus Tibial nerve Plantar flexor

Deep posterior compartment


Muscle Nerve Action
Flexor digitorum longus Tibial Flexes the lateral four toes
Flexor hallucis longus Tibial Flexes the great toe
Tibialis posterior Tibial Plantar flexor, inverts the foot

Question 53 of 438
A 43 year old lady underwent and attempted placement of a central line into the internal jugular vein.
Unfortunately, the doctor damaged the carotid artery and this necessitated surgical exploration. As the surgeons
incise the carotid sheath a nerve is identified lying between the internal jugular vein and the carotid artery.
Which of the following is this nerve most likely to be?

A. Glossopharyngeal nerve

B. Hypoglossal nerve

C. Superior laryngeal nerve

D. Recurrent laryngeal nerve

E. Vagus

The vagus lies in the carotid sheath. The hypoglossal nerve crosses the sheath, but does not lie within it.

Common carotid artery

The right common carotid artery arises at the bifurcation of the brachiocephalic trunk, the left common carotid
arises from the arch of the aorta. Both terminate at the level of the upper border of the thyroid cartilage (the
lower border of the third cervical vertebra) by dividing into the internal and external carotid arteries.

Left common carotid artery


This vessel arises immediately to the left and slightly behind the origin of the brachiocephalic trunk. Its thoracic
portion is 2.5- 3.5 cm in length and runs superolaterally to the sternoclavicular joint.

In the thorax
The vessel is in contact, from below upwards, with the trachea, left recurrent laryngeal nerve, left margin of the
oesophagus. Anteriorly the left brachiocephalic vein runs across the artery, and the cardiac branches from the
left vagus descend in front of it. These structures together with the thymus and the anterior margins of the left
lung and pleura separate the artery from the manubrium.
In the neck
The artery runs superiorly deep to sternocleidomastoid and then enters the anterior triangle. At this point it lies
within the carotid sheath with the vagus nerve and the internal jugular vein. Posteriorly the sympathetic trunk
lies between the vessel and the prevertebral fascia. At the level of C7 the vertebral artery and thoracic duct lie
behind it. The anterior tubercle of C6 transverse process is prominent and the artery can be compressed against
this structure (it corresponds to the level of the cricoid).
Anteriorly at C6 the omohyoid muscle passes superficial to the artery.
Within the carotid sheath the jugular vein lies lateral to the artery.

Right common carotid artery


The right common carotid arises from the brachiocephalic artery. The right common carotid artery corresponds
with the cervical portion of the left common carotid, except that there is no thoracic duct on the right. The
oesophagus is less closely related to the right carotid than the left.

Summary points about the carotid anatomy

Path
Passes behind the sternoclavicular joint (12% patients above this level) to the upper border of the thyroid
cartilage, to divide into the external (ECA) and internal carotid arteries (ICA).

Relations

• Level of 6th cervical vertebra crossed by omohyoid


• Then passes deep to the thyrohyoid, sternohyoid, sternomastoid muscles.
• Passes anterior to the carotid tubercle (transverse process 6th cervical vertebra)-NB compression here
stops haemorrhage.
• The inferior thyroid artery passes posterior to the common carotid artery.
• Then : Left common carotid artery crossed by thoracic duct, Right common carotid artery crossed by
recurrent laryngeal nerve

Question 54 of 438
A patient has a chest drain insertion. There is fresh blood at the chest drain insertion area. Which vessel has
been damaged?

A. Pericardiophrenic artery

B. Intercostal vein

C. Right ventricle

D. Vagus artery

E. Intercostal artery

Theme from 2009 Exam

Within the intercostal spaces there are thin, strong muscles, intercostal vessels, nerves and lymphatics. There
are 3 intercostal muscle layers corresponding to the lateral abdominal wall; external, internal, innermost
intercostals. At the mid axillary line there are thin intracostals which is an extension of the internal intercostal
muscle. In each intercostal space lies the neurovascular bundle, comprising, from superior to inferiorly; the
posterior intercostal vein, artery and nerve, lying protected in the subcostal groove of the rib above and situated
between the second and third layer of the intercostal muscles. These blood vessels anastomose anteriorly with
the anterior intercostal vessels, which arise from the internal thoracic artery and vein.

Chest drains

There are a number of different indications for chest drain insertion. In general terms large bore chest drains are
preferred for trauma and haemothorax drainage. Smaller diameter chest drains can be used for pneumothorax or
pleural effusion drainage.

Insertion can be performed either using anatomical guidance or through ultrasound guidance. In the exam, the
anatomical method is usually tested.

It is advised that chest drains are placed in the 'safe triangle'. The triangle is located in the mid axillary line of
the 5th intercostal space. It is bordered by:
Anterior edge latissimus dorsi, the lateral border of pectoralis major, a line superior to the horizontal level of the
nipple, and the apex below the axilla.

Another triangle is situated behind the scapula. It is bounded above by the trapezius, below by the latissimus
dorsi, and laterally by the vertebral border of the scapula; the floor is partly formed by the rhomboid major. If
the scapula is drawn forward by folding the arms across the chest, and the trunk bent forward, parts of the sixth
and seventh ribs and the interspace between them become subcutaneous and available for auscultation. The
space is therefore known as the triangle of auscultation.

Question 55 of 438
Two teenagers are playing with an airgun when one accidentally shoots his friend in the abdomen. He is
brought to the emergency department. On examination there is a bullet entry point immediately to the right of
the rectus sheath at the level of the 1st lumbar vertebra. Which of the following structures is most likely to be
injured by the bullet?

A. Head of pancreas

B. Right ureter

C. Right adrenal gland

D. Fundus of the gallbladder

E. Gastric antrum

Theme from September 2011 Exam

The fundus of the gallbladder lies at this level and is the most superficially located structure.

Levels

Transpyloric plane
Level of the body of L1

• Pylorus stomach
• Left kidney hilum (L1- left one!)
• Right hilum of the kidney (1.5cm lower than the left)
• Fundus of the gallbladder
• Neck of pancreas
• Duodenojejunal flexure
• Superior mesenteric artery
• Portal vein
• Left and right colic flexure
• Root of the transverse mesocolon
• 2nd part of the duodenum
• Upper part of conus medullaris
• Spleen

Can be identified by asking the supine patient to sit up without using their arms. The plane is located where the
lateral border of the rectus muscle crosses the costal margin.

Anatomical planes
Subcostal plane Lowest margin of 10th costal cartilage
Intercristal plane Level of body L4 (highest point of iliac crest)
Intertubercular plane Level of body L5

Common level landmarks


Inferior mesenteric artery L3
Bifurcation of aorta into common iliac arteries L4
Formation of IVC L5 (union of common iliac veins)
Diaphragm apertures • Vena cava T8
• Oesophagus T10
• Aortic hiatus T12

Question 56 of 438
Which of the following muscles inserts onto the lesser tuberostiy of the the humerus?

A. Subscapularis

B. Deltoid

C. Supraspinatus

D. Teres minor

E. Infraspinatus

With the exception of subscapularis which inserts into the lesser tuberosity, the muscles of the rotator cuff insert
into the greater tuberosity.

Shoulder joint

• Shallow synovial ball and socket type of joint.


• It is an inherently unstable joint, but is capable to a wide range of movement.
• Stability is provided by muscles of the rotator cuff that pass from the scapula to insert in the greater
tuberosity (all except sub scapularis-lesser tuberosity).

Glenoid labrum

• Fibrocartilaginous rim attached to the free edge of the glenoid cavity


• Tendon of the long head of biceps arises from within the joint from the supraglenoid tubercle, and is
fused at this point to the labrum.
• The long head of triceps attaches to the infraglenoid tubercle

Fibrous capsule

• Attaches to the scapula external to the glenoid labrum and to the labrum itself (postero-superiorly)
• Attaches to the humerus at the level of the anatomical neck superiorly and the surgical neck inferiorly
• Anteriorly the capsule is in contact with the tendon of subscapularis, superiorly with the supraspinatus
tendon, and posteriorly with the tendons of infraspinatus and teres minor. All these blend with the
capsule towards their insertion.
• Two defects in the fibrous capsule; superiorly for the tendon of biceps. Anteriorly there is a defect
beneath the subscapularis tendon.
• The inferior extension of the capsule is closely related to the axillary nerve at the surgical neck and this
nerve is at risk in anteroinferior dislocations. It also means that proximally sited osteomyelitis may
progress to septic arthritis.

Movements and muscles


Flexion Anterior part of deltoid
Pectoralis major
Biceps
Coracobrachialis
Extension Posterior deltoid
Teres major
Latissimus dorsi
Adduction Pectoralis major
Latissimus dorsi
Teres major
Coracobrachialis
Abduction Mid deltoid
Supraspinatus
Medial rotation Subscapularis
Anterior deltoid
Teres major
Latissimus dorsi
Lateral rotation Posterior deltoid
Infraspinatus
Teres minor

Important anatomical relations


Anteriorly Brachial plexus
Axillary artery and vein
Posterior Suprascapular nerve
Suprascapular vessels
Inferior Axillary nerve
Circumflex humeral vessels

Question 57 of 438
Which of the following nerves is not contained within the posterior triangle of the neck?

A. Accessory nerve

B. Phrenic nerve

C. Greater auricular nerve

D. Ansa cervicalis

E. Lesser occiptal nerve

Theme from September 2012 Exam


Ansa cervicalis is a content of the anterior triangle of the neck.

Posterior triangle of the neck

Boundaries
Apex Sternocleidomastoid and the Trapezius muscles at the Occipital bone
Anterior Posterior border of the Sternocleidomastoid
Posterior Anterior border of the Trapezius
Base Middle third of the clavicle
Image sourced from Wikipedia

Contents
Nerves • Accessory nerve
• Phrenic nerve
• Three trunks of the brachial plexus
• Branches of the cervical plexus: Supraclavicular nerve, transverse cervical nerve, great
auricular nerve, lesser occipital nerve

Vessels • External jugular vein


• Subclavian artery

Muscles • Inferior belly of omohyoid


• Scalene

Lymph • Supraclavicular
nodes • Occipital

Question 58 of 438
A 42 year old lady is reviewed in the outpatient clinic following a routine surgical procedure. She complains of
diminished sensation at the lateral aspect of her foot. Which of the following nerves is likely to be affected?

A. Sural

B. Superficial peroneal

C. Deep peroneal

D. Medial plantar

E. Lateral plantar

Theme from April 2012 Exam


The sural nerve supplies the lateral aspect of the foot. It runs alongside the short saphenous vein and may be
injured in short saphenous vein surgery.

Foot- Cutaneous sensation

Region Nerve
Lateral plantar Sural
Dorsum (not 1st web space) Superficial peroneal
1st Web space Deep peroneal
Extremities of toes Medial and lateral plantar nerves
Proximal plantar Tibial
Medial plantar Medial plantar nerve
Lateral plantar Lateral plantar nerve
Question 59 of 438
Which of the following anatomical planes separates the prostate from the rectum?

A. Sibsons fascia

B. Denonvilliers fascia

C. Levator ani muscle

D. Waldeyers fascia

E. None of the above

The Denonvilliers fascia separates the rectum from the prostate. Waldeyers fascia separates the rectum from the
sacrum

Prostate gland

The prostate gland is approximately the shape and size of a walnut and is located inferior to the bladder. It is
separated from the rectum by Denonvilliers fascia and its blood supply is derived from the internal iliac vessels.
The internal sphincter lies at the apex of the gland and may be damaged during prostatic surgery, affected
individuals may complain of retrograde ejaculation.

Summary of prostate gland


Arterial supply Inferior vesical artery (from internal iliac)
Venous drainage Prostatic venous plexus (to paravertebral veins)
Lymphatic Internal iliac nodes
drainage
Innervation Inferior hypogastric plexus
Dimensions • Transverse diameter (4cm)
• AP diameter (2cm)
• Height (3cm)

Lobes • Posterior lobe: posterior to urethra


• Median lobe: posterior to urethra, in between ejaculatory ducts
• Lateral lobes x 2
• Isthmus

Zones • Peripheral zone: subcapsular portion of posterior prostate. Most prostate cancers
are here
• Central zone
• Transition zone
• Stroma

Relations
Anterior Pubic symphysis
Prostatic venous plexus
Posterior Denonvilliers fascia
Rectum
Ejaculatory ducts
Lateral Venous plexus (lies on prostate)
Levator ani (immediately below the puboprostatic ligaments)

Question 60 of 438
A 56 year old lady is undergoing an adrenalectomy for Conns syndrome. During the operation the surgeon
damages the middle adrenal artery and haemorrhage ensues. From which of the following structures does this
vessel originate?

A. Aorta

B. Renal artery

C. Splenic artery

D. Coeliac axis

E. Superior mesenteric artery

The middle adrenal artery is usually a branch of the aorta, the lower adrenal artery typically arises from the
renal vessels.

Adrenal gland anatomy

Anatomy

Location Superomedially to the upper pole of each kidney


Relationships of the right Diaphragm-Posteriorly, Kidney-Inferiorly, Vena Cava-Medially, Hepato-renal
adrenal pouch and bare area of the liver-Anteriorly
Relationships of the left Crus of the diaphragm-Postero- medially, Pancreas and splenic vessels-Inferiorly,
adrenal Lesser sac and stomach-Anteriorly
Superior adrenal arteries- from inferior phrenic artery, Middle adrenal arteries -
Arterial supply from aorta, Inferior adrenal arteries -from renal arteries
Venous drainage of the Via one central vein directly into the IVC
right adrenal
Venous drainage of the Via one central vein into the left renal vein
left adrenal

Question 61 of 438
A 73 year old lady suffers a fracture at the surgical neck of the humerus. The decision is made to operate. There
are difficulties in reducing the fracture and a vessel lying posterior to the surgical neck is injured. Which of the
following is this vessel most likely to be?

A. Axillary artery

B. Brachial artery

C. Thoracoacromial artery

D. Transverse scapular artery

E. Posterior circumflex humeral artery

The circumflex humeral arteries lie at the surgical neck and is this scenario the posterior circumflex is likely to
be injured. The thoracoacromial and transverse scapular arteries lie more superomedially. The posterior
circumflex humeral artery is a branch of the axillary artery.

Shoulder joint

• Shallow synovial ball and socket type of joint.


• It is an inherently unstable joint, but is capable to a wide range of movement.
• Stability is provided by muscles of the rotator cuff that pass from the scapula to insert in the greater
tuberosity (all except sub scapularis-lesser tuberosity).

Glenoid labrum

• Fibrocartilaginous rim attached to the free edge of the glenoid cavity


• Tendon of the long head of biceps arises from within the joint from the supraglenoid tubercle, and is
fused at this point to the labrum.
• The long head of triceps attaches to the infraglenoid tubercle

Fibrous capsule

• Attaches to the scapula external to the glenoid labrum and to the labrum itself (postero-superiorly)
• Attaches to the humerus at the level of the anatomical neck superiorly and the surgical neck inferiorly
• Anteriorly the capsule is in contact with the tendon of subscapularis, superiorly with the supraspinatus
tendon, and posteriorly with the tendons of infraspinatus and teres minor. All these blend with the
capsule towards their insertion.
• Two defects in the fibrous capsule; superiorly for the tendon of biceps. Anteriorly there is a defect
beneath the subscapularis tendon.
• The inferior extension of the capsule is closely related to the axillary nerve at the surgical neck and this
nerve is at risk in anteroinferior dislocations. It also means that proximally sited osteomyelitis may
progress to septic arthritis.

Movements and muscles


Flexion Anterior part of deltoid
Pectoralis major
Biceps
Coracobrachialis
Extension Posterior deltoid
Teres major
Latissimus dorsi
Adduction Pectoralis major
Latissimus dorsi
Teres major
Coracobrachialis
Abduction Mid deltoid
Supraspinatus
Medial rotation Subscapularis
Anterior deltoid
Teres major
Latissimus dorsi
Lateral rotation Posterior deltoid
Infraspinatus
Teres minor

Important anatomical relations


Anteriorly Brachial plexus
Axillary artery and vein
Posterior Suprascapular nerve
Suprascapular vessels
Inferior Axillary nerve
Circumflex humeral vessels

Question 62 of 438
Which of the structures listed below lies posterior to the carotid sheath at the level of the 6th cervical vertebra?

A. Hypoglossal nerve

B. Vagus nerve
C. Cervical sympathetic chain

D. Ansa cervicalis

E. Glossopharyngeal nerve

The carotid sheath is crossed anteriorly by the hypoglossal nerves and the ansa cervicalis. The vagus lies within
it. The cervical sympathetic chain lies posteriorly between the sheath and the prevertebral fascia.

Common carotid artery

The right common carotid artery arises at the bifurcation of the brachiocephalic trunk, the left common carotid
arises from the arch of the aorta. Both terminate at the level of the upper border of the thyroid cartilage (the
lower border of the third cervical vertebra) by dividing into the internal and external carotid arteries.

Left common carotid artery


This vessel arises immediately to the left and slightly behind the origin of the brachiocephalic trunk. Its thoracic
portion is 2.5- 3.5 cm in length and runs superolaterally to the sternoclavicular joint.

In the thorax
The vessel is in contact, from below upwards, with the trachea, left recurrent laryngeal nerve, left margin of the
oesophagus. Anteriorly the left brachiocephalic vein runs across the artery, and the cardiac branches from the
left vagus descend in front of it. These structures together with the thymus and the anterior margins of the left
lung and pleura separate the artery from the manubrium.

In the neck
The artery runs superiorly deep to sternocleidomastoid and then enters the anterior triangle. At this point it lies
within the carotid sheath with the vagus nerve and the internal jugular vein. Posteriorly the sympathetic trunk
lies between the vessel and the prevertebral fascia. At the level of C7 the vertebral artery and thoracic duct lie
behind it. The anterior tubercle of C6 transverse process is prominent and the artery can be compressed against
this structure (it corresponds to the level of the cricoid).
Anteriorly at C6 the omohyoid muscle passes superficial to the artery.
Within the carotid sheath the jugular vein lies lateral to the artery.

Right common carotid artery


The right common carotid arises from the brachiocephalic artery. The right common carotid artery corresponds
with the cervical portion of the left common carotid, except that there is no thoracic duct on the right. The
oesophagus is less closely related to the right carotid than the left.

Summary points about the carotid anatomy

Path
Passes behind the sternoclavicular joint (12% patients above this level) to the upper border of the thyroid
cartilage, to divide into the external (ECA) and internal carotid arteries (ICA).

Relations

• Level of 6th cervical vertebra crossed by omohyoid


• Then passes deep to the thyrohyoid, sternohyoid, sternomastoid muscles.
• Passes anterior to the carotid tubercle (transverse process 6th cervical vertebra)-NB compression here
stops haemorrhage.
• The inferior thyroid artery passes posterior to the common carotid artery.
• Then : Left common carotid artery crossed by thoracic duct, Right common carotid artery crossed by
recurrent laryngeal nerve

Question 63 of 438
A sprinter attends A&E with severe leg pain. He had forgotten to warm up and ran a 100m sprint race. Towards
the end of the race he experienced pain in the posterior aspect of his thigh. The pain worsens, localising to the
lateral aspect of the knee. The sprinter is unable to flex the knee. What structure has been injured?

A. Anterior cruciate ligament

B. Posterior cruciate ligament

C. Semimembranosus tendon

D. Semiteninosus tendon

E. Biceps femoris tendon

Theme from 2009 Exam

The biceps femoris is commonly injured in sports that require explosive bending of the knee as seen in
sprinting, especially if the athlete has not warmed up first. Avulsion most commonly occurs where the long
head attaches to the ischial tuberosity. Injuries to biceps femoris are more common than to the other hamstrings.

Biceps femoris

The biceps femoris is one of the hamstring group of muscles located in the posterior upper thigh. It has two
heads.

Long head
Origin Ischial tuberosity
Insertion Fibular head
Action Knee flexion, lateral rotation tibia, extension hip
Innervation Tibial nerve (L5, S1, S2)
Arterial Profunda femoris artery, inferior gluteal artery, and the superior muscular branches of
supply popliteal artery

Image demonstrating the biceps femoris muscle, with the long head outlined
Image sourced from Wikipedia

Short head
Origin Lateral lip of linea aspera, lateral supracondylar ridge of femur
Insertion Fibular head
Action Knee flexion, lateral rotation tibia
Innervation Common peroneal nerve (L5, S1, S2)
Arterial Profunda femoris artery, inferior gluteal artery, and the superior muscular branches of
supply popliteal artery

Question 64 of 438
A 24 year old man falls and lands astride a manhole cover. He suffers from a injury to the anterior bulbar
urethra. Where will the extravasated urine tend to collect?

A. Lesser pelvis

B. Connective tissue of the scrotum

C. Deep perineal space

D. Ischiorectal fossa

E. Posterior abdominal wall

This portion of the urethra is contained between the perineal membrane an the membranous layer of the
superficial fascia. As these are densely adherent to the ischiopubic rami, extravasated urine cannot pass
posteriorly because the 2 layers are continuous around the superficial transverse perineal muscles.

Lower genitourinary tract trauma


• Most bladder injuries occur due to blunt trauma
• 85% associated with pelvic fractures
• Easily overlooked during assessment in trauma
• Up to 10% of male pelvic fractures are associated with urethral or bladder injuries

Types of injury

Urethral injury • Mainly in males


• Blood at the meatus (50% cases)
• There are 2 types:

i.Bulbar rupture
- most common
- straddle type injury e.g. bicycles
- triad signs: urinary retention, perineal haematoma, blood at the meatus
ii. Membranous rupture
- can be extra or intraperitoneal
- commonly due to pelvic fracture
- Penile or perineal oedema/ hematoma
- PR: prostate displaced upwards (beware co-existing retroperitoneal
haematomas as they may make examination difficult)

- Investigation: ascending urethrogram


- Management: suprapubic catheter (surgical placement, not percutaneously)
External genitalia injuries (i.e., • Secondary to injuries caused by penetration, blunt trauma,
the penis and the scrotum) continence- or sexual pleasure-enhancing devices, and mutilation

Bladder injury • rupture is intra or extraperitoneal


• presents with haematuria or suprapubic pain
• history of pelvic fracture and inability to void: always suspect
bladder or urethral injury
• inability to retrieve all fluid used to irrigate the bladder through a
Foley catheter indicates bladder injury
• investigation- IVU or cystogram
• management: laparotomy if intraperitoneal, conservative if
extraperitoneal

Question 65 of 438
A 73 year old man presents with symptoms of mesenteric ischaemia. As part of his diagnostic work up a
diagnostic angiogram is performed .The radiologist is attempting to cannulate the coeliac axis from the aorta. At
which of the following vertebral levels does this is usually originate?

A. T10

B. L2

C. L3
D. T8

E. T12
Coeliac trunk
branches:

Left Hand Side (LHS)

Left gastric
Hepatic
Splenic

The coeliac axis branches off the aorta at T12.

Coeliac axis

The coeliac axis has three main branches.

• Left gastric
• Hepatic: branches-Right Gastric, Gastroduodenal, Right Gastroepiploic, Superior Pancreaticoduodenal,
Cystic.
• Splenic: branches- Pancreatic, Short Gastric, Left Gastroepiploic

It occasionally gives off one of the inferior phrenic arteries.

Image sourced from Wikipedia

Relations
Anteriorly Lesser omentum
Right Right coeliac ganglion and caudate process of liver
Left Left coeliac ganglion and gastric cardia
Inferiorly Upper border of pancreas and renal vein
Question 66 of 438
A 43 year old man is diagnosed as having a malignancy of the right adrenal gland. The decision is made to
resect this via an open anterior approach. Which of the following will be most useful during the surgery?

A. Division of the coronary ligaments of the liver

B. Mobilisation of the colonic hepatic flexure

C. Division of the right renal vein

D. Division of the ligament of Trietz

E. Division of the right colic artery

Mobilisation of the hepatic flexure and right colon are standard steps in open adrenal surgery from an anterior
approach. Mobilisation of the liver is seldom required.

Adrenal gland anatomy

Anatomy

Location Superomedially to the upper pole of each kidney


Relationships of the right Diaphragm-Posteriorly, Kidney-Inferiorly, Vena Cava-Medially, Hepato-renal
adrenal pouch and bare area of the liver-Anteriorly
Relationships of the left Crus of the diaphragm-Postero- medially, Pancreas and splenic vessels-Inferiorly,
adrenal Lesser sac and stomach-Anteriorly
Superior adrenal arteries- from inferior phrenic artery, Middle adrenal arteries -
Arterial supply from aorta, Inferior adrenal arteries -from renal arteries
Venous drainage of the Via one central vein directly into the IVC
right adrenal
Venous drainage of the Via one central vein into the left renal vein
left adrenal

Question 67 of 438
A 45 year old man presents with a lipoma located posterior to the posterior border of the sternocleidomastoid
muscle, approximately 4cm superior to the middle third of the clavicle. During surgical excision of the lesion
troublesome bleeding is encountered. Which of the following is the most likely source?

A. Internal jugular vein

B. External jugular vein

C. Common carotid artery

D. Vertebral artery
E. Second part of the subclavian artery

The external jugular vein runs obliquely in the superficial fascia of the posterior triangle. It drains into the
subclavian vein. During surgical exploration of this area the external jugular vein may be injured and
troublesome bleeding may result. The internal jugular vein and carotid arteries are located in the anterior
triangle. The third, and not the second, part of the subclavian artery is also a content of the posterior triangle

Posterior triangle of the neck

Boundaries
Apex Sternocleidomastoid and the Trapezius muscles at the Occipital bone
Anterior Posterior border of the Sternocleidomastoid
Posterior Anterior border of the Trapezius
Base Middle third of the clavicle

Image sourced from Wikipedia

Contents
Nerves • Accessory nerve
• Phrenic nerve
• Three trunks of the brachial plexus
• Branches of the cervical plexus: Supraclavicular nerve, transverse cervical nerve, great
auricular nerve, lesser occipital nerve

Vessels • External jugular vein


• Subclavian artery

Muscles • Inferior belly of omohyoid


• Scalene

Lymph • Supraclavicular
nodes • Occipital
2/3 Question 68-70 of 438
Theme: Levels of spinal injury

A. C2
B. C3
C. C4
D. C5
E. C6
F. L1
G. L2
H. L3
I. L4
J. L5

Please select the most likely spinal level for the injury described. Each option may be used once, more than
once or not at all.

68. A 62 year old male complains of back pain. He has had a recent fall. Walking causes pain of the left
lower leg. On examination he is noted to have reduced sensation over the knee.

L3

Sensation over the knee is equivalent to the L3 dermatome. The four nerves involved include the
infrapatellar branch of the saphenous nerve, the lateral cutaneous nerve of the thigh, anterior
cutaneous nerve of the thigh (both lateral and medial branches).

69. A 42 year old woman is found to have a burst fracture of the C5 vertebral body. After a few months
where would the level of injury be?

You answered C5

The correct answer is C6

A C5 burst fracture usually injures the C6 spinal cord situated at the C5 vertebrae and also the C4
spinal roots that exits the spinal column between the C4 and C5 vertebra. Such an injury should cause
a loss of sensations in C4 dermatome and weak deltoids. Due to oedema , the biceps (C5) may be
initially weak but should recover. The wrist extensors (C6), however, should remain weak and
sensation at and below C6 should be severely compromised. A neurosurgeon would conclude that
there is a burst fracture at C5 from the x-rays, an initial sensory level at C4 (the first abnormal
sensory dermatome) and the partial loss of deltoids and biceps would imply a motor level at C4 (the
highest abnormal muscle level). Over time, as the patient recovers the C4 roots and the C5 spinal
cord, both the sensory level and motor level should end up at C6. Such recovery is often attributed to
'root' recovery.

70. A 56 year old man suddenly develops severe back pain. His pain has a radicular pattern. On
examination he is unable to extend his great toe.

L5

Extensor hallucis longus is derived from L5 and loss of EHL function is a useful test to determine
whether this level is involved.

Spinal disorders

Dorsal column lesion • Loss vibration and proprioception


• Tabes dorsalis, SACD

Spinothalamic tract lesion • Loss of pain, sensation and temperature

Central cord lesion • Flaccid paralysis of the upper limbs

Osteomyelitis • Normally progressive


• Staph aureus in IVDU, normally cervical region affected
• Fungal infections in immunocompromised
• Thoracic region affected in TB

Infarction spinal cord • Dorsal column signs (loss of proprioception and fine discrimination

Cord compression • UMN signs


• Malignancy
• Haematoma
• Fracture

Brown-sequard syndrome • Hemisection of the spinal cord


• Ipsilateral paralysis
• Ipsilateral loss of proprioception and fine discrimination
• Contralateral loss of pain and temperature
Image sourced from Wikipedia

Dermatomes

• C2 to C4 The C2 dermatome covers the occiput and the top part of the neck. C3 covers the lower part of
the neck to the clavicle. C4 covers the area just below the clavicle.
• C5 to T1 Situated in the arms. C5 covers the lateral arm at and above the elbow. C6 covers the forearm
and the radial (thumb) side of the hand. C7 is the middle finger, C8 is the lateral aspects of the hand, and
T1 covers the medial side of the forearm.
• T2 to T12 The thoracic covers the axillary and chest region. T3 to T12 covers the chest and back to the
hip girdle. The nipples are situated in the middle of T4. T10 is situated at the umbilicus. T12 ends just
above the hip girdle.
• L1 to L5 The cutaneous dermatome representing the hip girdle and groin area is innervated by L1 spinal
cord. L2 and 3 cover the front part of the thighs. L4 and L5 cover medial and lateral aspects of the lower
leg.
• S1 to S5 S1 covers the heel and the middle back of the leg. S2 covers the back of the thighs. S3 cover
the medial side of the buttocks and S4-5 covers the perineal region. S5 is of course the lowest
dermatome and represents the skin immediately at and adjacent to the anus.

Myotomes

Upper limb
Elbow flexors/Biceps C5
Wrist extensors C6
Elbow extensors/Triceps C7
Long finger flexors C8
Small finger abductors T1

Lower limb
Hip flexors (psoas) L1 and L2
Knee extensors (quadriceps) L3
Ankle dorsiflexors (tibialis anterior) L4 and L5
Toe extensors (hallucis longus) L5
Ankle plantar flexors (gastrocnemius) S1

The anal sphincter is innervated by S2,3,4

Question 71 of 438
The sciatic nerve lies deep to the following structures except:

A. Gluteus maximus

B. The femoral cutaneous nerve

C. Long head of biceps femoris

D. Gluteus medius

E. Branch of the inferior gluteal artery

The gluteus medius does not extend around to the sciatic nerve.

Sciatic nerve
Origin Spinal nerves L4 - S3
Articular Branches Hip joint
Muscular branches in upper • Semitendinosus
leg • Semimembranosus
• Biceps femoris
• Part of adductor magnus

Cutaneous sensation • Posterior aspect of thigh


• Gluteal region
• Entire lower leg (except the medial aspect)

Terminates At the upper part of the popliteal fossa by dividing into the tibial and peroneal
nerves

• The nerve to the short head of the biceps femoris comes from the common peroneal part of the sciatic
and the other muscular branches arise from the tibial portion.
• The tibial nerve goes on to innervate all muscles of the foot except the extensor digitorum brevis (which
is innervated by the common peroneal nerve).

• End and review


• Reference ranges

Question stats

A 15.9%
B 46.7%
C 12.8%
D 10.7%
E 14%

46.7% of users answered this question correctly

Search

Score: 55.6%
1
2
3
4
5
6
7
8
9
10
11
12
13
14
15
16
17
18
19
20
21
22
23
24
25
26
27-29 1 / 3
30
31
32
33
34
35
36
37
38
39
40
41
42
43
44
45
46
47
48
49
50
51
52
53
54
55
56
57
58
59
60
61
62
63
64
65
66
67
68-70 2 / 3
71
72
Question 72 of 438
Which of the following upper limb muscles is not innervated by the radial nerve?

A. Extensor carpi ulnaris

B. Abductor digit minimi

C. Anconeus
D. Supinator

E. Brachioradialis
Mnemonic for radial nerve muscles:
BEST

B rachioradialis
E xtensors
S upinator
T riceps

Abductor digiti minimi is innervated by the ulnar nerve.

Radial nerve

Continuation of posterior cord of the brachial plexus (root values C5 to T1)

Path

• In the axilla: lies posterior to the axillary artery on subscapularis, latissimus dorsi and teres major.
• Enters the arm between the brachial artery and the long head of triceps (medial to humerus).
• Spirals around the posterior surface of the humerus in the groove for the radial nerve.
• At the distal third of the lateral border of the humerus it then pierces the intermuscular septum and
descends in front of the lateral epicondyle.
• At the lateral epicondyle it lies deeply between brachialis and brachioradialis where it then divides into a
superficial and deep terminal branch.
• Deep branch crosses the supinator to become the posterior interosseous nerve.

In the image below the relationships of the radial nerve can be appreciated

Image sourced from Wikipedia


Regions innervated
Motor (main nerve) • Triceps
• Anconeus
• Brachioradialis
• Extensor carpi radialis

Motor (posterior • Extensor carpi ulnaris


interosseous branch • Extensor digitorum
• Extensor indicis
• Extensor digiti minimi
• Extensor pollicis longus and brevis
• Abductor pollicis longus

Sensory The area of skin supplying the proximal phalanges on the dorsal aspect of the hand is
supplied by the radial nerve (this does not apply to the little finger and part of the ring
finger)

Muscular innervation and effect of denervation


Anatomical Muscle affected Effect of paralysis
location
Shoulder Long head of triceps Minor effects on shoulder stability in abduction
Arm Triceps Loss of elbow extension
Forearm Supinator Weakening of supination of prone hand and elbow flexion
Brachioradialis in mid prone position
Extensor carpi radialis longus
and brevis

The cutaneous sensation of the upper limb- illustrating the contribution of the radial nerve
Question 73 of 438

Which of the following forms the floor of the anatomical


snuffbox?
A. Radial artery

B. Cephalic vein

C. Extensor pollicis brevis

D. Scaphoid bone

E. Cutaneous branch of the radial nerve

The scaphoid bone forms the floor of the anatomical


snuffbox. The cutaneous branch of the radial nerve is much
more superficially and proximally located.
Anatomical snuffbox

Posterior Tendon of extensor pollicis longus


border
Anterior Tendons of extensor pollicis brevis and
border abductor pollicis longus
Proximal Styloid process of the radius
border
Distal border Apex of snuffbox triangle
Floor Trapezium and scaphoid
Content Radial artery

Image showing the anatomical snuffbox

Question 74 of 438
During a liver resection a surgeon performs a pringles
manoeuvre to control bleeding. Which of the following
structures will lie posterior to the epiploic foramen at this
level?

A. Hepatic artery

B. Cystic duct

C. Greater omentum

D. Superior mesenteric artery

E. Inferior vena cava


Bleeding from liver trauma or a difficult cholecystectomy
can be controlled with a vascular clamp applied at the
epiploic foramen.

The epiploic foramen has the following boundaries:


Anteriorly (in the free edge of the lesser omentum): Bile duct
to the right, portal vein behind and hepatic artery to the left.
PosteriorlyInferior vena cava
Inferiorly1st part of the duodenum
SuperiorlyCaudate process of the liver

Liver

Structure of the liver


Right lobe • Supplied by right hepatic artery
• Contains Couinard segments V to VIII
(-/+Sg I)

Left lobe • Supplied by the left hepatic artery


• Contains Couinard segments II to IV
(+/- Sg1)

Quadrate • Part of the right lobe anatomically,


lobe functionally is part of the left
• Couinard segment IV
• Porta hepatis lies behind
• On the right lies the gallbladder fossa
• On the left lies the fossa for the
umbilical vein

Caudate • Supplied by both right and left hepatic


lobe arteries
• Couinard segment I
• Lies behind the plane of the porta
hepatis
• Anterior and lateral to the inferior vena
cava
• Bile from the caudate lobe drains into
both right and left hepatic ducts

Detailed knowledge of Couinard segments is not required for


MRCS Part A

• Between the liver lobules are portal canals which


contain the portal triad: Hepatic Artery, Portal Vein,
tributary of Bile Duct.

Relations of the liver


Anterior Postero inferiorly
Diaphragm Oesophagus
Xiphoid process Stomach
Duodenum
Hepatic flexure of colon
Right kidney
Gallbladder
Inferior vena cava

Porta hepatis
Location Postero inferior surface, it joins nearly at right
angles with the left sagittal fossa, and separates
the caudate lobe behind from the quadrate lobe in
front
Transmits • Common hepatic duct
• Hepatic artery
• Portal vein
• Sympathetic and parasympathetic nerve
fibres
• Lymphatic drainage of the liver (and
nodes)

Ligaments
Falciform • 2 layer fold peritoneum from the
ligament umbilicus to anterior liver surface
• Contains ligamentum teres
(remnant umbilical vein)
• On superior liver surface it splits
into the coronary and left triangular
ligaments

Ligamentum Joins the left branch of the portal vein in


teres the porta hepatis
Ligamentum Remnant of ductus venosus
venosum

Arterial supply

• Hepatic artery

Venous

• Hepatic veins
• Portal vein

Nervous supply

• Sympathetic and parasympathetic trunks of coeliac


plexus

Question 75 of 438
A 72 year old lady is suspected of having a femoral hernia. At which of the following sites is it most likely to be
identifiable clinically?

A. Mid inguinal point

B. Above and medial to the pubic tubercle

C. Below and lateral to the pubic tubercle

D. Mid point of the inguinal ligament

E. 3 cm superomedially to the superficial inguinal ring

Femoral hernias exit the femoral canal below and lateral to the pubic tubercle. Femoral hernia occur mainly in
women due to their difference in pelvic anatomy. They are at high risk of strangulation and therefore should be
repaired.

Question 76 of 438
Which muscle is responsible for causing flexion of the distal interphalangeal joint of the ring finger?

A. Flexor digitorum superficialis

B. Lumbricals

C. Palmar interossei

D. Flexor digitorum profundus

E. Flexor digiti minimi brevis

Flexor digitorum superficialis and flexor digitorum profundus are responsible for causing flexion. The
superficialis tendons insert on the bases of the middle phalanges; the profundus tendons insert on the bases of
the distal phalanges. Both tendons flex the wrist, MCP and PIP joints; however, only the profundus tendons flex
the DIP joints.

Hand

Anatomy of the hand


Bones • 8 Carpal bones
• 5 Metacarpals
• 14 phalanges

Intrinsic Muscles 7 Interossei - Supplied by ulnar nerve

• 3 palmar-adduct fingers
• 4 dorsal- abduct fingers
Intrinsic muscles Lumbricals

• Flex MCPJ and extend the IPJ.


• Origin deep flexor tendon and insertion dorsal extensor hood mechanism.
• Innervation: 1st and 2nd- median nerve, 3rd and 4th- deep branch of the ulnar
nerve.

Thenar eminence • Abductor pollicis brevis


• Opponens pollicis
• Flexor pollicis brevis

Hypothenar • Opponens digiti minimi


eminence • Flexor digiti minimi brevis
• Abductor digiti minimi

Question 77 of 438
A 34 year old lady undergoes a thyroidectomy for Graves disease. Post operatively she develops a tense
haematoma in the neck. In which of the following fascial planes will it be contained?

A. Gerotas fascia

B. Waldeyers fascia

C. Pretracheal fascia
D. Sibsons fascia

E. Clavipectoral fascia

The pretracheal fascia encloses the thyroid and is unyielding. Therefore tense haematomas can develop.

Thyroid gland

• Right and left lobes connected by isthmus


• Surrounded by sheath from pretracheal layer of deep fascia
• Apex: Lamina of thyroid cartilage
• Base: 4th-5th tracheal ring
• Pyramidal lobe: from isthmus
• May be attached to foramen caecum at the base of the tongue

Relations
Anteromedially • Sternothyroid
• Superior belly of omohyoid
• Sternohyoid
• Anterior aspect of sternocleidomastoid

Posterolaterally Carotid sheath


Medially • Larynx
• Trachea
• Pharynx
• Oesophagus
• Cricothyroid muscle
• External laryngeal nerve (near superior thyroid artery)
• Recurrent laryngeal nerve (near inferior thyroid artery)

Posterior • Parathyroid glands


• Anastomosis of superior and inferior thyroid arteries

Isthmus • Anteriorly: Sternothyroids, sternohyoids, anterior jugular veins


• Posteriorly: 2nd, 3rd, 4th tracheal rings (attached via Ligament of Berry)

Blood Supply
Arterial • Superior thyroid artery (1st branch of external carotid)
• Inferior thyroid artery (from thyrocervical trunk)
• Thyroidea ima (in 10% of population -from brachiocephalic artery or aorta)

Venous • Superior and middle thyroid veins - into the IJV


• Inferior thyroid vein - into the brachiocephalic veins
Question 78 of 438
A 32 year old lady complains of carpal tunnel syndrome. The carpal tunnel is explored surgically. Which of the
following structures will lie in closest proximity to the hamate bone within the carpal tunnel?

A. The tendon of abductor pollicis longus

B. The tendons of flexor digitorum profundus

C. The tendons of flexor carpi radialis longus

D. Median nerve

E. Radial artery

The carpal tunnel contains nine flexor tendons:

• Flexor digitorum profundus


• Flexor digitorum superficialis
• Flexor pollicis longus

The tendon of flexor digitorum profundus lies deepest in the tunnel and will thus lie nearest to the hamate bone.

Carpal bones

Diagrammatic image of carpal bones

Image sourced from Wikipedia

Key to image
A Scaphoid
B Lunate
C Triquetrum
D Pisiform
E Trapezium
F Trapezoid
G Capitate
H Hamate
1 Radius
2 Ulna
3 Metacarpals

• No tendons attach to: Scaphoid, lunate, triquetrum (stabilised by ligaments)

Question 79 of 438
A 45 year old man sustains a significant head injury and a craniotomy is performed. The sigmoid sinus is
bleeding profusely, into which of the following structures does it drain?

A. Internal jugular vein

B. Straight sinus

C. Petrosal sinus

D. Inferior sagittal sinus

E. External jugular vein

The sigmoid sinus is joined by the inferior petrosal sinus to drain into the internal jugular vein.

Cranial venous sinuses

The cranial venous sinuses are located within the dura mater. They have no valves which is important in the
potential for spreading sepsis. They eventually drain into the internal jugular vein.

They are:

Superior sagittal sinus


Inferior sagittal sinus
Straight sinus
Transverse sinus
Sigmoid sinus
Confluence of sinuses
Occipital sinus
Cavernous sinus

Topography of cranial venous sinuses


Question 80 of 438
Which nerve supplies the interossei of the fourth finger?

A. Radial

B. Median

C. Superficial ulnar

D. Deep ulnar

E. Posterior interosseous
Mnemonic:
PAD and DAB
Palmer interossei
ADduct
Dorsal interossei ABuct

Theme from April 2013 Exam

Interossei

Origin and insertion Nerve supply Actions


Four palmar and four dorsal interossei occupy the spaces between the They are all Dorsal interossei
metacarpal bones. Each palmar interossei originates from the innervated by abduct the fingers,
metacarpal of the digit on which it acts. the ulnar nerve palmar interossei
Each dorsal interossei comes from the surface of the adjacent adduct the fingers
metacarpal on which it acts. As a result the dorsal interossei are twice
the size of the palmar ones.
The interossei tendons, except the first palmar, pass to one or other
side of the metacarpophalangeal joint posterior to the deep transverse
metacarpal ligament. They become inserted into the base of the
proximal phalanx and partly into the extensor hood

Clinical notes
Along with the lumbricals the interossei flex the metacarpophalangeal joints and extend the proximal and distal
interphalangeal joints. They are responsible for fine tuning these movements. When the interossei and
lumbricals are paralysed the digits are pulled into hyperextension by extensor digitorum and a claw hand is
seen.

Question 81 of 438
In which of the following cranial bones does the foramen spinosum lie?

A. Sphenoid bone

B. Frontal bone

C. Temporal bone

D. Occipital bone

E. Parietal bone

The foramen spinosum (which transmits the middle meningeal artery and vein) lies in the sphenoid bone.

Foramina of the base of the skull

Foramen Location Contents


Foramen ovale Sphenoid Otic ganglion
bone V3 (Mandibular nerve:3rd branch of
trigeminal)
Accessory meningeal artery
Lesser petrosal nerve
Emissary veins
Foramen Sphenoid Middle meningeal artery
spinosum bone Meningeal branch of the Mandibular nerve
Foramen Sphenoid Maxillary nerve (V2)
rotundum bone
Foramen lacerum Sphenoid Base of the medial pterygoid plate.
bone Internal carotid artery
Nerve and artery of the pterygoid canal
Jugular foramen Temporal Anterior: inferior petrosal sinus
bone Intermediate: glossopharyngeal, vagus, and accessory nerves.
Posterior: sigmoid sinus (becoming the internal jugular vein) and some
meningeal branches from the occipital and ascending pharyngeal arteries.
Foramen Occipital Anterior and posterior spinal arteries
magnum bone Vertebral arteries
Medulla oblongata
Stylomastoid Temporal Stylomastoid artery
foramen bone Facial nerve
Superior orbital Sphenoid Oculomotor nerve (III)
fissure bone trochlear nerve (IV)
lacrimal, frontal and nasociliary branches of ophthalmic nerve (V1)
abducent nerve (VI)
Superior and inferior ophthalmic vein

Question 82 of 438
Which of the following is not considered a major branch of the descending thoracic aorta?

A. Bronchial artery

B. Mediastinal artery

C. Inferior thyroid artery

D. Posterior intercostal artery

E. Oesophageal artery

The inferior thyroid artery is usually derived from the thyrocervical trunk, a branch of the subclavian artery.

Thoracic aorta

Origin T4
Terminates T12
Relations • Anteriorly (from top to bottom)-root of the left lung, the pericardium, the oesophagus, and
the diaphragm
• Posteriorly-vertebral column, azygos vein
• Right- hemiazygos veins, thoracic duct
• Left- left pleura and lung

Branches • Lateral segmental branches: Posterior intercostal arteries


• Lateral visceral: Bronchial arteries supply bronchial walls and lung excluding the alveoli
• Midline branches: Oesophageal arteries

Question 83 of 438
An 18 year old lady with troublesome hyperhidrosis of the hands and arms is due to undergo a sympathectomy
to treat the condition. Which of the following should the surgeons divide to most effectively treat her condition?
A. Sympathetic ganglia at T1, T2 and T3

B. Sympathetic ganglia at T2 and T3

C. Sympathetic ganglia at T1 and T2

D. Stellate ganglion

E. Superior cervical ganglion

To treat hyperhidrosis the sympathetic ganglia at T2 and T3 should be divided. Dividing the other structures
listed would either carry a risk of Horners syndrome or be ineffective.

Sympathetic nervous system- anatomy

The cell bodies of the pre-ganglionic efferent neurones lie in the lateral horn of the grey matter of the spinal
cord in the thoraco-lumbar regions.
The pre-ganglionic efferents leave the spinal cord at levels T1-L2. These pass to the sympathetic chain.
Lateral branches of the sympathetic chain connect it to every spinal nerve. These post ganglionic nerves will
pass to structures that receive sympathetic innervation at the periphery.

Sympathetic chains
These lie on the vertebral column and run from the base of the skull to the coccyx.
Cervical Lie anterior to the transverse processes of the cervical vertebrae and posterior to the carotid
region sheath.
Thoracic Lie anterior to the neck of the upper ribs and and lateral sides of the lower thoracic
region vertebrae.They are covered by the parietal pleura
Lumbar Enter by passing posterior to the median arcuate ligament. Lie anteriorly to the vertebrae and
region medial to psoas major.

Sympathetic ganglia

• Superior cervical ganglion lies anterior to C2 and C3.


• Middle cervical ganglion (if present) C6
• Stellate ganglion- anterior to transverse process of C7, lies posterior to the subclavian artery, vertebral
artery and cervical pleura.
• Thoracic ganglia are segmentally arranged.
• There are usually 4 lumbar ganglia.

Clinical importance

• Interruption of the head and neck supply of the sympathetic nerves will result in an ipsilateral Horners
syndrome.
• For treatment of hyperhidrosis the sympathetic denervation can be achieved by removing the second and
third thoracic ganglia with their rami. Removal of T1 will cause a Horners syndrome and is therefore not
performed.
• In patients with vascular disease of the lower limbs a lumbar sympathetomy may be performed, either
radiologically or (more rarely now) surgically. The ganglia of L2 and below are disrupted. If L1 is
removed then ejaculation may be compromised (and little additional benefit conferred as the
preganglionic fibres do not arise below L2.

Question 84 of 438
A 44 year old lady is recovering following a transphenoidal hypophysectomy. Unfortunately there is a post
operative haemorrhage. Which of the following features is most likely to occur initially?

A. Cavernous sinus thrombosis

B. Abducens nerve palsy

C. Bi-temporal homonymous hemianopia

D. Inferior homonymous hemianopia

E. Central retinal vein occlusion

The pituitary is covered by a sheath of dura and an expanding haematoma at this site may compress the optic
chiasm in the same manner as an expanding pituitary tumour.

Pituitary Gland

The pituitary gland is located within the sella turcica within the sphenoid bone in the middle cranial fossa. It is
covered by a dural fold and weighs around 0.5g. It is attached to the hypothalamus by the infundibulum. The
anterior pituitary receives hormonal stimuli from the hypothalamus by way of the hypothalamo-pituitary portal
system. It develops from a depression in the wall of the pharynx (Rathkes pouch).

Anterior pituitary hormones

• Growth hormone
• Thyroid stimulating hormone
• ACTH
• Prolactin
• LH and FSH
• Melanocyte releasing hormone

Posterior pituitary hormones

• Oxytocin
• Anti diuretic hormone

Question 85 of 438
During a right hemicolectomy the caecum is mobilised. As the bowel is retracted medially a vessel is injured,
posterior to the colon. Which of the following is the most likely vessel?

A. Right colic artery

B. Inferior vena cava

C. Aorta

D. External iliac artery

E. Gonadal vessels

The gonadal vessels and ureter are important posterior relations that are at risk during a right hemicolectomy.

Caecum

Location • Proximal right colon below the ileocaecal valve


• Intraperitoneal

Posterior relations • Psoas


• Iliacus
• Femoral nerve
• Genitofemoral nerve
• Gonadal vessels

Anterior relations Greater omentum


Arterial supply Ileocolic artery
Lymphatic drainage Mesenteric nodes accompany the venous drainage

• The caecum is the most distensible part of the colon and in complete large bowel obstruction with a
competent ileocaecal valve the most likely site of eventual perforation.

Question 86 of 438
A 53 year old man with a carcinoma of the lower third of the oesophagus is undergoing an
oesophagogastrectomy. As the surgeons mobilise the lower part of the oesophagus, where are they most likely
to encounter the thoracic duct?

A. Anterior to the oesophagus

B. On the left side of the oesophagus

C. On the right side of the oesophagus

D. Immediately anterior to the azygos vein


E. Posterior to the oesophagus

The thoracic duct lies posterior to the oesophagus and passes to the left at the level of the Angle of Louis. It
enters the thorax at T12 together with the aorta.

Thoracic duct

• Continuation of the cisterna chyli in the abdomen.


• Enters the thorax at T12
• Lies posterior to the oesophagus for most of its intrathoracic course. Passes to the left at T5.
• Lymphatics draining the left side of the head and neck join the thoracic duct prior to its insertion into the
left brachiocephalic vein.
• Lymphatics draining the right side of the head and neck drain via the subclavian and jugular trunks into
the right lymphatic duct and thence into the mediastinal trunk and eventually the right brachiocephalic
vein.
• Its location in the thorax makes it prone to injury during oesophageal surgery. Some surgeons administer
cream to patients prior to oesophagectomy so that it is easier to identify the cut ends of the duct.

Question 87 of 438
Which of the following represents the root values of the sciatic nerve?

A. L4 to S3

B. L1 to L4

C. L3 to S1

D. S1 to S4

E. L5 to S1

The sciatic nerve most commonly arises from L4 to S3.

Sciatic nerve

Origin Spinal nerves L4 - S3


Articular Branches Hip joint
Muscular branches in upper • Semitendinosus
leg • Semimembranosus
• Biceps femoris
• Part of adductor magnus

Cutaneous sensation • Posterior aspect of thigh


• Gluteal region
• Entire lower leg (except the medial aspect)
Terminates At the upper part of the popliteal fossa by dividing into the tibial and peroneal
nerves

• The nerve to the short head of the biceps femoris comes from the common peroneal part of the sciatic
and the other muscular branches arise from the tibial portion.
• The tibial nerve goes on to innervate all muscles of the foot except the extensor digitorum brevis (which
is innervated by the common peroneal nerve).

Question 88 of 438
The common peroneal nerve, or its branches, supply the following muscles except:

A. Peroneus longus

B. Tibialis anterior

C. Extensor hallucis longus

D. Flexor digitorum brevis

E. Extensor digitorum longus

Flexor digitorum is supplied by the tibial nerve.

Common peroneal nerve

Derived from the dorsal divisions of the sacral plexus (L4, L5, S1 and S2).

This nerve supplies the skin and fascia of the anterolateral surface of the leg and the dorsum of the foot. It also
innervates the muscles of the anterior and peroneal compartments of the leg, extensor digitorum brevis as well
as the knee, ankle and foot joints.

It is laterally placed within the sciatic nerve. From the bifurcation of the sciatic nerve it passes inferolaterally in
the lateral and proximal part of the popliteal fossa, under the cover of biceps femoris and its tendon. To reach
the posterior aspect of the fibular head. It ends by dividing into the deep and superficial peroneal nerves at the
point where it winds around the lateral surface of the neck of the fibula in the body of peroneus longus,
approximately 2cm distal to the apex of the head of the fibula. It is palpable posterior to the head of the fibula.

Branches
In the thigh Nerve to the short head of biceps
Articular branch (knee)
In the popliteal fossa Lateral cutaneous nerve of the calf
Neck of fibula Superficial and deep peroneal nerves

Question 1 of 350
An 83 year old lady presents with a femoral hernia and undergoes a femoral hernia repair. Which of the
following forms the posterior wall of the femoral canal?

A. Pectineal ligament

B. Lacunar ligament

C. Inguinal ligament

D. Adductor longus

E. Sartorius

Femoral canal

The femoral canal lies at the medial aspect of the femoral sheath. The femoral sheath is a fascial tunnel
containing both the femoral artery laterally and femoral vein medially. The canal lies medial to the vein.

Borders of the femoral canal


Laterally Femoral vein
Medially Lacunar ligament
Anteriorly Inguinal ligament
Posteriorly Pectineal ligament

Image showing dissection of femoral canal


Image sourced from Wikipedia

Contents

• Lymphatic vessels
• Cloquet's lymph node

Physiological significance
Allows the femoral vein to expand to allow for increased venous return to the lower limbs.

Pathological significance
As a potential space, it is the site of femoral hernias. The relatively tight neck places these at high risk of
strangulation.

Question 2 of 350
A 45 year man presents with hand weakness. He is given a piece of paper to hold between his thumb and index
finger. When the paper is pulled, the patient has difficulty maintaining a grip. Grip pressure is maintained by
flexing the thumb at the interphalangeal joint. What is the most likely nerve lesion?

A. Posterior interosseous nerve


B. Deep branch of ulnar nerve

C. Anterior interosseous nerve

D. Superficial branch of the ulnar nerve

E. Radial nerve

Theme from January 2012 exam

This is a description of Froment's sign, which tests for ulnar nerve palsy. It mainly tests for the function of
adductor pollicis. This is supplied by the deep branch of the ulnar nerve. Remember the anterior interosseous
branch, which innervates the flexor pollicis longus (hence causing flexion of the thumb IP joint), branches off
more proximally to the wrist.

Ulnar nerve

Origin

• C8, T1

Supplies (no muscles in the upper arm)

• Flexor carpi ulnaris


• Flexor digitorum profundus
• Flexor digiti minimi
• Abductor digiti minimi
• Opponens digiti minimi
• Adductor pollicis
• Interossei muscle
• Third and fourth lumbricals
• Palmaris brevis

Path

• Posteromedial aspect of ulna to flexor compartment of forearm, then along the ulnar. Passes beneath the
flexor carpi ulnaris muscle, then superficially through the flexor retinaculum into the palm of the hand.
Image sourced from Wikipedia

Branches
Branch Supplies
Articular branch Flexor carpi ulnaris
Medial half of the flexor digitorum profundus
Palmar cutaneous branch (Arises near the middle Skin on the medial part of the palm
of the forearm)
Dorsal cutaneous branch Dorsal surface of the medial part of the hand
Superficial branch Cutaneous fibres to the anterior surfaces of the medial one
and one-half digits
Deep branch Hypothenar muscles
All the interosseous muscles
Third and fourth lumbricals
Adductor pollicis
Medial head of the flexor pollicis brevis

Effects of injury
Damage at the wrist • Wasting and paralysis of intrinsic hand muscles (claw hand)
• Wasting and paralysis of hypothenar muscles
• Loss of sensation medial 1 and half fingers

Damage at the elbow • Radial deviation of the wrist


• Clawing less in 3rd and 4th digits

Question 3 of 350
Which of the following statements relating to the right phrenic nerve is false?

A. It lies deep to the prevertebral layer of deep cervical fascia

B. Crosses posterior to the 2nd part of the subclavian artery

C. It runs on the anterior surface of the scalene muscle

D. On the right side it leaves the mediastinum via the vena cava hiatus at
a level of T8

E. The right phrenic nerve passes over the right atrium

Phrenic nerve

Origin

• C3,4,5

Supplies

• Diaphragm, sensation central diaphragm and pericardium

Path

• The phrenic nerve passes with the internal jugular vein across scalenus anterior. It passes deep to
prevertebral fascia of deep cervical fascia.
• Left: crosses anterior to the 1st part of the subclavian artery.
• Right: Anterior to scalenus anterior and crosses anterior to the 2nd part of the subclavian artery.
• On both sides, the phrenic nerve runs posterior to the subclavian vein and posterior to the internal
thoracic artery as it enters the thorax.

Right phrenic nerve

• In the superior mediastinum: anterior to right vagus and laterally to superior vena cava
• Middle mediastinum: right of pericardium
• It passes over the right atrium to exit the diaphragm at T8
Left phrenic nerve

• Passes lateral to the left subclavian artery, aortic arch and left ventricle
• Passes anterior to the root of the lung
• Pierces the diaphragm alone

Image showing the passage of the phrenic nerve in the neck

Question 4 of 350
Which of the following cranial foramina pairings are incorrect?

A. The foramen lacerum and internal carotid artery.

B. Foramen ovale and mandibular nerve.

C. Optic canal and ophthalmic artery.

D. Optic canal and ophthalmic nerve.

E. Foramen rotundum and maxillary nerve.

Question derived from 2010 and 2011 exams

The optic canal transmits the optic nerve. The ophthalmic nerve traverses the superior orbital fissure.

Foramina of the base of the skull

Foramen Location Contents


Foramen ovale Sphenoid Otic ganglion
bone V3 (Mandibular nerve:3rd branch of
trigeminal)
Accessory meningeal artery
Lesser petrosal nerve
Emissary veins
Foramen Sphenoid Middle meningeal artery
spinosum bone Meningeal branch of the Mandibular nerve
Foramen Sphenoid Maxillary nerve (V2)
rotundum bone
Foramen lacerum Sphenoid Base of the medial pterygoid plate.
bone Internal carotid artery
Nerve and artery of the pterygoid canal
Jugular foramen Temporal Anterior: inferior petrosal sinus
bone Intermediate: glossopharyngeal, vagus, and accessory nerves.
Posterior: sigmoid sinus (becoming the internal jugular vein) and some
meningeal branches from the occipital and ascending pharyngeal arteries.
Foramen Occipital Anterior and posterior spinal arteries
magnum bone Vertebral arteries
Medulla oblongata
Stylomastoid Temporal Stylomastoid artery
foramen bone Facial nerve
Superior orbital Sphenoid Oculomotor nerve (III)
fissure bone trochlear nerve (IV)
lacrimal, frontal and nasociliary branches of ophthalmic nerve (V1)
abducent nerve (VI)
Superior and inferior ophthalmic ve

Question 5 of 350
A 22 year old man is involved in a fight and sustains a stab wound in his upper forearm. On examination there
is a small, but deep laceration. There is an obvious loss of pincer movement involving the thumb and index
finger with minimal loss of sensation. The most likely nerve injury is to the:

A. Ulnar nerve

B. Radial nerve

C. Anterior interosseous nerve

D. Axillary nerve damage

E. Median nerve damage above the elbow

The anterior interosseous nerve is a motor branch of the median nerve just below the elbow. When damaged it
classically causes:

• Pain in the forearm


• Loss of pincer movement of the thumb and index finger (innervates the long flexor muscles of flexor
pollicis longus & flexor digitorum profundus of the index and middle finger)
• Minimal loss of sensation due to lack of a cutaneous branch

Median nerve

The median nerve is formed by the union of a lateral and medial root respectively from the lateral (C5,6,7) and
medial (C8 and T1) cords of the brachial plexus; the medial root passes anterior to the third part of the axillary
artery. The nerve descends lateral to the brachial artery, crosses to its medial side (usually passing anterior to
the artery). It passes deep to the bicipital aponeurosis and the median cubital vein at the elbow.
It passes between the two heads of the pronator teres muscle, and runs on the deep surface of flexor digitorum
superficialis (within its fascial sheath).
Near the wrist it becomes superficial between the tendons of flexor digitorum superficialis and flexor carpi
radialis, deep to palmaris longus tendon. It passes deep to the flexor retinaculum to enter the palm, but lies
anterior to the long flexor tendons within the carpal tunnel.

Branches
Region Branch
Upper arm No branches, although the nerve commonly communicates with the musculocutaneous nerve
Forearm Pronator teres
Flexor carpi radialis
Palmaris longus
Flexor digitorum superficialis
Flexor pollicis longus
Flexor digitorum profundus (only the radial half)
Distal Palmar cutaneous branch
forearm
Hand Motor supply (LOAF)
(Motor)
• Lateral 2 lumbricals
• Opponens pollicis
• Abductor pollicis brevis
• Flexor pollicis brevis

Hand • Over thumb and lateral 2 ½ fingers


(Sensory) • On the palmar aspect this projects proximally, on the dorsal aspect only the distal regions
are innervated with the radial nerve providing the more proximal cutaneous innervation.

Patterns of damage
Damage at wrist

• e.g. carpal tunnel syndrome


• paralysis and wasting of thenar eminence muscles and opponens pollicis (ape hand deformity)
• sensory loss to palmar aspect of lateral (radial) 2 ½ fingers

Damage at elbow, as above plus:


• unable to pronate forearm
• weak wrist flexion
• ulnar deviation of wrist

Anterior interosseous nerve (branch of median nerve)

• leaves just below the elbow


• results in loss of pronation of forearm and weakness of long flexors of thumb and index finger

Topography of the median nerve

Question 6 of 350
A 66 year old man is undergoing a left nephro-ureterectomy. The surgeons remove the ureter, which of the
following is responsible for the blood supply to the proximal ureter?

A. Branches of the renal artery

B. External iliac artery


C. Internal iliac artery

D. Direct branches from the aorta

E. Common iliac artery

The proximal ureter is supplied by branches from the renal artery. For the other feeding vessels - see below.

Ureter

• 25-35 cm long
• Muscular tube lined by transitional epithelium
• Surrounded by thick muscular coat. Becomes 3 muscular layers as it crosses the bony pelvis
• Retroperitoneal structure overlying transverse processes L2-L5
• Lies anterior to bifurcation of iliac vessels
• Blood supply is segmental; renal artery, aortic branches, gonadal branches, common iliac and internal
iliac
• Lies beneath the uterine artery

Question 7 of 350
Which of the following structures does not pass behind the lateral malleolus?

A. Peroneus brevis tendon

B. Sural nerve

C. Short saphenous vein

D. Peroneus longus tendon

E. Tibialis anterior tendon

Tibialis anterior tendon passes at the medial malleolus.

Lateral malleolus

Structures posterior to the lateral malleolus and superficial to superior peroneal retinaculum

• Sural nerve
• Short saphenous vein

Structures posterior to the lateral malleolus and deep to superior peroneal retinaculum

• Peroneus longus tendon


• Peroneus brevis tendon

The calcaneofibular ligament is attached at the lateral malleolus

Question 8 of 350
A 78 year old man presents with symptoms consistent with intermittent claudication. To assess the severity of
his disease you decide to measure his ankle brachial pressure index. To do this you will identify the dorsalis
pedis artery. Which of the following statements relating to this vessel is false?

A. It originates from the peroneal artery

B. It is crossed by the tendon of extensor hallucis brevis

C. Two veins are usually closely related to it

D. It passes under the inferior extensor retinaculum

E. The tendon of extensor hallucis longus lies medial to it.

The dorsalis pedis artery is a direct continuation of the anterior tibial artery.

Foot- anatomy

Arches of the foot


The foot is conventionally considered to have two arches.

• The longitudinal arch is higher on the medial than on the lateral side. The posterior part of the
calcaneum forms a posterior pillar to support the arch. The lateral part of this structure passes via the
cuboid bone and the lateral two metatarsal bones. The medial part of this structure is more important.
The head of the talus marks the summit of this arch, located between the sustentaculum tali and the
navicular bone. The anterior pillar of the medial arch is composed of the navicular bone, the three
cuneiforms and the medial three metatarsal bones.
• The transverse arch is situated on the anterior part of the tarsus and the posterior part of the metatarsus.
The cuneiforms and metatarsal bases narrow inferiorly, which contributes to the shape of the arch.

Intertarsal joints
Sub talar joint Formed by the cylindrical facet on the lower surface of the body of the talus and the
posterior facet on the upper surface of the calcaneus. The facet on the talus is
concave anteroposteriorly, the other is convex. The synovial cavity of this joint does
not communicate with any other joint.
Talocalcaneonavicular The anterior part of the socket is formed by the concave articular surface of the
joint navicular bone, posteriorly by the upper surface of the sustentaculum tali. The talus
sits within this socket
Calcaneocuboid joint Highest point in the lateral part of the longitudinal arch. The lower aspect of this
joint is reinforced by the long plantar and plantar calcaneocuboid ligaments.
Transverse tarsal joint The talocalcaneonavicular joint and the calcaneocuboid joint extend across the tarsus
in an irregular transverse plane, between the talus and calcaneus behind and the
navicular and cuboid bones in front. This plane is termed the transverse tarsal joint.
Cuneonavicular joint Formed between the convex anterior surface of the navicular bone and the concave
surface of the the posterior ends of the three cuneiforms.
Intercuneiform joints Between the three cuneiform bones.
Cuneocuboid joint Between the circular facets on the lateral cuneiform bone and the cuboid. This joint
contributes to the tarsal part of the transverse arch.
A detailed knowledge of the joints is not required for MRCS Part A. However, the contribution they play to the
overall structure of the foot should be appreciated

Ligaments of the ankle joint and foot

Image sourced from Wikipedia

Muscles of the foot


Muscle Origin Insertion Nerve Action
supply
Abductor Medial side of the calcaneus, flexor Medial side of the Medial Abducts the great toe
hallucis retinaculum, plantar aponeurosis base of the plantar
proximal phalanx nerve
Flexor Medial process of the calcaneus, Via 4 tendons into Medial Flexes all the joints of the
digitorum plantar eponeurosis. the middle plantar lateral 4 toes except for the
brevis phalanges of the nerve interphalangeal joint.
lateral 4 toes.
Abductor From the tubercle of the calcaneus and Together with Lateral Abducts the little toe at the
digit from the plantar aponeurosis flexor digit minimi plantar metatarsophalangeal joint
minimi brevis into the nerve
lateral side of the
base of the
proximal phalanx
of the little toe
Flexor From the medial side of the plantar Into the proximal Medial Flexes the
hallucis surface of the cuboid bone, from the phalanx of the plantar metatarsophalangeal joint of
brevis adjacent part of the lateral cuneiform great toe, the nerve the great toe.
bone and from the tendon of tibialis tendon contains a
posterior. sesamoid bone
Adductor Arises from two heads. The oblique Lateral side of the Lateral Adducts the great toe
hallucis head arises from the sheath of the base of the plantar towards the second toe.
peroneus longus tendon, and from the proximal phalanx nerve Helps maintain the
plantar surfaces of the bases of the 2nd, of the great toe. transverse arch of the foot.
3rd and 4th metatarsal bones. The
transverse head arises from the plantar
surface of the lateral 4
metatarsophalangeal joints and from
the deep transverse metatarsal
ligament.
Extensor On the dorsal surface of the foot from Via four thin Deep Extend the
digitorum the upper surface of the calcaneus and tendons which run peroneal metatarsophalangeal joint of
brevis its associated fascia forward and the medial four toes. It is
medially to be unable to extend the
inserted into the interphalangeal joint without
medial four toes. the assistance of the
The lateral three lumbrical muscles.
tendons join with
hoods of extensor
digitorum longus.

Detailed knowledge of the foot muscles are not needed for the MRCS part A

Nerves in the foot

Lateral plantar nerve


Passes anterolaterally towards the base of the 5th metatarsal between flexor digitorum brevis and flexor
accessorius. On the medial aspect of the lateral plantar artery. At the base of the 5th metatarsal it splits into
superficial and deep branches.

Medial plantar nerve


Passes forwards with the medial plantar artery under the cover of the flexor retinaculum to the interval between
abductor hallucis and flexor digitorum brevis on the sole of the foot.

Plantar arteries
Arise under the cover of the flexor retinaculum, midway between the tip of the medial malleolus and the most
prominent part of the medial side of the heel.

• Medial plantar artery. Passes forwards medial to medial plantar nerve in the space between abductor
hallucis and flexor digitorum brevis.Ends by uniting with a branch of the 1st plantar metatarsal artery.
• Lateral plantar artery. Runs obliquely across the sole of the foot. It lies lateral to the lateral plantar
nerve. At the base of the 5th metatarsal bone it arches medially across the foot on the metatarsals
Dorsalis pedis artery
This vessel is a direct continuation of the anterior tibial artery. It commences on the front of the ankle joint and
runs to the proximal end of the first metatarsal space. Here is gives off the arcuate artery and continues forwards
as the first dorsal metatarsal artery. It is accompanied by two veins throughout its length. It is crossed by the
extensor hallucis brevis

Question 9 of 350
Which of the following is not a content of the anterior triangle of the neck?

A. Vagus nerve

B. Submandibular gland

C. Phrenic nerve

D. Internal jugular vein

E. Hypoglossal nerve

The phrenic nerve is a content of the posterior triangle. The anterior triangle contains the carotid sheath and its
contents.

Anterior triangle of the neck

Boundaries
Anterior border of the Sternocleidomastoid
Lower border of mandible
Anterior midline

Sub triangles (divided by Digastric above and Omohyoid)

• Muscular triangle: Neck strap muscles


• Carotid triangle: Carotid sheath
• Submandibular Triangle (digastric)

Contents of the anterior triangle


Digastric triangle Submandibular gland
Submandibular nodes
Facial vessels
Hypoglossal nerve
Muscular triangle Strap muscles
External jugular vein
Carotid triangle Carotid sheath (Common carotid, vagus and internal jugular vein)
Ansa cervicalis

Nerve supply to digastric muscle


• Anterior: Mylohyoid nerve
• Posterior: Facial nerve

Question 10 of 350
A 32 year old attends neurology clinic complaining of tingling in his hand. He has radial deviation of his wrist
and there is mild clawing of his fingers, with the 3rd and 4th digits being relatively spared. What is the most
likely lesion?

A. Ulnar nerve damage at the wrist

B. Ulnar nerve damage at the elbow

C. Radial nerve damage at the elbow

D. Median nerve damage at the wrist

E. Median nerve damage at the elbow

At the elbow the ulnar nerve lesion affects the flexor carpi ulnaris and flexor digitorum profundus.

Ulnar nerve

Origin

• C8, T1

Supplies (no muscles in the upper arm)

• Flexor carpi ulnaris


• Flexor digitorum profundus
• Flexor digiti minimi
• Abductor digiti minimi
• Opponens digiti minimi
• Adductor pollicis
• Interossei muscle
• Third and fourth lumbricals
• Palmaris brevis

Path

• Posteromedial aspect of ulna to flexor compartment of forearm, then along the ulnar. Passes beneath the
flexor carpi ulnaris muscle, then superficially through the flexor retinaculum into the palm of the hand.
Image sourced from Wikipedia

Branches
Branch Supplies
Articular branch Flexor carpi ulnaris
Medial half of the flexor digitorum profundus
Palmar cutaneous branch (Arises near the middle Skin on the medial part of the palm
of the forearm)
Dorsal cutaneous branch Dorsal surface of the medial part of the hand
Superficial branch Cutaneous fibres to the anterior surfaces of the medial one
and one-half digits
Deep branch Hypothenar muscles
All the interosseous muscles
Third and fourth lumbricals
Adductor pollicis
Medial head of the flexor pollicis brevis

Effects of injury
Damage at the wrist • Wasting and paralysis of intrinsic hand muscles (claw hand)
• Wasting and paralysis of hypothenar muscles
• Loss of sensation medial 1 and half fingers

Damage at the elbow • Radial deviation of the wrist


• Clawing less in 3rd and 4th digits

Question 11 of 350
A 22 year old man is undergoing an endotracheal intubation. Which of the following vertebral levels is
consistent with the origin of the trachea?

A. C2

B. T1

C. C6

D. C4

E. C3

The trachea commences at C6. It terminates at the level of T5 (or T6 in tall subjects in deep inspiration).

Trachea

Trachea
Location C6 vertebra to the upper border of T5 vertebra (bifurcation)
Arterial and venous supply Inferior thyroid arteries and the thyroid venous plexus.
Nerve Branches of vagus, sympathetic and the recurrent nerves

Relations in the neck


Anterior(Superior to inferior) • Isthmus of the thyroid gland
• Inferior thyroid veins
• Arteria thyroidea ima (when that vessel exists)
• Sternothyroid
• Sternohyoid
• Cervical fascia
• Anastomosing branches between the anterior jugular veins

Posterior Oesophagus.
Laterally • Common carotid arteries
• Right and left lobes of the thyroid gland
• Inferior thyroid arteries
• Recurrent laryngeal nerves

Relations in the thorax

Anterior

• Manubrium, the remains of the thymus, the aortic arch, left common carotid arteries, and the deep
cardiac plexus

Lateral

• In the superior mediastinum, on the right side is the pleura and right vagus; on its left side are the left
recurrent nerve, the aortic arch, and the left common carotid and subclavian arteries.

Question 12 of 350
A young child undergoes a difficult craniotomy for fulminant mastoiditis and associated abscess. During the
procedure the trigeminal nerve is severely damaged within Meckels cave. Which deficit is least likely to be
present?

A. Anaesthesia over the ipsilateral anterior aspect of the scalp

B. Loss of the corneal reflex

C. Weakness of the ipsilateral masseter muscle

D. Anaesthesia of the anterior aspect of the tongue

E. Anaesthesia over the entire ipsilateral side of the face

The angle of the jaw is not innervated by sensory fibres of the trigeminal nerve and is spared in this type of
injury.
Remember the trigeminal nerve provides motor innervation to the muscles of mastication.

Trigeminal nerve

The trigeminal nerve is the main sensory nerve of the head. In addition to its major sensory role, it also
innervates the muscles of mastication.

Distribution of the trigeminal nerve


Sensory • Scalp
• Face
• Oral cavity (and teeth)
• Nose and sinuses
• Dura mater

Motor • Muscles of mastication


• Mylohyoid
• Anterior belly of digastric
• Tensor tympani
• Tensor palati

Autonomic connections (ganglia) • Ciliary


• Sphenopalatine
• Otic
• Submandibular

Path

• Originates at the pons


• Sensory root forms the large, crescentic trigeminal ganglion within Meckel's cave, and contains the cell
bodies of incoming sensory nerve fibres. Here the 3 branches exit.
• The motor root cell bodies are in the pons and the motor fibres are distributed via the mandibular nerve.
The motor root is not part of the trigeminal ganglion.

Branches of the trigeminal nerve


Ophthalmic nerve Sensory only
Maxillary nerve Sensory only
Mandibular nerve Sensory and motor

Sensory
Ophthalmic Exits skull via the superior orbital fissure
Sensation of: scalp and forehead, the upper eyelid, the conjunctiva and cornea of the eye, the
nose (including the tip of the nose, except alae nasi), the nasal mucosa, the frontal sinuses, and
parts of the meninges (the dura and blood vessels).
Maxillary Exit skull via the foramen rotundum
nerve Sensation: lower eyelid and cheek, the nares and upper lip, the upper teeth and gums, the nasal
mucosa, the palate and roof of the pharynx, the maxillary, ethmoid and sphenoid sinuses, and
parts of the meninges.
Mandibular Exit skull via the foramen ovale
nerve Sensation: lower lip, the lower teeth and gums, the chin and jaw (except the angle of the jaw),
parts of the external ear, and parts of the meninges.

Motor
Distributed via the mandibular nerve.
The following muscles of mastication are innervated:

• Masseter
• Temporalis
• Medial pterygoid
• Lateral pterygoid

Other muscles innervated include:

• Tensor veli palatini


• Mylohyoid
• Anterior belly of digastric
• Tensor tympani

2/3 Question 13-15 of 350


Theme: Nerve lesions

A. Iliohypogastric nerve
B. Ilioinguinal nerve
C. Lateral cutaneous nerve of the thigh
D. Femoral nerve
E. Saphenous nerve
F. Genitofemoral nerve

Please select the most likely nerve implicated in the situation described. Each option may be used once, more
than once or not at all.

13. A 42 year old woman complains of a burning pain of her anterior thigh which worsens on walking.
There is a positive tinel sign over the inguinal ligament.

You answered Genitofemoral nerve

The correct answer is Lateral cutaneous nerve of the thigh

The lateral cutaneous nerve supplies sensation to the anterior and lateral aspect of the thigh.
Entrapment is commonly due to intra and extra pelvic causes. Treatment involves local anaesthetic
injections.

14. A 29 year old woman has had a Pfannenstiel incision. She has pain over the inguinal ligament which
radiates to the lower abdomen. There is tenderness when the inguinal canal is compressed.

Ilioinguinal nerve

15. A 22 year man is shot in the groin. On examination he has weak hip flexion, weak knee extension,
and impaired quadriceps tendon reflex, as well as sensory deficit in the anteromedial aspect of the
thigh.

Femoral nerve

This is a classical description of a femoral nerve injury.

Nerve lesions during surgery

A variety of different procedures carry the risk of iatrogenic nerve injury. These are important not only from the
patients perspective but also from a medicolegal standpoint.

The following operations and their associated nerve lesions are listed here:
• Posterior triangle lymph node biopsy and accessory nerve lesion.
• Lloyd Davies stirrups and common peroneal nerve.
• Thyroidectomy and laryngeal nerve.
• Anterior resection of rectum and hypogastric autonomic nerves.
• Axillary node clearance; long thoracic nerve, thoracodorsal nerve and intercostobrachial nerve.
• Inguinal hernia surgery and ilioinguinal nerve.
• Varicose vein surgery- sural and saphenous nerves.
• Posterior approach to the hip and sciatic nerve.
• Carotid endarterectomy and hypoglossal nerve.

There are many more, with sound anatomical understanding of the commonly performed procedures the
incidence of nerve lesions can be minimised. They commonly occur when surgeons operate in an unfamiliar
tissue plane or by blind placement of haemostats (not recommended).

Question 16 of 350
Which of the following is not a branch of the external carotid artery?

A. Facial artery

B. Lingual artery

C. Superior thyroid artery

D. Mandibular artery

E. Maxillary artery
External carotid artery branches mnemonic:

'Some Angry Lady Figured Out PMS'

Superior thyroid (superior laryngeal artery


branch)
Ascending pharyngeal
Lingual
Facial (tonsillar and labial artery)
Occipital
Posterior auricular
Maxillary (inferior alveolar artery, middle
meningeal artery)
Superficial temporal

External carotid artery

The external carotid commences immediately lateral to the pharyngeal side wall. It ascends and lies anterior to
the internal carotid and posterior to the posterior belly of digastric and stylohyoid. More inferiorly it is covered
by sternocleidomastoid, passed by hypoglossal nerves, lingual and facial veins.
It then pierces the fascia of the parotid gland finally dividing into its terminal branches within the gland itself.

Surface marking of the carotid


This is an imaginary line drawn from the bifurcation of the common carotid passing behind the angle of the jaw
to a point immediately anterior to the tragus of the ear.

Branches of the external carotid artery


It has six branches, three in front, two behind and one deep.
Three in front Superior thyroid
Lingual
Facial
Two behind Occipital
Posterior auricular
Deep Ascending pharyngeal

It terminates by dividing into the superficial temporal and maxillary arteries in the parotid gland.

Question 17 of 350
A 23 year old man is stabbed in the groin, several structures are injured and the adductor longus muscle has
been lacerated. Which of the following nerves is responsible for the innervation of adductor longus?

A. Femoral nerve

B. Obturator nerve
C. Sciatic nerve

D. Common peroneal nerve

E. Ilioinguinal nerve

The adductors are innervated by the obturator nerve

Adductor longus

Origin Anterior body of pubis


Insertion Middle third of linea aspera
Action Adducts and flexes the thigh, medially rotate the hip
Innervation Anterior division of obturator nerve (L2, L3, L4)

The schematic image below demonstrates the relationship of the adductor muscles

Question 18 of 350
Which of the following statements relating to the basilar artery and its branches is false?

A. The superior cerebellar artery may be decompressed to treat


trigeminal neuralgia
B. Occlusion of the posterior cerebral artery causes contralateral loss of
the visual field

C. The oculomotor nerve lies between the superior cerebellar and


posterior cerebral arteries

D. The posterior inferior cerebellar artery is the largest of the cerebellar


arteries arising from the basilar artery

E. The labyrinthine branch is accompanied by the facial nerve

The posterior inferior cerebellar artery is the largest of the cerebellar arteries arising from the vertebral artery.

Circle of Willis

May also be called the circulus arteriosus

• Inferior surface of brain


• Supplied by the internal carotid arteries and the vertebral arteries
• If artery is occluded, collaterals may be able to compensate
• Components include:

1. Anterior communicating arteries


2. Anterior cerebral arteries
3. Internal carotid arteries
4. Posterior communicating arteries
5. Posterior cerebral arteries and the termination of the basilar artery

• Supply: Corpus striatum, internal capsule, diencephalon, midbrain


Image sourced from Wikipedia

Vertebral arteries

• Enter the cranial cavity via foramen magnum


• Lie in the subarachnoid space
• Ascend on anterior surface of medulla oblongata
• Unite to form the basilar artery at the base of the pons

Branches:

• Posterior spinal artery


• Anterior spinal artery
• Posterior inferior cerebellar artery

Basilar artery
Branches:

• Anterior inferior cerebellar artery


• Labyrinthine artery
• Pontine arteries
• Superior cerebellar artery
• Posterior cerebral artery

Internal carotid arteries


Branches:

• Posterior communicating artery


• Anterior cerebral artery
• Middle cerebral artery
• Anterior choroid artery

Question 19 of 350
Which of the following muscles does not recieve any innervation from the sciatic nerve?

A. Semimembranosus

B. Quadriceps femoris

C. Biceps femoris

D. Semitendinosus

E. Adductor magnus

The sciatic nerve is traditionally viewed as being a nerve of the posterior compartment. It is known to contribute
to the innervation of adductor magnus (although the main innervation to this muscle is from the obturator
nerve). The quadriceps femoris is nearly always innervated by the femoral nerve.

Sciatic nerve

Origin Spinal nerves L4 - S3


Articular Branches Hip joint
Muscular branches in upper • Semitendinosus
leg • Semimembranosus
• Biceps femoris
• Part of adductor magnus

Cutaneous sensation • Posterior aspect of thigh


• Gluteal region
• Entire lower leg (except the medial aspect)

Terminates At the upper part of the popliteal fossa by dividing into the tibial and peroneal
nerves

• The nerve to the short head of the biceps femoris comes from the common peroneal part of the sciatic
and the other muscular branches arise from the tibial portion.
• The tibial nerve goes on to innervate all muscles of the foot except the extensor digitorum brevis (which
is innervated by the common peroneal nerve).

Question 20 of 350
A 23 year old man is involved in a fight and is stabbed in his upper arm. The ulnar nerve is transected. Which of
the following muscles will not demonstrate compromised function as a result?

A. Flexor carpi ulnaris

B. Medial half of flexor digitorum profundus

C. Palmaris brevis

D. Hypothenar muscles

E. Pronator teres
M edial lumbricals
A dductor pollicis
F lexor digitorum profundus/Flexor digiti minimi
I nterossei
A bductor digiti minimi and opponens

Innervates all intrinsic muscles of the hand (EXCEPT 2: thenar muscles & first two lumbricals - supplied by
median nerve)

Pronator teres is innervated by the median nerve. Palmaris brevis is innervated by the ulnar nerve

Question 21 of 350
Which of the structures listed below overlies the cephalic vein?

A. Extensor retinaculum

B. Bicipital aponeurosis

C. Biceps muscle

D. Antebrachial fascia

E. None of the above

The cephalic vein is superficially located in the upper limb and overlies most the fascial planes. It pierces the
coracoid membrane (continuation of the clavipectoral fascia) to terminate in the axillary vein. It lies
anterolaterally to biceps.

Cephalic vein
Path

• Dorsal venous arch drains laterally into the cephalic vein


• Crosses the anatomical snuffbox and travels laterally up the arm
• At the antecubital fossa connected to the basilic vein by the median cubital vein
• Pierces deep fascia of deltopectoral groove to join axillary vein

Question 22 of 350
Which of the following pairings are incorrect?

A. Aortic bifurcation and L4

B. Transpyloric plane and L1

C. Termination of dural sac and L4

D. Oesophageal passage through diaphragm and T10

E. Transition between pharynx and oesophagus at C6


Vena cava T8 (eight letters)
Oesophagus T10 (ten letters)
Aortic hiatus T12 (twelve
letters)

It terminates at S2, which is why it is safe to undertake an LP at L4/5 levels. The spinal cord itself terminates at
L1.

Levels

Transpyloric plane
Level of the body of L1

• Pylorus stomach
• Left kidney hilum (L1- left one!)
• Right hilum of the kidney (1.5cm lower than the left)
• Fundus of the gallbladder
• Neck of pancreas
• Duodenojejunal flexure
• Superior mesenteric artery
• Portal vein
• Left and right colic flexure
• Root of the transverse mesocolon
• 2nd part of the duodenum
• Upper part of conus medullaris
• Spleen

Can be identified by asking the supine patient to sit up without using their arms. The plane is located where the
lateral border of the rectus muscle crosses the costal margin.
Anatomical planes
Subcostal plane Lowest margin of 10th costal cartilage
Intercristal plane Level of body L4 (highest point of iliac crest)
Intertubercular plane Level of body L5

Common level landmarks


Inferior mesenteric artery L3
Bifurcation of aorta into common iliac arteries L4
Formation of IVC L5 (union of common iliac veins)
Diaphragm apertures • Vena cava T8
• Oesophagus T10
• Aortic hiatus T12

Question 23 of 350
A 22 year old man is involved in a fight. He sustains a laceration to the posterior aspect of his wrist. In the
emergency department the wound is explored and the laceration is found to be transversely orientated and
overlies the region of the extensor retinaculum, which is intact. Which of the following structures is least likely
to be injured in this scenario?

A. Dorsal cutaneous branch of the ulnar nerve

B. Tendon of extensor indicis

C. Basilic vein

D. Superficial branch of the radial nerve

E. Cephalic vein

The extensor retinaculum attaches to the radius proximal to the styloid, thereafter it runs obliquely and distally
to wind around the ulnar styloid (but does not attach to it). The extensor tendons lie deep to the extensor
retinaculum and would therefore be less susceptible to injury than the superficial structures.

Extensor retinaculum

The extensor rentinaculum is a thickening of the deep fascia that stretches across the back of the wrist and holds
the long extensor tendons in position.
Its attachments are:

• The pisiform and hook of hamate medially


• The end of the radius laterally

Structures related to the extensor retinaculum


Structures superficial to the retinaculum • Basilic vein
• Dorsal cutaneous branch of the ulnar nerve
• Cephalic vein
• Superficial branch of the radial nerve

Structures passing deep to the extensor • Extensor carpi ulnaris tendon


retinaculum • Extensor digiti minimi tendon
• Extensor digitorum and extensor indicis tendon
• Extensor pollicis longus tendon
• Extensor carpi radialis longus tendon
• Abductor pollicis longus and extensor pollicis brevis
tendons

Beneath the extensor retinaculum fibrous septa form six compartments that contain the extensor muscle
tendons. Each compartment has its own synovial sheath.

The radial artery


The radial artery passes between the lateral collateral ligament of the wrist joint and the tendons of the abductor
pollicis longus and extensor pollicis brevis.

Image illustrating the topography of tendons passing under the extensor retinaculum

Question 24 of 350
Which of the following is not a content of the porta hepatis?
A. Portal vein

B. Hepatic artery

C. Cystic duct

D. Hepatic lymph nodes

E. None of the above

The cystic duct lies outside the porta hepatis and is an important landmark in laparoscopic cholecystectomy.
The structures in the porta hepatis are:

• Portal vein
• Hepatic artery
• Common hepatic duct

These structures divide immediately after or within the porta hepatis to supply the functional left and right lobes
of the liver.
The porta hepatis is also surrounded by lymph nodes, that may enlarge to produce obstructive jaundice and
parasympathetic nervous fibres that travel along vessels to enter the liver.

Liver

Structure of the liver


Right lobe • Supplied by right hepatic artery
• Contains Couinard segments V to VIII (-/+Sg I)

Left lobe • Supplied by the left hepatic artery


• Contains Couinard segments II to IV (+/- Sg1)

Quadrate lobe • Part of the right lobe anatomically, functionally is part of the left
• Couinard segment IV
• Porta hepatis lies behind
• On the right lies the gallbladder fossa
• On the left lies the fossa for the umbilical vein

Caudate lobe • Supplied by both right and left hepatic arteries


• Couinard segment I
• Lies behind the plane of the porta hepatis
• Anterior and lateral to the inferior vena cava
• Bile from the caudate lobe drains into both right and left hepatic ducts

Detailed knowledge of Couinard segments is not required for MRCS Part A

• Between the liver lobules are portal canals which contain the portal triad: Hepatic Artery, Portal Vein,
tributary of Bile Duct.
Relations of the liver
Anterior Postero inferiorly
Diaphragm Oesophagus
Xiphoid process Stomach
Duodenum
Hepatic flexure of colon
Right kidney
Gallbladder
Inferior vena cava

Porta hepatis
Location Postero inferior surface, it joins nearly at right angles with the left sagittal fossa, and separates the
caudate lobe behind from the quadrate lobe in front
Transmits • Common hepatic duct
• Hepatic artery
• Portal vein
• Sympathetic and parasympathetic nerve fibres
• Lymphatic drainage of the liver (and nodes)

Ligaments
Falciform ligament • 2 layer fold peritoneum from the umbilicus to anterior liver surface
• Contains ligamentum teres (remnant umbilical vein)
• On superior liver surface it splits into the coronary and left triangular ligaments

Ligamentum teres Joins the left branch of the portal vein in the porta hepatis
Ligamentum Remnant of ductus venosus
venosum

Arterial supply

• Hepatic artery

Venous

• Hepatic veins
• Portal vein

Nervous supply

• Sympathetic and parasympathetic trunks of coeliac plexus

Question 25 of 350
Which of the following structures is not closely related to the carotid sheath?

A. Sternothyroid muscle

B. Sternohyoid muscle

C. Hypoglossal nerve

D. Superior belly of omohyoid muscle

E. Anterior belly of digastric muscle

At its lower end the carotid sheath is related to sternohyoid and sternothyroid. Opposite the cricoid cartilage the
sheath is crossed by the superior belly of omohyoid. Above this level the sheath is covered by the
sternocleidomastoid muscle. Above the level of the hyoid the vessels pass deep to the posterior belly of
digastric and stylohyoid. Opposite the hyoid bone the sheath is crossed obliquely by the hypoglossal nerve.

Common carotid artery

The right common carotid artery arises at the bifurcation of the brachiocephalic trunk, the left common carotid
arises from the arch of the aorta. Both terminate at the level of the upper border of the thyroid cartilage (the
lower border of the third cervical vertebra) by dividing into the internal and external carotid arteries.

Left common carotid artery


This vessel arises immediately to the left and slightly behind the origin of the brachiocephalic trunk. Its thoracic
portion is 2.5- 3.5 cm in length and runs superolaterally to the sternoclavicular joint.

In the thorax
The vessel is in contact, from below upwards, with the trachea, left recurrent laryngeal nerve, left margin of the
oesophagus. Anteriorly the left brachiocephalic vein runs across the artery, and the cardiac branches from the
left vagus descend in front of it. These structures together with the thymus and the anterior margins of the left
lung and pleura separate the artery from the manubrium.

In the neck
The artery runs superiorly deep to sternocleidomastoid and then enters the anterior triangle. At this point it lies
within the carotid sheath with the vagus nerve and the internal jugular vein. Posteriorly the sympathetic trunk
lies between the vessel and the prevertebral fascia. At the level of C7 the vertebral artery and thoracic duct lie
behind it. The anterior tubercle of C6 transverse process is prominent and the artery can be compressed against
this structure (it corresponds to the level of the cricoid).
Anteriorly at C6 the omohyoid muscle passes superficial to the artery.
Within the carotid sheath the jugular vein lies lateral to the artery.

Right common carotid artery


The right common carotid arises from the brachiocephalic artery. The right common carotid artery corresponds
with the cervical portion of the left common carotid, except that there is no thoracic duct on the right. The
oesophagus is less closely related to the right carotid than the left.

Summary points about the carotid anatomy

Path
Passes behind the sternoclavicular joint (12% patients above this level) to the upper border of the thyroid
cartilage, to divide into the external (ECA) and internal carotid arteries (ICA).

Relations

• Level of 6th cervical vertebra crossed by omohyoid


• Then passes deep to the thyrohyoid, sternohyoid, sternomastoid muscles.
• Passes anterior to the carotid tubercle (transverse process 6th cervical vertebra)-NB compression here
stops haemorrhage.
• The inferior thyroid artery passes posterior to the common carotid artery.
• Then : Left common carotid artery crossed by thoracic duct, Right common carotid artery crossed by
recurrent laryngeal nerve

Question 26 of 350
A 21 year old develops tonsillitis. He is in considerable pain. Which of the following nerves is responsible for
the sensory innervation of the tonsillar fossa?

A. Facial nerve

B. Trigeminal nerve

C. Glossopharyngeal nerve

D. Hypoglossal nerve

E. Vagus

The glossopharyngeal nerve is the main sensory nerve for the tonsillar fossa. A lesser contribution is made by
the lesser palatine nerve. Because of this otalgia may occur following tonsillectomy.

Tonsil

Anatomy

• Each palatine tonsil has two surfaces, a medial surface which projects into the pharynx and a lateral
surface that is embedded in the wall of the pharynx.
• They are usually 25mm tall by 15mm wide, although this varies according to age and may be almost
completely atrophied in the elderly.
• Their arterial supply is from the tonsillar artery, a branch of the facial artery.
• Its veins pierce the constrictor muscle to join the external palatine or facial veins. The external palatine
vein is immediately lateral to the tonsil, which may result in haemorrhage during tonsillectomy.
• Lymphatic drainage is the jugulodigastric node and the deep cervical nodes.

Tonsillitis

• Usually bacterial (50%)- group A Streptococcus. Remainder viral.


• May be complicated by development of abscess (quinsy). This may distort the uvula.
- Indications for tonsillectomy include recurrent acute tonsillitis, suspected malignancy, enlargement causing
sleep apnoea.
- Dissection tonsillectomy is the preferred technique with haemorrhage being the commonest complication.
Delayed otalgia may occur owing to irritation of the glossopharyngeal nerve.

Question 27 of 350
A man is stabbed in the abdomen during a fight. He is brought to the emergency department. On examination
there is a laceration in the anterior abdominal wall immediately lateral to the left rectus abdominis muscle on a
level with the upper border of the first lumbar vertebra. Which of the following structures is most likely to have
been injured?

A. Head of the pancreas

B. Gastric antrum

C. Spleen

D. Right lobe of the liver

E. Superior mesenteric artery

The spleen is the most likely target in this instance.

Question 28 of 350
A man has an incision sited than runs 8cm from the deltopectoral groove to the midline. Which of the following
is not at risk of injury?

A. Cephalic vein

B. Shoulder joint capsule

C. Axillary artery

D. Pectoralis major

E. Trunk of the brachial plexus

Theme from April 2012 Exam


This region will typically lie medial to the joint capsule. The diagram below illustrates the plane that this would
transect and as it can be appreciated the other structures are all at risk of injury.
Image sourced from Wikipedia

Pectoralis major muscle

Origin From the medial two thirds of the clavicle, manubrium and sternocostal angle
Insertion Crest of the greater tubercle of the humerus
Nerve supply Lateral pectoral nerve
Actions Adductor and medial rotator of the humerus

Question 29 of 350
A surgeon is due to perform a laparotomy for perforated duodenal ulcer. An upper midline incision is to be
performed. Which of the following structures is the incision most likely to divide?

A. Rectus abdominis muscle


B. External oblique muscle

C. Linea alba

D. Internal oblique muscle

E. None of the above

Theme from September 2011 Exam


Upper midline abdominal incisions will involve the division of the linea alba. Division of muscles will not
usually improve access in this approach and they would not be routinely encountered during this incision.

Abdominal incisions

Midline incision • Commonest approach to the abdomen


• Structures divided: linea alba, transversalis fascia, extraperitoneal fat, peritoneum
(avoid falciform ligament above the umbilicus)
• Bladder can be accessed via an extraperitoneal approach through the space of
Retzius

Paramedian • Parallel to the midline (about 3-4cm)


incision • Structures divided/retracted: anterior rectus sheath, rectus (retracted), posterior
rectus sheath, transversalis fascia, extraperitoneal fat, peritoneum
• Incision is closed in layers

Battle • Similar location to paramedian but rectus displaced medially (and thus denervated)
• Now seldom used

Kocher's Incision under right subcostal margin e.g. Cholecystectomy (open)


Lanz Incision in right iliac fossa e.g. Appendicectomy
Gridiron Oblique incision centered over McBurneys point- usually appendicectomy (less
cosmetically acceptable than Lanz
Gable Rooftop incision
Pfannenstiel's Transverse supra pubic, primarily used to access pelvic organs
McEvedy's Groin incision e.g. Emergency repair strangulated femoral hernia
Rutherford Extraperitoneal approach to left or right lower quadrants. Gives excellent access to iliac
Morrison vessels and is the approach of choice for first time renal transplantation.
Question 30 of 350
A 59 year old man is undergoing an extended right hemicolectomy for a carcinoma of the hepatic flexure of the
colon. The surgeons divide the middle colonic vein close to its origin. Into which of the following structures
does this vessel primarily drain?

A. Superior mesenteric vein

B. Portal vein

C. Inferior mesenteric vein

D. Inferior vena cava

E. Ileocolic vein

The middle colonic vein drains into the SMV, if avulsed during mobilisation then dramatic haemorrhage can
occur and be difficult to control.

Transverse colon

• The right colon undergoes a sharp turn at the level of the hepatic flexure to become the transverse colon.
• At this point it also becomes intraperitoneal.
• It is connected to the inferior border of the pancreas by the transverse mesocolon.
• The greater omentum is attached to the superior aspect of the transverse colon from which it can easily
be separated. The mesentery contains the middle colic artery and vein. The greater omentum remains
attached to the transverse colon up to the splenic flexure. At this point the colon undergoes another sharp
turn.

Relations
Superior Liver and gall-bladder, the greater curvature of the stomach, and the lower end of the spleen
Inferior Small intestine
Anterior Greater omentum
Posterior From right to left with the descending portion of the duodenum, the head of the pancreas,
convolutions of the jejunum and ileum, spleen

0/3 Question 31-33 of 350


Theme: Nerve Injury

A. Median nerve
B. Ulnar nerve
C. Radial nerve
D. Musculocutaneous nerve
E. Axillary nerve
F. Anterior interosseous nerve
G. Posterior interosseous nerve

For each scenario please select the most likely underlying nerve injury. Each option may be used once, more
than once or not at all.

31. A 19 year old student is admitted to A&E after falling off a wall. He is unable to flex his index finger.
An x-ray confirms a supracondylar fracture.

You answered Ulnar nerve

The correct answer is Median nerve

This median nerve is at risk during a supracondylar fracture.

32. A well toned weight lifter attends clinic reporting weakness of his left arm. There is weakness of
flexion and supination of the forearm.

You answered Radial nerve

The correct answer is Musculocutaneous nerve

Mucocutaneous nerve compression due to entrapment of the nerve between biceps and brachialis.
Elbow flexion and supination of the arm are affected. This is a rare isolated injury.

33. An 18 year old girl sustains an Holstein-Lewis fracture. Which nerve is at risk?

You answered Musculocutaneous nerve

The correct answer is Radial nerve


Proximal lesions affect the triceps. Also paralysis of wrist extensors and forearm supinators occur.
Reduced sensation of dorsoradial aspect of hand and dorsal 31/2 fingers. Holstein-Lewis fractures are
fractures of the distal humerus with radial nerve entrapment.

Question 34 of 350
A 35 year old farm labourer is injures the posterior aspect of his hand with a mechanical scythe. He severs some
of his extensor tendons in this injury. How many tunnels lie in the extensor retinaculum that transmit the
tendons of the extensor muscles?

A. One

B. Three

C. Four

D. Five

E. Six

There are six tunnels, each lined by its own synovial sheath.

Extensor retinaculum

The extensor rentinaculum is a thickening of the deep fascia that stretches across the back of the wrist and holds
the long extensor tendons in position.
Its attachments are:

• The pisiform and hook of hamate medially


• The end of the radius laterally

Structures related to the extensor retinaculum


Structures superficial to the retinaculum • Basilic vein
• Dorsal cutaneous branch of the ulnar nerve
• Cephalic vein
• Superficial branch of the radial nerve

Structures passing deep to the extensor • Extensor carpi ulnaris tendon


retinaculum • Extensor digiti minimi tendon
• Extensor digitorum and extensor indicis tendon
• Extensor pollicis longus tendon
• Extensor carpi radialis longus tendon
• Abductor pollicis longus and extensor pollicis brevis
tendons

Beneath the extensor retinaculum fibrous septa form six compartments that contain the extensor muscle
tendons. Each compartment has its own synovial sheath.

The radial artery


The radial artery passes between the lateral collateral ligament of the wrist joint and the tendons of the abductor
pollicis longus and extensor pollicis brevis.

Image illustrating the topography of tendons passing under the extensor retinaculum

Question 35 of 350
A 23 year old man is stabbed in the chest approximately 10cm below the right nipple. In the emergency
department a abdominal ultrasound scan shows a large amount of intraperitoneal blood. Which of the following
statements relating to the likely site of injury is untrue?

A. Part of its posterior surface is devoid of peritoneum.

B. The quadrate lobe is contained within the functional right lobe.

C. Its nerve supply is from the coeliac plexus.

D. The hepatic flexure of the colon lies posterio-inferiorly.

E. The right kidney is closely related posteriorly.

The right lobe of the liver is the most likely site of injury. Therefore the answer is B as the quadrate lobe is
functionally part of the left lobe of the liver. The liver is largely covered in peritoneum. Posteriorly there is an
area devoid of peritoneum (the bare area of the liver). The right lobe of the liver has the largest bare area (ans is
larger thant the left lobe).

Liver

Structure of the liver


Right lobe • Supplied by right hepatic artery
• Contains Couinard segments V to VIII (-/+Sg I)

Left lobe • Supplied by the left hepatic artery


• Contains Couinard segments II to IV (+/- Sg1)

Quadrate lobe • Part of the right lobe anatomically, functionally is part of the left
• Couinard segment IV
• Porta hepatis lies behind
• On the right lies the gallbladder fossa
• On the left lies the fossa for the umbilical vein

Caudate lobe • Supplied by both right and left hepatic arteries


• Couinard segment I
• Lies behind the plane of the porta hepatis
• Anterior and lateral to the inferior vena cava
• Bile from the caudate lobe drains into both right and left hepatic ducts

Detailed knowledge of Couinard segments is not required for MRCS Part A

• Between the liver lobules are portal canals which contain the portal triad: Hepatic Artery, Portal Vein,
tributary of Bile Duct.

Relations of the liver


Anterior Postero inferiorly
Diaphragm Oesophagus
Xiphoid process Stomach
Duodenum
Hepatic flexure of colon
Right kidney
Gallbladder
Inferior vena cava

Porta hepatis
Location Postero inferior surface, it joins nearly at right angles with the left sagittal fossa, and separates the
caudate lobe behind from the quadrate lobe in front
Transmits • Common hepatic duct
• Hepatic artery
• Portal vein
• Sympathetic and parasympathetic nerve fibres
• Lymphatic drainage of the liver (and nodes)

Ligaments
Falciform ligament • 2 layer fold peritoneum from the umbilicus to anterior liver surface
• Contains ligamentum teres (remnant umbilical vein)
• On superior liver surface it splits into the coronary and left triangular ligaments

Ligamentum teres Joins the left branch of the portal vein in the porta hepatis
Ligamentum Remnant of ductus venosus
venosum

Arterial supply

• Hepatic artery

Venous

• Hepatic veins
• Portal vein

Nervous supply

• Sympathetic and parasympathetic trunks of coeliac plexus

• End and review


• Reference ranges

Question stats

A 33.7%
B 20.6%
C 22.9%
D 11.9%
E 10.9%

33.7% of users answered this question correctly

Search

Score: 38.9%
1
2
3
4
5
6
7
8
9
10
11
12
13-15 2 / 3
16
17
18
19
20
21
22
23
24
25
26
27
28
29
30
31-33 0 / 3
34
35
36
Question 36 of 350
A 22 year old man is involved in a fight and sustains a skull fracture with an injury to the middle meningeal
artery. A craniotomy is performed, and with considerable difficulty the haemorrhage from the middle meningeal
artery is controlled by ligating it close to its origin. What is the most likely sensory impairment that the patient
may notice post operatively?

A. Parasthesia of the ipsilateral external ear

B. Loss of taste sensation from the anterior two thirds of the tongue

C. Parasthesia overlying the angle of the jaw

D. Loss of sensation from the ipsilateral side of the tongue

E. Loss of taste from the posterior two thirds of the tongue

The auriculotemporal nerve is closely related to the middle meningeal artery and may be damaged in this
scenario. The nerve supplied sensation to the external ear and outermost part of the tympanic membrane. The
angle of the jaw is innervated by C2,3 roots and would not be affected. The tongue is supplied by the
glossopharyngeal nerve.

Middle meningeal artery

• Middle meningeal artery is typically the third branch of the first part of the maxillary artery, one of the
two terminal branches of the external carotid artery. After branching off the maxillary artery in the
infratemporal fossa, it runs through the foramen spinosum to supply the dura mater (the outermost
meninges) .
• The middle meningeal artery is the largest of the three (paired) arteries which supply the meninges, the
others being the anterior meningeal artery and the posterior meningeal artery.
• The middle meningeal artery runs beneath the pterion. It is vulnerable to injury at this point, where the
skull is thin. Rupture of the artery may give rise to an extra dural hematoma.
• In the dry cranium, the middle meningeal, which runs within the dura mater surrounding the brain,
makes a deep indention in the calvarium.
• The middle meningeal artery is intimately associated with the auriculotemporal nerve which wraps
around the artery making the two easily identifiable in the dissection of human cadavers and also easily
damaged in surgery.

Question 37 of 350
A 72 year old man presents with haemoptysis and undergoes a bronchoscopy. The carina is noted to be
widened. At which level does the trachea bifurcate?

A. T3

B. T5

C. T7

D. T2

E. T8
The trachea bifurcates at the level of the fifth thoracic vertebra. Or the sixth in tall subjects.

Trachea

Trachea
Location C6 vertebra to the upper border of T5 vertebra (bifurcation)
Arterial and venous supply Inferior thyroid arteries and the thyroid venous plexus.
Nerve Branches of vagus, sympathetic and the recurrent nerves

Relations in the neck


Anterior(Superior to inferior) • Isthmus of the thyroid gland
• Inferior thyroid veins
• Arteria thyroidea ima (when that vessel exists)
• Sternothyroid
• Sternohyoid
• Cervical fascia
• Anastomosing branches between the anterior jugular veins

Posterior Oesophagus.
Laterally • Common carotid arteries
• Right and left lobes of the thyroid gland
• Inferior thyroid arteries
• Recurrent laryngeal nerves

Relations in the thorax

Anterior

• Manubrium, the remains of the thymus, the aortic arch, left common carotid arteries, and the deep
cardiac plexus

Lateral

• In the superior mediastinum, on the right side is the pleura and right vagus; on its left side are the left
recurrent nerve, the aortic arch, and the left common carotid and subclavian arteries.

Question 38 of 350
A 23 year old man is injured during a game of rugby. He suffers a fracture of the distal third of his clavicle, it is
a compound fracture and there is evidence of arterial haemorrhage. Which of the following vessels is most
likely to be encountered first during subsequent surgical exploration?

A. Posterior circumflex humeral artery


B. Axillary artery

C. Thoracoacromial artery

D. Sub scapular artery

E. Lateral thoracic artery

Similar theme in September 2011 Exam

The thoracoacromial artery arises from the second part of the axillary artery. It is a short, wide trunk, which
pierces the clavipectoral fascia, and ends, deep to pectoralis major by dividing into four branches.

Thoracoacromial artery

The thoracoacromial artery (acromiothoracic artery; thoracic axis) is a short trunk, which arises from the
forepart of the axillary artery, its origin being generally overlapped by the upper edge of the Pectoralis minor.

Projecting forward to the upper border of the Pectoralis minor, it pierces the coracoclavicular fascia and divides
into four branches: pectoral, acromial, clavicular, and deltoid.

Branch Description
Pectoral Descends between the two Pectoral muscles, and is distributed to them and to the breast,
branch anastomosing with the intercostal branches of the internal thoracic artery and with the lateral
thoracic.
Acromial Runs laterally over the coracoid process and under the Deltoid, to which it gives branches; it then
branch pierces that muscle and ends on the acromion in an arterial network formed by branches from the
suprascapular, thoracoacromial, and posterior humeral circumflex arteries.
Clavicular Runs upwards and medially to the sternoclavicular joint, supplying this articulation, and the
branch Subclavius
Deltoid Arising with the acromial, it crosses over the Pectoralis minor and passes in the same groove as
branch the cephalic vein, between the Pectoralis major and Deltoid, and gives branches to both muscles.

Question 39 of 350
The following are true of the femoral nerve except:

A. It is derived from L2, L3 and L4 nerve roots

B. It supplies sartorius

C. It supplies quadriceps femoris

D. It gives cutaneous innervations via the saphenous nerve

E. It supplies adductor longus


Adductor longus is supplied by the obturator nerve.

Femoral nerve

Root values L2, 3, 4


Innervates • Pectineus
• Sartorius
• Quadriceps femoris
• Vastus lateralis/medialis/intermedius

Branches • Medial cutaneous nerve of thigh


• Saphenous nerve
• Intermediate cutaneous nerve of thigh

Path
Penetrates psoas major and exits the pelvis by passing under the inguinal ligament to enter the femoral triangle,
lateral to the femoral artery and vein.

Image sourced from Wikipedia

Mnemonic for femoral nerve supply

(don't) M I S V Q Scan for PE


M edial cutaneous nerve of the thigh
I ntermediate cutaneous nerve of the thigh
S aphenous nerve

V astus
Q uadriceps femoris
S artorius

PE ectineus

Question 40 of 350
Where is the vomiting centre located?

A. Medulla oblongata

B. Substantia nigra

C. Antrum of stomach

D. Pons

E. Midbrain
ABC's of Non- GI causes of vomiting

Acute renal failure


Brain (Increased ICP)
Cardiac (Inferior MI)
DKA
Ears (labyrinthitis)
Foreign substances (Tylenol, theo,
etc)
Glaucoma
Hyperemesis Gravidarum
Infections (pyelonephritis, meningitis)

Vomiting

Reflex oral expulsion of gastric (and sometimes intestinal) contents - reverse peristalsis and abdominal
contraction

The vomiting centre is in part of the medulla oblongata and is triggered by receptors in several locations:

• Labyrinthine receptors of ear (motion sickness)


• Overdistention receptors of duodenum and stomach
• Trigger zone of CNS - many drugs (e.g., opiates) act here
• Touch receptors in throat
• Sensory innervation rich, both extrinsic and intrinsic

Question 41 of 350
Which of the following nerves conveys sensory information from the laryngeal mucosa?

A. Glossopharyngeal

B. Laryngeal branches of the vagus

C. Ansa cervicalis

D. Laryngeal branches of the trigeminal

E. None of the above

The laryngeal branches of the vagus supply sensory information from the larynx.

Larynx

The larynx lies in the anterior part of the neck at the levels of C3 to C6 vertebral bodies. The laryngeal skeleton
consists of a number of cartilagenous segments. Three of these are paired; arytenoid, corniculate and cuneiform.
Three are single; thyroid, cricoid and epiglottic. The cricoid cartilage forms a complete ring (the only one to do
so).
The laryngeal cavity extends from the laryngeal inlet to the level of the inferior border of the cricoid cartilage.

Divisions of the laryngeal cavity


Laryngeal vestibule Superior to the vestibular folds
Laryngeal ventricle Lies between vestibular folds and superior to the vocal cords
Infraglottic cavity Extends from vocal cords to inferior border of the cricoid cartilage

The vocal folds (true vocal cords) control sound production. The apex of each fold projects medially into the
laryngeal cavity. Each vocal fold includes:

• Vocal ligament
• Vocalis muscle (most medial part of thyroarytenoid muscle)

The glottis is composed of the vocal folds, processes and rima glottidis. The rima glottidis is the narrowest
potential site within the larynx, as the vocal cords may be completely opposed, forming a complete barrier.

Muscles of the larynx


Muscle Origin Insertion Innervation Action
Posterior Posterior aspect of Muscular process of Recurrent Abducts vocal fold
cricoarytenoid lamina of cricoid arytenoid Laryngeal
Lateral Arch of cricoid Muscular process of Recurrent Adducts vocal fold
cricoarytenoid arytenoid laryngeal
Thyroarytenoid Posterior aspect of Muscular process of Recurrent Relaxes vocal fold
thyroid cartilage arytenoid laryngeal
Transverse and Arytenoid cartilage Contralateral arytenoid Recurrent Closure of
oblique arytenoids laryngeal intercartilagenous part of
the rima glottidis
Vocalis Depression between Vocal ligament and Recurrent Relaxes posterior vocal
lamina of thyroid vocal process of laryngeal ligament, tenses anterior
cartilage arytenoid cartilage part
Cricothyroid Anterolateral part of Inferior margin and External Tenses vocal fold
cricoid horn of thyroid laryngeal
cartilage

Blood supply
Arterial supply is via the laryngeal arteries, branches of the superior and inferior thyroid arteries. The superior
laryngeal artery is closely related to the internal laryngeal nerve. The inferior laryngeal artery is related to the
inferior laryngeal nerve. Venous drainage is via superior and inferior laryngeal veins, the former draining into
the superior thyroid vein and the latter draining into the middle thyroid vein, or thyroid venous plexus.

Lymphatic drainage
The vocal cords have no lymphatic drainage and this site acts as a lymphatic watershed.
Supraglottic part Upper deep cervical nodes
Subglottic part Prelaryngeal and pretracheal nodes and inferior deep cervical nodes
The aryepiglottic fold and vestibular folds have a dense plexus of lymphatics associated with them and
malignancies at these sites have a greater propensity for nodal metastasis.

Topography of the larynx

Question 42 of 350
Which of the following nerves passes through the greater sciatic foramen and innervates the perineum?

A. Pudendal

B. Sciatic

C. Superior gluteal

D. Inferior gluteal

E. Posterior cutaneous nerve of the thigh


3 divisions of the pudendal nerve:

• Rectal nerve
• Perineal nerve
• Dorsal nerve of penis/ clitoris

All these pass through the greater sciatic


foramen.

The pudendal nerve innervates the perineum. It passes between piriformis and coccygeus medial to the sciatic
nerve.

Gluteal region

Gluteal muscles

• Gluteus maximus: inserts to gluteal tuberosity of the femur and iliotibial tract
• Gluteus medius: attach to lateral greater trochanter
• Gluteus minimis: attach to anterior greater trochanter
• All extend and abduct the hip

Deep lateral hip rotators

• Piriformis
• Gemelli
• Obturator internus
• Quadratus femoris

Nerves
Superior gluteal nerve (L5, S1) • Gluteus medius
• Gluteus minimis
• Tensor fascia lata

Inferior gluteal nerve Gluteus maximus


Damage to the superior gluteal nerve will result in the patient developing a Trendelenberg gait. Affected
patients are unable to abduct the thigh at the hip joint. During the stance phase, the weakened abductor muscles
allow the pelvis to tilt down on the opposite side. To compensate, the trunk lurches to the weakened side to
attempt to maintain a level pelvis throughout the gait cycle. The pelvis sags on the opposite side of the lesioned
superior gluteal nerve.

Question 43 of 350
Which of the following is true in relation to the sartorius muscle?

A. Innervated by the deep branch of the femoral nerve

B. Inserts at the fibula

C. It is the shortest muscle in the body

D. Forms the Pes anserinus with Gracilis and semitendinous muscle

E. Causes extension of the knee

It is innervated by the superficial branch of the femoral nerve. It is a component of the pes anserinus.

Sartorius

• Longest strap muscle in the body


• Most superficial muscle in the anterior compartment of the thigh

Origin Anterior superior iliac spine


Insertion Medial surface of the of the body of the tibia (upper part). It inserts anterior to gracilis
and semitendinosus
Nerve Supply Femoral nerve (L2,3)
Action • Flexor of the hip and knee, slight abducts the thigh and rotates it laterally
• It assists with medial rotation of the tibia on the femur. For example it would play
a pivotal role in placing the right heel onto the left knee ( and vice versa)

Important The middle third of this muscle, and its strong underlying fascia forms the roof of the
relations adductor canal , in which lie the femoral vessels, the saphenous nerve and the nerve to
vastus medialis.
2/3 Question 44-46 of 350
Theme: Nerve lesions

A. Sciatic nerve
B. Peroneal nerve
C. Tibial Nerve
D. Obturator nerve
E. Ilioinguinal nerve
F. Femoral nerve
G. None of the above

Please select the most likely nerve injury for the scenario given. Each option may be used once, more than once
or not at all

44. A 56 year old man undergoes a low anterior resection with legs in the Lloyd-Davies position. Post
operatively he complains of foot drop.

Peroneal nerve

Positioning legs in Lloyd- Davies stirrups can carry the risk of peroneal nerve neuropraxia if not done
carefully.

45. A 23 year old man complains of severe groin pain several weeks after a difficult inguinal hernia
repair.

Ilioinguinal nerve

The ilioinguinal nerve may have been entrapped in the mesh causing a neuroma.

46. A 72 year old man develops a foot drop after a revision total hip replacement.

You answered Femoral nerve

The correct answer is Sciatic nerve

This may be done by a number of approaches, in this scenario a posterior approach is the most likely
culprit.

Lower limb- Muscular compartments

Anterior compartment
Muscle Nerve Action
Tibialis anterior Deep peroneal nerve Dorsiflexes ankle joint, inverts foot
Extensor digitorum longus Deep peroneal nerve Extends lateral four toes, dorsiflexes ankle joint
Peroneus tertius Deep peroneal nerve Dorsiflexes ankle, everts foot
Extensor hallucis longus Deep peroneal nerve Dorsiflexes ankle joint, extends big toe

Peroneal compartment
Muscle Nerve Action
Peroneus longus Superficial peroneal nerve Everts foot, assists in plantar flexion
Peroneus brevis Superficial peroneal nerve Plantar flexes the ankle joint
Superficial posterior compartment
muscle Nerve Action
Gastrocnemius Tibial nerve Plantar flexes the foot, may also flex
the knee
Soleus Tibial nerve Plantar flexor

Deep posterior compartment


Muscle Nerve Action
Flexor digitorum longus Tibial Flexes the lateral four toes
Flexor hallucis longus Tibial Flexes the great toe
Tibialis posterior Tibial Plantar flexor, inverts the foot

Question 47 of 350
A 68 year old man falls onto an outstretched hand. Following the accident he is examined in the emergency
department. On palpating his anatomical snuffbox there is tenderness noted in the base. What is the most likely
injury in this scenario?

A. Rupture of the tendon of flexor pollicis

B. Scaphoid fracture

C. Distal radius fracture

D. Rupture of flexor carpi ulnaris tendon

E. None of the above

A fall onto an outstretched hand is a common mechanism of injury for a scaphoid fracture. This should be
suspected clinically if there is tenderness in the base of the anatomical snuffbox. A tendon rupture would not
result in bony tenderness.

Question 48 of 350
A 25 year old man sustains a severe middle cranial fossa basal skull fracture. Once he has recovered it is
noticed that he has impaired tear secretion. This is most likely to be the result of damage to which of the
following?

A. Stellate ganglion

B. Ciliary ganglion

C. Otic ganglion

D. Trigeminal nerve
E. Greater petrosal nerve

The greater petrosal nerve may be injured and carries fibres for lacrimation (see below).

Lacrimal system

Lacrimal gland
Consists of an orbital part and palpebral part. They are continuous posterolaterally around the concave lateral
edge of the levator palpebrae superioris muscle.
The ducts of the lacrimal gland open into the superior fornix. Those from the orbital part penetrate the
aponeurosis of levator palpebrae superioris to join those from the palpebral part. Therefore excision of the
palpebral part is functionally similar to excision of the entire gland.

Blood supply
Lacrimal branch of the opthalmic artery. Venous drainage is to the superior opthalmic vein.

Innervation
The gland is innervated by the secretomotor paraympathetic fibres from the pterygopalatine ganglion which in
turn may reach the gland via the zygomatic or lacrimal branches of the maxillary nerve or pass directly to the
gland. The preganglionic fibres travel to the ganglion in the greater petrosal nerve (a branch of the facial nerve
at the geniculate ganglion).

Nasolacrimal duct
Descends from the lacrimal sac to open anteriorly in the inferior meatus of the nose.

Lacrimation reflex
Occurs in response to conjunctival irritation (or emotional events). The conjunctiva will send signals via the
opthalmic nerve. These then pass to the superior salivary centre. The efferent signals pass via the greater
petrosal nerve (parasympathetic preganglionic fibres) and the deep petrosal nerve which carries the post
ganglionic sympathetic fibres. The parasympathetic fibres will relay in the pterygopalatine ganglion, the
sympathetic fibres do not synapse. They in turn will relay to the lacrimal apparatus.

Question 49 of 350
Which of the following structures passes through the quadrangular space near the humeral head?

A. Axillary artery

B. Radial nerve

C. Axillary nerve

D. Median nerve

E. Transverse scapular artery

The quadrangular space is bordered by the humerus laterally, subscapularis superiorly, teres major inferiorly
and the long head of triceps medially. It lies lateral to the triangular space. It transmits the axillary nerve and
posterior circumflex humeral artery.

Image sourced from Wikipedia

Shoulder joint

• Shallow synovial ball and socket type of joint.


• It is an inherently unstable joint, but is capable to a wide range of movement.
• Stability is provided by muscles of the rotator cuff that pass from the scapula to insert in the greater
tuberosity (all except sub scapularis-lesser tuberosity).

Glenoid labrum

• Fibrocartilaginous rim attached to the free edge of the glenoid cavity


• Tendon of the long head of biceps arises from within the joint from the supraglenoid tubercle, and is
fused at this point to the labrum.
• The long head of triceps attaches to the infraglenoid tubercle

Fibrous capsule

• Attaches to the scapula external to the glenoid labrum and to the labrum itself (postero-superiorly)
• Attaches to the humerus at the level of the anatomical neck superiorly and the surgical neck inferiorly
• Anteriorly the capsule is in contact with the tendon of subscapularis, superiorly with the supraspinatus
tendon, and posteriorly with the tendons of infraspinatus and teres minor. All these blend with the
capsule towards their insertion.
• Two defects in the fibrous capsule; superiorly for the tendon of biceps. Anteriorly there is a defect
beneath the subscapularis tendon.
• The inferior extension of the capsule is closely related to the axillary nerve at the surgical neck and this
nerve is at risk in anteroinferior dislocations. It also means that proximally sited osteomyelitis may
progress to septic arthritis.
Movements and muscles
Flexion Anterior part of deltoid
Pectoralis major
Biceps
Coracobrachialis
Extension Posterior deltoid
Teres major
Latissimus dorsi
Adduction Pectoralis major
Latissimus dorsi
Teres major
Coracobrachialis
Abduction Mid deltoid
Supraspinatus
Medial rotation Subscapularis
Anterior deltoid
Teres major
Latissimus dorsi
Lateral rotation Posterior deltoid
Infraspinatus
Teres minor

Important anatomical relations


Anteriorly Brachial plexus
Axillary artery and vein
Posterior Suprascapular nerve
Suprascapular vessels
Inferior Axillary nerve
Circumflex humeral vessels

Question 50 of 350
Which of the following pairings of foramina and their contents is not correct?

A. Superior orbital fissure and the oculomotor nerve

B. Foramina rotundum and the maxillary nerve

C. Jugular foramen and the hypoglossal nerve

D. Foramina spinosum and the middle meningeal artery

E. Foramina lacerum and the internal carotid artery

The hypoglossal nerve passes through the hypoglossal canal.


Question 51 of 350
A 55 year old man with carcinoma of the larynx is undergoing a difficult laryngectomy. The surgeons divide the
thyrocervical trunk, from which of the following vessels does this structure most commonly originate?

A. Subclavian artery

B. Common carotid artery

C. Vertebral artery

D. External carotid artery

E. Internal carotid artery

The thyrocervical trunk is a branch of the subclavian artery. It arises from the first part between the subclavian
artery and the inner border of scalenus anterior. It branches off the subclavian distal to the vertebral artery.

Root of the neck

Thoracic Outlet

• Where the subclavian artery and vein and the brachial plexus exit the thorax and enter the arm.
• They pass over the 1st rib and under the clavicle.
• The subclavian vein is the most anterior structure and is immediately anterior to scalenus anterior and its
attachment to the first rib.
• Scalenus anterior has 2 parts, the subclavian artery leaves the thorax by passing over the first rib and
between these 2 portions of the muscle.
• At the level of the first rib, the lower cervical nerve roots combine to form the 3 trunks of the brachial
plexus. The lowest trunk is formed by the union of C8 and T1, and this trunk lies directly posterior to the
artery and is in contact with the superior surface of the first rib.

Thoracic outlet obstruction causes neurovascular compromise.

Question 52 of 350
The following structures are closely related to the brachiocephalic artery except:

A. Trachea posteriorly

B. Right brachiocephalic vein

C. Inferior thyroid vein

D. Right recurrent laryngeal nerve

E. None of the above


There is no brachiocephalic artery on the left, however the left brachiocephalic vein lies anteriorly to the roots
of all the 3 great arteries (including the brachiocephalic artery). The right recurrent laryngeal nerve has no
relation to the brachiocephalic artery.

Question 53 of 350
Which of the following structures separates the ulnar artery from the median nerve?

A. Brachioradialis

B. Pronator teres

C. Tendon of biceps brachii

D. Flexor carpi ulnaris

E. Brachialis

It lies deep to pronator teres and this separates it from the median nerve.

Ulnar artery

Path

• Starts: middle of antecubital fossa


• Passes obliquely downward, reaching the ulnar side of the forearm at a point about midway between the
elbow and the wrist. It follows the ulnar border to the wrist, crossing over the flexor retinaculum. It then
divides into the superficial and deep volar arches.

Relations
Deep to- Pronator teres, Flexor carpi radialis, Palmaris longus
Lies on- Brachialis and Flexor digitorum profundus
Superficial to the flexor retinaculum at the wrist

The median nerve is in relation with the medial side of the artery for about 2.5 cm. And then crosses the vessel,
being separated from it by the ulnar head of the Pronator teres

The ulnar nerve lies medially to the lower two-thirds of the artery

Branch

• Anterior interosseous artery


Question 54 of 350
Which muscle is supplied by the superficial peroneal nerve?

A. Peroneus tertius

B. Sartorius

C. Adductor magnus

D. Peroneus brevis

E. Gracilis

Superficial peroneal nerve

Supplies

• Lateral compartment of leg: peroneus longus, peroneus brevis (action: eversion and plantar flexion)
• Sensation over dorsum of the foot (except the first web space, which is innervated by the deep peroneal
nerve)
Path

• Passes between peroneus longus and peroneus brevis along the length of the proximal one third of the
fibula
• 10-12 cm above the tip of the lateral malleolus, the superficial peroneal nerve pierces the fascia
• 6-7 cm distal to the fibula, the superficial peroneal nerve bifurcates into intermediate and medial dorsal
cutaneous nerves

Question 55 of 350
A 32 year old motorcyclist is involved in a road traffic accident. His humerus is fractured and severely
displaced. At the time of surgical repair the surgeon notes that the radial nerve has been injured. Which of the
following muscles is least likely to be affected by an injury at this site?

A. Extensor carpi radialis brevis

B. Brachioradialis

C. Abductor pollicis longus

D. Extensor pollicis brevis

E. None of the above


Muscles supplied by the radial
nerve

BEST
Brachioradialis
Extensors
Supinator
Triceps

The radial nerve supplies the extensor muscles, abductor pollicis longus and extensor pollicis brevis (the latter
two being innervated by the posterior interosseous branch of the radial nerve).

Radial nerve

Continuation of posterior cord of the brachial plexus (root values C5 to T1)

Path

• In the axilla: lies posterior to the axillary artery on subscapularis, latissimus dorsi and teres major.
• Enters the arm between the brachial artery and the long head of triceps (medial to humerus).
• Spirals around the posterior surface of the humerus in the groove for the radial nerve.
• At the distal third of the lateral border of the humerus it then pierces the intermuscular septum and
descends in front of the lateral epicondyle.
• At the lateral epicondyle it lies deeply between brachialis and brachioradialis where it then divides into a
superficial and deep terminal branch.
• Deep branch crosses the supinator to become the posterior interosseous nerve.

In the image below the relationships of the radial nerve can be appreciated

Image sourced from Wikipedia

Regions innervated
Motor (main nerve) • Triceps
• Anconeus
• Brachioradialis
• Extensor carpi radialis

Motor (posterior • Extensor carpi ulnaris


interosseous branch • Extensor digitorum
• Extensor indicis
• Extensor digiti minimi
• Extensor pollicis longus and brevis
• Abductor pollicis longus

Sensory The area of skin supplying the proximal phalanges on the dorsal aspect of the hand is
supplied by the radial nerve (this does not apply to the little finger and part of the ring
finger)

Muscular innervation and effect of denervation


Anatomical Muscle affected Effect of paralysis
location
Shoulder Long head of triceps Minor effects on shoulder stability in abduction
Arm Triceps Loss of elbow extension
Forearm Supinator Weakening of supination of prone hand and elbow flexion
Brachioradialis in mid prone position
Extensor carpi radialis longus
and brevis

The cutaneous sensation of the upper limb- illustrating the contribution of the radial nerve

Question 56 of 350
A man develops an infection in his external auditory meatus. The infection is extremely painful. Which of the
following nerves conveys sensation from this region?

A. Occipital branch of the trigeminal nerve

B. Vestibulocochlear nerve

C. Facial nerve

D. Auriculotemporal nerve

E. Maxillary branch of the trigeminal nerve


Tensor tympania and stapedius are the only two muscles of the middle ear. Contraction of tensor tympani will
tend to dampen the vibrations produced by loud sounds, it is innervated by a branch of the trigeminal nerve.
The stapedius dampens movements of the ossicles in response to loud sounds and is innervated by a branch of
the facial nerve.

The auriculotemporal nerve, which is derived from the mandibular branch of the trigeminal nerve supplies this
area.
Ear- anatomy

The ear is composed of three anatomically distinct regions.

External ear
Auricle is composed of elastic cartilage covered by skin. The lobule has no cartilage and contains fat and
fibrous tissue.

External auditory meatus is approximately 2.5cm long.


Lateral third of the external auditory meatus is cartilaginous and the medial two thirds is bony.

The region is innervated by the greater auricular nerve. The auriculotemporal branch of the trigeminal nerve
supplies most the of external auditory meatus and the lateral surface of the auricle.

Middle ear
Space between the tympanic membrane and cochlea. The aditus leads to the mastoid air cells is the route
through which middle ear infections may cause mastoiditis. Anteriorly the eustacian tube connects the middle
ear to the naso pharynx.
The tympanic membrane consists of:

• Outer layer of stratified squamous epithelium.


• Middle layer of fibrous tissue.
• Inner layer of mucous membrane continuous with the middle ear.

The tympanic membrane is approximately 1cm in diameter.


The chorda tympani nerve passes on the medial side of the pars flaccida.

The middle ear is innervated by the glossopharyngeal nerve and pain may radiate to the middle ear following
tonsillectomy.

Ossicles
Malleus attaches to the tympanic membrane (the Umbo).
Malleus articulates with the incus (synovial joint).
Incus attaches to stapes (another synovial joint).

Internal ear
Cochlea, semi circular canals and vestibule

Organ of corti is the sense organ of hearing and is located on the inside of the cochlear duct on the basilar
membrane.

Vestibule accommodates the utricule and the saccule. These structures contain endolymph and are surrounded
by perilymph within the vestibule.

The semicircular canals lie at various angles to the petrous temporal bone. All share a common opening into the
vestibule.

Question 57 of 350
Which muscle is responsible for causing flexion of the interphalangeal joint of the thumb?

A. Flexor pollicis longus

B. Flexor pollicis brevis

C. Flexor digitorum superficialis

D. Flexor digitorum profundus

E. Adductor pollicis

There are 8 muscles:


1. Two flexors (flexor pollicis brevis and flexor pollicis longus)
2. Two extensors (extensor pollicis brevis and longus)
3. Two abductors (abductor pollicis brevis and longus)
4. One adductor (adductor pollicis)
5. One muscle that opposes the thumb by rotating the CMC joint (opponens pollicis).

Flexor and extensor longus insert on the distal phalanx moving both the MCP and IP joints.

Hand

Anatomy of the hand


Bones • 8 Carpal bones
• 5 Metacarpals
• 14 phalanges

Intrinsic Muscles 7 Interossei - Supplied by ulnar nerve

• 3 palmar-adduct fingers
• 4 dorsal- abduct fingers

Intrinsic muscles Lumbricals

• Flex MCPJ and extend the IPJ.


• Origin deep flexor tendon and insertion dorsal extensor hood mechanism.
• Innervation: 1st and 2nd- median nerve, 3rd and 4th- deep branch of the ulnar
nerve.

Thenar eminence • Abductor pollicis brevis


• Opponens pollicis
• Flexor pollicis brevis

Hypothenar • Opponens digiti minimi


eminence • Flexor digiti minimi brevis
• Abductor digiti minimi
Question 58 of 350
Which of the following structures separates the posterior cruciate ligament from the popliteal artery?

A. Oblique popliteal ligament

B. Transverse ligament

C. Popliteus tendon

D. Biceps femoris

E. Semitendinosus

The posterior cruciate ligament is separated from the popliteal vessels at its origin by the oblique popliteal
ligament. The transverse ligament is located anteriorly.

Knee joint

The knee joint is a synovial joint, the largest and most complicated. It consists of two condylar joints between
the femure and tibia and a sellar joint between the patella and the femur. The tibiofemoral articular surfaces are
incongruent, however, this is improved by the presence of the menisci. The degree of congruence is related to
the anatomical position of the knee joint and is greatest in full extension.

Knee joint compartments


Tibiofemoral • Comprise of the patella/femur joint, lateral and medial compartments (between femur
condyles and tibia)
• Synovial membrane and cruciate ligaments partially separate the medial and lateral
compartments

Patellofemoral • Ligamentum patellae


• Actions: provides joint stability in full extension

Fibrous capsule
The capsule of the knee joint is a complex, composite structure with contributions from adjacent tendons.
Anterior The capsule does not pass proximal to the patella. It blends with the tendinous expansions of
fibres vastus medialis and lateralis
Posterior These fibres are vertical and run from the posterior surface of the femoral condyles to the
fibres posterior aspect of the tibial condyle
Medial fibres Attach to the femoral and tibial condyles beyond their articular margins, blending with the
tibial collateral ligament
Lateral fibres Attach to the femur superior to popliteus, pass over its tendon to head of fibula and tibial
condyle

Bursae
Anterior • Subcutaneous prepatellar bursa; between patella and skin
• Deep infrapatellar bursa; between tibia and patellar ligament
• Subcutaneous infrapatellar bursa; between distal tibial tuberosity and skin

Laterally • Bursa between lateral head of gastrocnemius and joint capsule


• Bursa between fibular collateral ligament and tendon of biceps femoris
• Bursa between fibular collateral ligament and tendon of popliteus

Medially • Bursa between medial head of gastrocnemius and the fibrous capsule
• Bursa between tibial collateral ligament and tendons of sartorius, gracilis and semitendinosus
• Bursa between the tendon of semimembranosus and medial tibial condyle and medial head of
gastrocnemius

Posterior Highly variable and inconsistent

Ligaments
Medial collateral Medial epicondyle femur to medial tibial condyle: valgus stability
ligament
Lateral collateral Lateral epicondyle femur to fibula head: varus stability
ligament
Anterior cruciate Anterior tibia to lateral intercondylar notch femur: prevents tibia sliding anteriorly
ligament
Posterior cruciate Posterior tibia to medial intercondylar notch femur: prevents tibia sliding
ligament posteriorly
Patellar ligament Central band of the tendon of quadriceps femoris, extends from patella to tibial
tuberosity
Image sourced from Wikipedia

Image sourced from Wikipedia

Menisci
Medial and lateral menisci compensate for the incongruence of the femoral and tibial condyles.
Composed of fibrous tissue.
Medial meniscus is attached to the tibial collateral ligament.
Lateral meniscus is attached to the loose fibres at the lateral edge of the joint and is separate from the fibular
collateral ligament. The lateral meniscus is crossed by the popliteus tendon.

Nerve supply
The knee joint is supplied by the femoral, tibial and common peroneal divisions of the sciatic and by a branch
from the obturator nerve. Hip pathology pain may be referred to the knee.

Blood supply
Genicular branches of the femoral artery, popliteal and anterior tibial arteries all supply the knee joint.

Question 59 of 350
How many compartments are there in the lower leg?

A. 2

B. 1

C. 3

D. 5

E. 4

The deep compartment of the lower leg has both superficial and deep posterior layers, together with the anterior
and lateral compartments this allows for four compartments. Decompression of the deep posterior compartment
during fasciotomy may be overlooked with significant sequelae.

Fascial compartments of the leg

Compartments of the thigh

Formed by 3 septae passing from the femur to the fascia lata.


Compartment Nerve Muscles Blood supply
Anterior Femoral • Iliacus Femoral artery
compartment • Tensor fasciae latae
• Sartorius
• Quadriceps femoris

Medial compartment Obturator • Adductor longus/magnus/brevis Profunda femoris artery and


• Gracilis obturator artery
• Obturator externus

Posterior Sciatic • Semimembranosus Branches of Profunda femoris artery


compartment • Semitendinosus
• Biceps femoris
Compartments of the lower leg
Separated by the interosseous membrane (anterior and posterior compartments), anterior fascial septum
(separate anterior and lateral compartments) and posterior fascial septum (separate lateral and posterior
compartments)

Compartment Nerve Muscles Blood supply


Anterior Deep peroneal • Tibialis anterior Anterior tibial
compartment nerve • Extensor digitorum longus artery
• Extensor hallucis longus
• Peroneus tertius

Posterior Tibial • Muscles: deep and superficial compartments Posterior tibial


compartment (separated by deep transverse fascia)
• Deep: Flexor hallucis longus, Flexor digitalis
longus, Tibialis posterior, Popliteus
• Superficial: Gastrocnemius, Soleus, Plantaris

Lateral Superficial • Peroneus longus/brevis Anterior tibial


compartment peroneal

Question 60 of 350
Which structure is not at the level of the sternal angle?

A. Left brachiocephalic vein

B. Intervertebral discs T4-T5

C. Start of aortic arch

D. 2nd pair of costal cartilages

E. Bifurcation of the trachea into left and right bronchi

The left brachiocephalic vein lies posterior to the manubrium, at the level of its upper border. The sternal angle
refers to the transition between manubrium and sternum and therefore will not include the left brachiocephalic
vein.

Sternal angle

Anatomical structures at the level of the manubrium and upper sternum


Upper part of the manubrium • Left brachiocephalic vein
• Brachiocephalic artery
• Left common carotid
• Left subclavian artery
Lower part of the manubrium/ manubrio- • Costal cartilages of the 2nd ribs
sternal angle • Transition point between superior and inferior
mediastinum
• Arch of the aorta
• Tracheal bifurcation
• Union of the azygos vein and superior vena cava
• The thoracic duct crosses to the midline

Question 61 of 350
A 53 year old man is undergoing a left hemicolectomy for carcinoma of the descending colon. From which
embryological structure is this region of the gastrointestinal tract derived?

A. Vitellino-intestinal duct

B. Hind gut

C. Mid gut

D. Fore gut

E. Woolffian duct

The left colon is embryologically part of the hind gut. Which accounts for its separate blood supply via the
IMA.

Colon anatomy

The colon commences with the caecum. This represents the most dilated segment of the human colon and its
base (which is intraperitoneal) is marked by the convergence of teniae coli. At this point is located the
vermiform appendix. The colon continues as the ascending colon, the posterior aspect of which is
retroperitoneal. The line of demarcation between the intra and retro peritoneal right colon is visible as a white
line, in the living, and forms the line of incision for colonic resections.
The ascending colon becomes the transverse colon after passing the hepatic flexure. At this located the colon
becomes wholly intra peritoneal once again. The superior aspect of the transverse colon is the point of
attachment of the transverse colon to the greater omentum. This is an important anatomical site since division of
these attachments permits entry into the lesser sac. Separation of the greater omentum from the transverse colon
is a routine operative step in both gastric and colonic resections.
At the left side of the abdomen the transverse colon passes to the left upper quadrant and makes an oblique
inferior turn at the splenic flexure. Following this, the posterior aspect becomes retroperitoneal once again.
At the level of approximately L4 the descending colon becomes wholly intraperitoneal and becomes the
sigmoid colon. Whilst the sigmoid is wholly intraperitoneal there are usually attachments laterally between the
sigmoid and the lateral pelvic sidewall. These small congenital adhesions are not formal anatomical attachments
but frequently require division during surgical resections.
At is distal end the sigmoid passes to the midline and at the region around the sacral promontary it becomes the
upper rectum. This transition is visible macroscopically as point where the teniae fuse. More distally the rectum
passes through the peritoneum at the region of the peritoneal reflection and becomes extraperitoneal.
Arterial supply
Superior mesenteric artery and inferior mesenteric artery: linked by the marginal artery.
Ascending colon: ileocolic and right colic arteries
Transverse colon: middle colic artery
Descending and sigmoid colon: inferior mesenteric artery

Venous drainage
From regional veins (that accompany arteries) to superior and inferior mesenteric vein

Lymphatic drainage
Initially along nodal chains that accompany supplying arteries, then para-aortic nodes.

Embryology
Midgut- Second part of duodenum to 2/3 transverse colon
Hindgut- Distal 1/3 transverse colon to anus

Peritoneal location
The right and left colon are part intraperitoneal and part extraperitoneal. The sigmoid and transverse colon are
generally wholly intraperitoneal. This has implications for the sequelae of perforations, which will tend to result
in generalised peritonitis in the wholly intra peritoneal segments.

Colonic relations
Region of colon Relation
Caecum/ right colon Right ureter, gonadal vessels
Hepatic flexure Gallbladder (medially)
Splenic flexure Spleen and tail of pancreas
Distal sigmoid/ upper rectum Left ureter
Rectum Ureters, autonomic nerves, seminal vesicles, prostate, urethra (distally)

Question 62 of 350
What is the most useful test to clinically distinguish between an upper and lower motor neurone lesion of the
facial nerve?

A. Blow cheeks out

B. Loss of chin reflex

C. Close eye

D. Raise eyebrow

E. Open mouth against resistance


Upper motor neurone lesions of the facial nerve- Paralysis of the lower half of face.
Lower motor neurone lesion- Paralysis of the entire ipsilateral face.

Theme from April 2012 Exam


Upper Vs Lower motor neurone lesions - Facial nerve

The nucleus of the facial nerve is located in the caudal aspect of the ventrolateral pontine tegmentum. Its axons
exit the ventral pons medial to the spinal trigeminal nucleus.

Any lesion occurring within or affecting the corticobulbar tract is known as an upper motor neuron lesion. Any
lesion affecting the individual branches (temporal, zygomatic, buccal, mandibular and cervical) is known as a
lower motor neuron lesion.

Branches of the facial nerve leaving the facial motor nucleus (FMN) for the muscles do so via both left and
right posterior (dorsal) and anterior (ventral) routes. In other words, this means lower motor neurons of the
facial nerve can leave either from the left anterior, left posterior, right anterior or right posterior facial motor
nucleus. The temporal branch travels out from the left and right posterior components. The inferior four
branches do so via the left and right anterior components. The left and right branches supply their respective
sides of the face (ipsilateral innervation). Accordingly, the posterior components receive motor input from both
hemispheres of the cerebral cortex (bilaterally), whereas the anterior components receive strictly contralateral
input. This means that the temporal branch of the facial nerve receives motor input from both hemispheres of
the cerebral cortex whereas the zygomatic, buccal, mandibular and cervical branches receive information from
only contralateral hemispheres.

Now, because the anterior FMN receives only contralateral cortical input whereas the posterior receives that
which is bilateral, a corticobulbar lesion (UMN lesion) occurring in the left hemisphere would eliminate motor
input to the right anterior FMN component, thus removing signaling to the inferior four facial nerve branches,
thereby paralyzing the right mid- and lower-face. The posterior component, however, although now only
receiving input from the right hemisphere, is still able to allow the temporal branch to sufficiently innervate the
entire forehead. This means that the forehead will not be paralyzed.

The same mechanism applies for an upper motor neuron lesion in the right hemisphere. The left anterior FMN
component no longer receives cortical motor input due to its strict contralateral innervation, whereas the
posterior component is still sufficiently supplied by the left hemisphere. The result is paralysis of the left mid-
and lower-face with an unaffected forehead.

On the other hand, a lower motor neuron lesion is a bit different.

A lesion on either the left or right side would affect both the anterior and posterior routes on that side because of
their close physical proximity to one another. So, a lesion on the left side would inhibit muscle innervation from
both the left posterior and anterior routes, thus paralyzing the whole left side of the face (Bells Palsy). With this
type of lesion, the bilateral and contalateral inputs of the posterior and anterior routes, respectively, become
irrelevant because the lesion is below the level of the medulla and the facial motor nucleus. Whereas at a level
above the medulla a lesion occurring in one hemisphere would mean that the other hemisphere could still
sufficiently innervate the posterior facial motor nucleus, a lesion affecting a lower motor neuron would
eliminate innervation altogether because the nerves no longer have a means to receive compensatory
contralateral input at a downstream decussation.

Question 63 of 350
An 18 year old man is stabbed in the axilla during a fight. His axillary artery is lacerated and repaired.
However, the surgeon neglects to repair an associated injury to the upper trunk of the brachial plexus. Which of
the following muscles is least likely to demonstrate impaired function as a result?
A. Palmar interossei

B. Infraspinatus

C. Brachialis

D. Supinator brevis

E. None of the above

The palmar interossei are supplied by the ulnar nerve. Which lies inferiorly and is therefore less likely to be
injured.

Question 64 of 350
A 23 year old man is involved in a fight, during the dispute he sustains a laceration to the posterior aspect of his
right arm, approximately 2cm proximal to the olecranon process. On assessment in the emergency department
he is unable to extend his elbow joint. Which of the following tendons is most likely to have been cut?

A. Triceps

B. Pronator teres

C. Brachioradialis

D. Brachialis

E. Biceps

Theme from 2009 Exam

The triceps muscle extends the elbow joint. The other muscles listed all produce flexion of the elbow joint.

Triceps

Origin • Long head- infraglenoid tubercle of the scapula.


• Lateral head- dorsal surface of the humerus, lateral and proximal to the groove of the
radial nerve
• Medial head- posterior surface of the humerus on the inferomedial side of the radial
groove and both of the intermuscular septae

Insertion • Olecranon process of the ulna. Here the olecranon bursa is between the triceps tendon and
olecranon.
• Some fibres insert to the deep fascia of the forearm, posterior capsule of the elbow
(preventing the capsule from being trapped between olecranon and olecranon fossa during
extension)
Innervation Radial nerve
Blood Profunda brachii artery
supply
Action Elbow extension. The long head can adduct the humerus and and extend it from a flexed position
Relations The radial nerve and profunda brachii vessels lie between the lateral and medial heads

Question 65 of 350
A 25 year old man undergoes an excision of a pelvic chondrosarcoma, during the operation the obturator nerve
is sacrificed. Which of the following muscles is least likely to be affected as a result?

A. Adductor longus

B. Pectineus

C. Adductor magnus

D. Sartorius

E. Gracilis

Sartorius is supplied by the femoral nerve. In approximately 20% of the population, pectineus is supplied by the
accessory obturator nerve.

Obturator nerve

The obturator nerve arises from L2, L3 and L4 by branches from the ventral divisions of each of these nerve
roots. L3 forms the main contribution and the second lumbar branch is occasionally absent. These branches
unite in the substance of psoas major, descending vertically in its posterior part to emerge from its medial
border at the lateral margin of the sacrum. It then crosses the sacroiliac joint to enter the lesser pelvis, it
descends on obturator internus to enter the obturator groove. In the lesser pelvis the nerve lies lateral to the
internal iliac vessels and ureter, and is joined by the obturator vessels lateral to the ovary or ductus deferens.

Supplies

• Medial compartment of thigh


• Muscles supplied: external obturator, adductor longus, adductor brevis, adductor magnus (not the lower
part-sciatic nerve), gracilis
• The cutaneous branch is often absent. When present, it passes between gracilis and adductor longus near
the middle part of the thigh, and supplies the skin and fascia of the distal two thirds of the medial aspect.

Obturator canal

• Connects the pelvis and thigh: contains the obturator artery, vein, nerve which divides into anterior and
posterior branches.

Cadaveric cross section demonstrating relationships of the obturator nerve


Question 66 of 350
You excitedly embark on your first laparoscopic cholecystectomy and during the operation the anatomy of
Calots triangle is more hostile than anticipated. Whilst trying to apply a haemostatic clip you avulse the cystic
artery. This is followed by brisk haemorrhage. From which source is this most likely to originate ?

A. Right hepatic artery

B. Portal vein

C. Gastroduodenal artery

D. Liver bed

E. Common hepatic artery

The cystic artery is a branch of the right hepatic artery. There are recognised variations in the anatomy of the
blood supply to the gallbladder. However, the commonest situation is for the cystic artery to branch from the
right hepatic artery.

Gallbladder

• Fibromuscular sac with capacity of 50ml


• Columnar epithelium

Relations of the gallbladder


Anterior Liver
Posterior • Covered by peritoneum
• Transverse colon
• 1st part of the duodenum

Laterally Right lobe of liver


Medially Quadrate lobe of liver

Arterial supply
Cystic artery (branch of Right hepatic artery)

Venous drainage
Cystic vein

Nerve supply
Sympathetic- mid thoracic spinal cord, Parasympathetic- anterior vagal trunk

Common bile duct

Origin Confluence of cystic and common hepatic ducts


Relations at origin • Medially - Hepatic artery
• Posteriorly- Portal vein

Relations distally • Duodenum - anteriorly


• Pancreas - medially and laterally
• Right renal vein - posteriorly

Arterial supply Branches of hepatic artery and retroduodenal branches of gastroduodenal artery

Calot's triangle

Medially Common hepatic duct


Inferiorly Cystic duct
Superiorly Inferior edge of liver
Contents Cystic artery

Question 67 of 350
A 43 year old man suffers a pelvic fracture which is complicated by an injury to the junction of the membranous
urethra to the bulbar urethra. In which of the following directions is the extravasated urine most likely to pass?

A. Posteriorly into extra peritoneal tissues

B. Laterally into the buttocks

C. Into the abdomen

D. Anteriorly into the connective tissues surrounding the scrotum

E. None of the above


The superficial perineal pouch is a compartment bounded superficially by the superficial perineal fascia, deep
by the perineal membrane (inferior fascia of the urogenital diaphragm), and laterally by the ischiopubic ramus.
It contains the crura of the penis or clitoris, muscles, viscera, blood vessels, nerves, the proximal part of the
spongy urethra in males, and the greater vestibular glands in females.
When urethral rupture occurs as in this case the urine will tend to pass anteriorly because the fascial
condensations will prevent lateral and posterior passage of the urine.

Urogenital triangle

The urogenital triangle is formed by the:

• Ischiopubic inferior rami


• Ischial tuberosities

A fascial sheet is attached to the sides, forming the inferior fascia of the urogenital diaphragm.

It transmits the urethra in males and both the urethra and vagina in females. The membranous urethra lies deep
this structure and is surrounded by the external urethral sphincter.

Superficial to the urogenital diaphragm lies the superficial perineal pouch. In males this contains:

• Bulb of penis
• Crura of the penis
• Superficial transverse perineal muscle
• Posterior scrotal arteries
• Posterior scrotal nerves

In females the internal pudendal artery branches to become the posterior labial arteries in the superficial
perineal pouch.

Question 68 of 350
Which of the following does not pass through the superior orbital fissure?

A. Oculomotor nerve

B. Abducens nerve

C. Ophthalmic artery

D. Ophthalmic division of the trigeminal nerve

E. Ophthalmic veins

The ophthalmic artery, a branch of the internal carotid enters the orbit with the optic nerve in the canal.
Question 69 of 350
Which of the following muscles does not attach to the radius?

A. Pronator quadratus

B. Biceps

C. Brachioradialis

D. Supinator

E. Brachialis

The brachialis muscle inserts into the ulna. The other muscles are all inserted onto the radius.

Radius

• Bone of the forearm extending from the lateral side of the elbow to the thumb side of the wrist

Upper end

• Articular cartilage- covers medial > lateral side


• Articulates with radial notch of the ulna by the annular ligament
• Muscle attachment- biceps brachii at the tuberosity

Shaft

• Muscle attachment-

Upper third of the body Supinator, Flexor digitorum superficialis, Flexor pollicis longus
Middle third of the body Pronator teres
Lower quarter of the body Pronator quadratus , tendon of supinator longus

Lower end

• Quadrilateral
• Anterior surface- capsule of wrist joint
• Medial surface- head of ulna
• Lateral surface- ends in the styloid process
• Posterior surface: 3 grooves containing:

1. Tendons of extensor carpi radialis longus and brevis


2. Tendon of extensor pollicis longus
3. Tendon of extensor indicis
Question 70 of 350
Which nerve supplies the 1st web space of the foot?

A. Popliteal nerve

B. Superficial peroneal nerve

C. Deep peroneal nerve

D. Tibial nerve

E. Saphenous nerve
The first web space is innervated by the deep peroneal nerve. See diagram below:

Image sourced from Wikipedia

Deep peroneal nerve

Origin From the common peroneal nerve, at the lateral aspect of the fibula, deep to peroneus
longus
Nerve root values L4, L5, S1, S2
Course and relation • Pierces the anterior intermuscular septum to enter the anterior compartment of
the lower leg
• Passes anteriorly down to the ankle joint, midway between the two malleoli

Terminates In the dorsum of the foot


Muscles innervated • Tibialis anterior
• Extensor hallucis longus
• Extensor digitorum longus
• Peroneus tertius
• Extensor digitorum brevis

Cutaneous Web space of the first and second toes


innervation
Actions • Dorsiflexion of ankle joint
• Extension of all toes (extensor hallucis longus and extensor digitorum longus)
• Eversion of the foot

After its bifurcation past the ankle joint, the lateral branch of the deep peroneal nerve innervates the extensor
digitorum brevis and the extensor hallucis brevis
The medial branch supplies the web space between the first and second digits.

Question 71 of 350
During the course of a radical gastrectomy the surgeons detach the omentum and ligate the right gastro-epiploic
artery. From which vessel does it originate?

A. Superior mesenteric artery

B. Inferior mesenteric artery

C. Coeliac axis

D. Common hepatic artery

E. Gastroduodenal artery

Theme from January 2013 Exam


The gastroduodenal artery arises at the superior part of the duodenum and descends behind it to terminate at its
lower border. It terminates by dividing into the right gastro-epiploic artery and the superior pancreaticoduodenal
artery. The right gastro-opiploic artery passes to the left and passes between the layers of the greater omentum
to anastomose with the left gastro-epiploic artery.

Gastroduodenal artery

Supplies
Pylorus, proximal part of the duodenum, and indirectly to the pancreatic head (via the anterior and posterior
superior pancreaticoduodenal arteries)

Path
Most commonly arises from the common hepatic artery of the coeliac trunk
Terminates by bifurcating into the right gastroepiploic artery and the superior pancreaticoduodenal artery

Image showing stomach reflected superiorly to illustrate the relationship of the gastroduodenal artery to the
first part of the duodenum
Question 72 of 350
Which of the following is not an intrinsic muscle of the hand?

A. Opponens pollicis

B. Palmaris longus

C. Flexor pollicis brevis

D. Flexor digiti minimi brevis

E. Opponens digiti minimi


Mnemonic for intrinsic hand muscles
'A OF A OF A'

A bductor pollicis brevis


O pponens pollicis
F lexor pollicis brevis
A dductor pollicis (thenar muscles)
O pponens digiti minimi
F lexor digiti minimi brevis
A bductor digiti minimi (hypothenar
muscles)
Palmaris longus originates in the forearm.

Hand

Anatomy of the hand


Bones • 8 Carpal bones
• 5 Metacarpals
• 14 phalanges

Intrinsic Muscles 7 Interossei - Supplied by ulnar nerve

• 3 palmar-adduct fingers
• 4 dorsal- abduct fingers

Intrinsic muscles Lumbricals

• Flex MCPJ and extend the IPJ.


• Origin deep flexor tendon and insertion dorsal extensor hood mechanism.
• Innervation: 1st and 2nd- median nerve, 3rd and 4th- deep branch of the ulnar
nerve.

Thenar eminence • Abductor pollicis brevis


• Opponens pollicis
• Flexor pollicis brevis

Hypothenar • Opponens digiti minimi


eminence • Flexor digiti minimi brevis
• Abductor digiti minimi
Question 73 of 350
A man with lung cancer and bone metastasis in the thoracic spinal vertebral bodies, sustains a pathological
fracture at the level of T4. The fracture is unstable and the spinal cord is severely compressed at this level.
Which of the findings below will not be present?

A. Extensor plantar reflexes

B. Spasticity of the lower limbs

C. Diminished patellar tendon reflex

D. Urinary incontinence

E. Sensory ataxia

A thoracic cord lesion causes spastic paraperesis, hyperrflexia and extensor plantar responses (UMN lesion),
incontinence, sensory loss below the lesion and 'sensory' ataxia.

Spinal cord

• Located in a canal within the vertebral column that affords it structural support.
• Rostrally is continues to the medulla oblongata of the brain and caudally it tapers at a level
corresponding to the L1-2 interspace (in the adult), a central structure, the filum terminale anchors the
cord to the first coccygeal vertebra.
• The spinal cord is characterised by cervico-lumbar enlargements and these, broadly speaking, are the
sites which correspond to the brachial and lumbar plexuses respectively.

There are some key points to note when considering the surgical anatomy of the spinal cord:

* During foetal growth the spinal cord becomes shorter than the spinal canal, hence the adult site of cord
termination at the L1-2 level.

* Due to growth of the vertebral column the spine segmental levels may not always correspond to bony
landmarks as they do in the cervical spine.

* The spinal cord is incompletely divided into two symmetrical halves by a dorsal median sulcus and ventral
median fissure. Grey matter surrounds a central canal that is continuous rostrally with the ventricular system of
the CNS.

* The grey matter is sub divided cytoarchitecturally into Rexeds laminae.

* Afferent fibres entering through the dorsal roots usually terminate near their point of entry but may travel for
varying distances in Lissauers tract. In this way they may establish synaptic connections over several levels

* At the tip of the dorsal horn are afferents associated with nociceptive stimuli. The ventral horn contains
neurones that innervate skeletal muscle.

The key point to remember when revising CNS anatomy is to keep a clinical perspective in mind. So it is worth
classifying the ways in which the spinal cord may become injured. These include:

• Trauma either direct or as a result of disc protrusion


• Neoplasia either by direct invasion (rare) or as a result of pathological vertebral fracture
• Inflammatory diseases such as Rheumatoid disease, or OA (formation of osteophytes compressing
nerve roots etc.
• Vascular either as a result of stroke (rare in cord) or as complication of aortic dissection
• Infection historically diseases such as TB, epidural abscesses.

The anatomy of the cord will, to an extent dictate the clinical presentation. Some points/ conditions to
remember:

• Brown- Sequard syndrome-Hemisection of the cord producing ipsilateral loss of proprioception and
upper motor neurone signs, plus contralateral loss of pain and temperature sensation. The explanation of
this is that the fibres decussate at different levels.
• Lesions below L1 will tend to present with lower motor neurone signs

Question 74 of 350
Through which of the following foramina does the genital branch of the genitofemoral nerve exit the abdominal
cavity?

A. Superficial inguinal ring

B. Sciatic notch
C. Obturator foramen

D. Femoral canal

E. Deep inguinal ring

The genitofemoral nerve divides into two branches as it approaches the inguinal ligament. The genital branch
passes anterior to the external iliac artery through the deep inguinal ring into the inguinal canal. It
communicates with the ilioinguinal nerve in the inguinal canal (though this is seldom of clinical significance).

Genitofemoral nerve

Supplies
- Small area of the upper medial thigh

Path
- Arises from the first and second lumbar nerves
- Passes obliquely through Psoas major, and emerges from its medial border opposite the fibrocartilage between
the third and fourth lumbar vertebrae.
- It then descends on the surface of Psoas major, under cover of the peritoneum
- Divides into genital and femoral branches.
- The genital branch passes through the inguinal canal, within the spermatic cord, to supply the skin overlying
the skin and fascia of the scrotum. The femoral branch enters the thigh posterior to the inguinal ligament, lateral
to the femoral artery. It supplies an area of skin and fascia over the femoral triangle.

• It may be injured during abdominal or pelvic surgery, or during inguinal hernia repairs.

Question 75 of 350
A 28 year old man lacerates the posterolateral aspect of his wrist with a knife in an attempted suicide. On arrival
in the emergency department the wound is inspected and found to be located over the lateral aspect of the
extensor retinaculum (which is intact). Which of the following structures is at greatest risk of injury?

A. Superficial branch of the radial nerve

B. Radial artery

C. Dorsal branch of the ulnar nerve

D. Tendon of extensor carpi radialis brevis

E. Tendon of extensor digiti minimi

The superficial branch of the radial nerve passes superior to the extensor retinaculum in the position of this
laceration and is at greatest risk of injury. The dorsal branch of the ulnar nerve and artery also pass superior to
the extensor retinaculum n but are located medially.
Question 76 of 350
A 43 year old man is reviewed in the clinic following a cardiac operation. A chest x-ray is performed and a
circular radio-opaque structure is noted medial to the 4th interspace on the left. Which of the following
procedures is the patient most likely to have undergone?

A. Aortic valve replacement with metallic valve

B. Tricuspid valve replacement with metallic valve

C. Tricuspid valve replacement with porcine valve

D. Pulmonary valve replacement with porcine valve

E. Mitral valve replacement with metallic valve

Theme from April 2012 Exam

Prosthetic heart valves on Chest X-rays

The aortic and mitral valves are most commonly replaced and when a metallic valve is used, can be most
readily identified on plain x-rays.
The presence of cardiac disease (such as cardiomegaly) may affect the figures quoted here.

Aortic
Usually located medial to the 3rd interspace on the right.

Mitral
Usually located medial to the 4th interspace on the left.

Tricuspid
Usually located medial to the 5th interspace on the right.

Please note that these are the sites at which an artificial valve may be located and are NOT the sites of
auscultation.

Question 77 of 350
A 63 year old lady is diagnosed as having an endometrial carcinoma arising from the uterine body. To which
nodal region will the tumour initially metastasise?

A. Para aortic nodes

B. Iliac lymph nodes

C. Inguinal nodes

D. Pre sacral nodes


E. Mesorectal lymph nodes

Theme from 2011 exam


Tumours of the uterine body will tend to spread to the iliac nodes initially. When the tumour is expanding to
cross different nodal margins this is of considerable clinical significance if nodal clearance is performed during
a Wertheims type hysterectomy.

Lymphatic drainage of the uterus and cervix

• The uterine fundus has a lymphatic drainage that runs with the ovarian vessels and may thus drain to the
para-aortic nodes. Some drainage may also pass along the round ligament to the inguinal nodes.
• The body of the uterus drains through lymphatics contained within the broad ligament to the iliac lymph
nodes.
• The cervix drains into three potential nodal stations; laterally through the broad ligament to the external
iliac nodes, along the lymphatics of the uterosacral fold to the presacral nodes and posterolaterally along
lymphatics lying alongside the uterine vessels to the internal iliac nodes.

Question 78 of 350
Transection of the radial nerve at the level of the axilla will result in all of the following except:

A. Loss of elbow extension.

B. Loss of extension of the interphalangeal joints.

C. Loss of metacarpophalangeal extension.

D. Loss of triceps reflex.

E. Loss of sensation overlying the first dorsal interosseous.

These may still extend by virtue of retained lumbrical muscle function.

Question 79 of 350
Which of the following structures is not located in the superficial perineal space in females?

A. Posterior labial arteries

B. Pudendal nerve

C. Superficial transverse perineal muscle

D. Greater vestibular glands

E. None of the above


The pudendal nerve is located in the deep perineal space and then branches to innervate more superficial
structures.

Urogenital triangle

The urogenital triangle is formed by the:

• Ischiopubic inferior rami


• Ischial tuberosities

A fascial sheet is attached to the sides, forming the inferior fascia of the urogenital diaphragm.

It transmits the urethra in males and both the urethra and vagina in females. The membranous urethra lies deep
this structure and is surrounded by the external urethral sphincter.

Superficial to the urogenital diaphragm lies the superficial perineal pouch. In males this contains:

• Bulb of penis
• Crura of the penis
• Superficial transverse perineal muscle
• Posterior scrotal arteries
• Posterior scrotal nerves

In females the internal pudendal artery branches to become the posterior labial arteries in the superficial
perineal pouch.

Question 80 of 350
Which of the following is not a branch of the hepatic artery?

A. Pancreatic artery

B. Cystic artery

C. Right gastric artery

D. Right hepatic artery

E. Right Gastroepiploic artery

The pancreatic artery is a branch of the splenic artery.

Question 81 of 350
Which of the following structures does not pass behind the piriformis muscle in the greater sciatic foramen?
A. Sciatic nerve

B. Posterior cutaneous nerve of the thigh

C. Inferior gluteal artery

D. Obturator nerve

E. None of the above

The obturator nerve does not pass through the greater sciatic foramen.

Greater sciatic foramen

Contents
Nerves • Sciatic Nerve
• Superior and Inferior Gluteal Nerves
• Internal Pudendal Nerve
• Posterior Femoral Cutaneous Nerve
• Nerve to Quadratus Femoris
• Nerve to Obturator internus

Vessels • Superior Gluteal Artery and vein


• Inferior Gluteal Artery and vein
• Internal Pudendal Artery and vein

Piriformis
The piriformis is a landmark for identifying structures passing out of the sciatic notch

• Above piriformis: Superior gluteal vessels


• Below piriformis: Inferior gluteal vessels, sciatic nerve (10% pass through it, <1% above it), posterior
cutaneous nerve of the thigh

Greater sciatic foramen boundaries


Anterolaterally Greater sciatic notch of the ilium
Posteromedially Sacrotuberous ligament
Inferior Sacrospinous ligament and the ischial spine
Superior Anterior sacroiliac ligament

The greater sciatic foramen


Image sourced from Wikipedia

Structures passing between both foramina (Medial to lateral)

• Pudendal nerve
• Internal pudendal artery
• Nerve to obturator internus

Contents of the lesser sciatic foramen

• Tendon of the obturator internus


• Pudendal nerve
• Internal pudendal artery and vein
• Nerve to the obturator internus

Question 82 of 350
A 56 year old man is undergoing a nephrectomy. The surgeons divide the renal artery. At what level do these
usually branch off the abdominal aorta?

A. T9

B. L2
C. L3

D. T10

E. L4

The renal arteries usually branch off the aorta on a level with L2.

Renal arteries

• The right renal artery is longer than the left renal artery
• The renal vein/artery/pelvis enter the kidney at the hilum

Relations

• Right:

Anterior- IVC, right renal vein, the head of the pancreas, and the descending part of the duodenum.

• Left:

Anterior- left renal vein, the tail of the pancreas.

Branches

• The renal arteries are direct branches off the aorta (upper border of L2)
• In 30% there may be accessory arteries (mainly left side). Instead of entering the kidney at the hilum,
they usually pierce the upper or lower part of the organ.
• Before reaching the hilum of the kidney, each artery divides into four or five segmental branches (renal
vein anterior and ureter posterior); which then divide within the sinus into lobar arteries supplying each
pyramid and cortex.
• Each vessel gives off some small inferior suprarenal branches to the suprarenal gland, the ureter, and the
surrounding cellular tissue and muscles.

You might also like